Archer Maternal & Newborn Health

Ace your homework & exams now with Quizwiz!

A client presents to the obstetrics floor at 39 weeks gestation with irregular contractions. After you get the client situated in a labor, delivery, and recovery room, you notice the client's health care provider (HCP) enter the room to evaluate the client. Following the evaluation, the HCP exits the room, and shortly thereafter, you enter. During your discussion, the client states the HCP "went to order oxytocin." In anticipation of that order, you understand this client's oxytocin will be administered via which route of administration? A. Intramuscular administration B. Intravenous administration via mainline infusion using an infusion pump C. Intravenous administration via piggyback using an infusion pump D. Oral administration Submit Answer

A client presents to the obstetrics floor at 39 weeks gestation with irregular contractions. After you get the client situated in a labor, delivery, and recovery room, you notice the client's health care provider (HCP) enter the room to evaluate the client. Following the evaluation, the HCP exits the room, and shortly thereafter, you enter. During your discussion, the client states the HCP "went to order oxytocin." In anticipation of that order, you understand this client's oxytocin will be administered via which route of administration? A. Intramuscular administration B. Intravenous administration via mainline infusion using an infusion pump C. Intravenous administration via piggyback using an infusion pump D. Oral administration Submit Answer

The nurse performs an obstetrics history on a client seeking antenatal care for a confirmed pregnancy. The client reports having six-year-old twins born at 39 gestational weeks. She had an ectopic pregnancy at four gestational weeks that occurred sixteen weeks ago. When completing the client's documentation, the nurse should record the GTPAL as A. G = 3, T = 1, P = 0, A = 1, L = 2 B. G = 3, T = 2, P = 0, A = 1, L = 2 C. G = 4, T = 2, P = 0, A = 1, L = 2 D. G = 3, T = 2, P = 0, A = 1, L = 1 Submit Answer

Explanation Choice A is correct. ➢ The client's total number of pregnancies (including the current one) is three. This is documented under gravida (G). She is currently pregnant, had one ectopic pregnancy, and was pregnant with twins. ➢ The client has had one-term birth: the twins were born at 39 gestational weeks. This is documented under the term (T). Term is any birth after 37 gestational weeks. ➢ The client has not had any preterm (P) births (any birth between 20 and 37 gestational weeks). ➢ The client has had one ectopic pregnancy, which is documented as an abortion (A). ➢ The client has two living (L) children, the twins, who were born at 39 gestational weeks. Additional Info ✓ A method for calculating gravida and para is to separate pregnancies and their outcome using the acronym GTPAL: G = gravida, T = term, P = preterm, A = abortions, and L = living children. ✓ G = pregnancies or gravida, T = term pregnancies delivered, P = preterm pregnancies delivered, A = abortions (spontaneous and induced), and L = living children ✓ Term pregnancies are any pregnancy 37 weeks or greater; preterm is any births between 20-37 weeks; abortions are any abortions spontaneous or induced before 20 weeks. Last Updated - 24, Jan 2023

The nurse is providing education to a client who is pregnant about a nonstress test (NST). Which of the following statements by the client would require a follow-up? A. "I cannot have anything to eat eight hours prior to this test." B. "I will have an external monitor across my abdomen." C. "A reactive result means my baby is doing well." D. "If this test is abnormal, I will need further testing." Submit Answer

Explanation Choice A is correct. A Nonstress test does not require a client to abstain from eating or drinking before the test. This statement is false and would require follow-up. Choices B, C, and D are incorrect. These statements are true and do not require follow-up. An external fetal heart rate monitor will be applied across the client's abdomen during this test, and results that are reactive indicate fetal well-being. Abnormal testing does require additional testing, such as a biophysical profile or a contraction stress test. Additional Info Source : Archer Review A nonstress test is performed in the third trimester if the client has indications such as a high-risk pregnancy that may result in a stillbirth or complications such as fetal hypoxia. A reactive finding indicates fetal well-being; specifically, the fetal heart rate increased by 15 beats per minute, lasting for 15 seconds. A nonreactive NST is non-reassuring and indicates decreased variability with an absence in a fetal heart rate acceleration. Last Updated - 22, Nov 2022

The nurse working in the maternity ward is caring for a 24-hour post-partum client. When assessing the client, the nurse notes that her fundus is firm at the level of the umbilicus and is veering a little bit to the right. The initial action for the nurse is to: A. Check for bladder distention B. Check the client's blood pressure C. Check if the client has been given oxytocin D. Check the pad count Submit Answer

Explanation Choice A is correct. A displaced fundus is an indication of a distended bladder. The nurse should assess the client for bladder distention and encourage the client to empty her bladder. Choice B is incorrect. The nurse can check the client's blood pressure; however, this action is unrelated to the situation. Choice C is incorrect. The nurse has assessed that the client's fundus is firm. Thus, the nurse does not need to check if oxytocin was given. Choice D is incorrect. The nurse does not need to check the client's pad count as the client is not showing any signs of uterine atony. Last Updated - 27, Jan 2022

The nurse is caring for four newborns during her shift in the unit. After performing an assessment, which newborn should the nurse give her attention to? A. A 24-hour old newborn that has not yet passed meconium. B. A 3-day old infant with mild jaundice and a bilirubin of 3 mg/dL. C. A 3-hour old infant that has just passed meconium. D. A 5-day old infant with a positive Babinski reflex. Submit Answer

Explanation Choice A is correct. A newborn that has not yet passed meconium after 24 hours should be evaluated for Hirschsprung's disease. Choice B is incorrect. An infant with slight jaundice after the first 24 to 48 hours of life should not cause concern to the nurse. The physician should be notified if the disease occurs within the early 24 hours to evaluate if the jaundice is pathological. Jaundice is pathologic if it presents within the first 24 hours after birth, the total serum bilirubin level rises by more than 5 mg/dL/day or absolute level at anytime higher than 17 mg/dL, or an infant has signs and symptoms suggestive of serious illness. Choice C is incorrect. This is entirely normal as meconium is expected to be passed within the first 24 hours of the child's life. Choice D is incorrect. The Babinski reflex is a primitive reflex that is present in newborns. The nurse should not be concerned about this assessment finding. Last Updated - 16, Jan 202

An experienced nurse is caring for a 24-hour old newborn in the nursery. She suspects asphyxia in utero. All of the following assessment findings would indicate asphyxia in utero, except: A. There is a present palmar-grasp reflex. B. The nurse strokes the sole of the newborn's feet but there is no response. C. The neonate is unresponsive when the nurse claps her hands above him. D. The neonate has weak and ineffective sucking. Submit Answer

Explanation Choice A is correct. A present palmar-grasp reflex indicates that there is an intact neurologic response from the neonate. Choice B is incorrect. Asphyxia in utero manifests as neurological damage in the neonate. Neurological damage is seen as absent or depressed reflexes of the newborn. This absence in Babinski reflex indicates asphyxia in utero. Choice C is incorrect. Asphyxia in utero manifests as neurological damage in the neonate. Neurological damage is seen as absent or depressed reflexes of the newborn. This absence in the Startle reflex indicates asphyxia in utero. Choice D is incorrect. Asphyxia in utero manifests as neurological damage in the neonate. Neurological damage is seen as absent or depressed reflexes of the newborn. This absence or depression in the sucking reflex indicates asphyxia in utero. Last Updated - 13, Feb 2022

The nurse is caring for a primigravida client with the following clinical data. The nurse should take which of the following actions based on the result? See the exhibit. A. Inform the client of the normal finding B. Prepare the client for a contraction stress test C. Arrange for a repeat test D. Inquire if the client ate prior to the test Submit Answer

Explanation Choice A is correct. A reactive NST is an expected finding and indicates fetal well-being. Choices B, C, and D are incorrect. A reactive nonstress test is a normal finding, and the client will not need a contraction stress test (CST). A CST is only utilized if the NST is abnormal. A normal finding will not require repeat testing. A client may or may not eat before the test. Dietary restrictions are not necessary before an NST. Additional Info A nonstress test is performed in the third trimester if the client has indications such as a high-risk pregnancy that may result in a stillbirth or complications such as fetal hypoxia. A reactive finding indicates fetal well-being; specifically, the fetal heart rate increased by 15 beats per minute, lasting for 15 seconds. A nonreactive NST is non-reassuring and indicates decreased variability with an absence in a fetal heart rate acceleration.

The nurse in the delivery room has just delivered a newborn from a 41-year-old mother. Which assessment finding would lead the nurse to suspect Down syndrome in the child? A. Simian crease B. Jitteriness C. Acrocyanosis D. Uneven skin folds in the buttocks when the legs are extended Submit Answer

Explanation Choice A is correct. A simian crease, also known as a transverse palmar crease, is one of the characteristic signs of Down syndrome. Choice B is incorrect. Jitteriness may indicate a drug-dependent newborn. Choice C is incorrect. Acrocyanosis is cyanosis of the extremities in the newborn. This is a normal occurrence in newborns. Choice D is incorrect. This sign is characteristic of congenital hip dislocation. Last Updated - 11, Oct 2021

The nurse is caring for a pregnant client with a decreased alpha-fetoprotein level. The nurse plans to A. notify the primary healthcare provider (PHCP) of the results. B. instruct the client to increase their intake of folic acid. C. document the result as normal. D. obtain a urine specimen. Submit Answer

Explanation Choice A is correct. AFP is a substance made by the baby that enters the amniotic fluid and the mother's bloodstream. A small amount of AFP is usually found in amniotic fluid and the mother's blood. When the amount is high, it signals the physician to look further for the possibility of a neural tube defect. A woman carrying a baby with Down syndrome may have lower blood levels of AFP. High levels may suggest neural tube defects or be caused by a multifetal pregnancy. Regardless of the etiology of the alterations in the level, the nurse needs to notify the PHCP of the result. Choices B, C, and D are incorrect. A neural tube defect can be detected with MSAFP, but an increase in folic acid will not change once the error has occurred. Taking folic acid before becoming pregnant and continuing through the pregnancy can be beneficial to prevent neural tube defects. Obtaining a urine specimen is not needed and is required if the woman is showing signs of pre-eclampsia. Documenting the result as normal is inappropriate. Additional Info Folic acid supplementation prevents neural tube defects (spina bifida, meningocele, myelomeningocele). Foods rich in folic acid include dark green leafy vegetables, whole grains, and grapefruit. Last Updated - 25, Nov 2022

A 28-year-old female presents to the obstetrics office, suspecting she may be pregnant. Which of the following would the nurse recognize as a presumptive sign of pregnancy? A. Amenorrhea B. Positive fetal cardiac activity on ultrasound C. Enlarged uterus D. Auscultation of fetal heart tones Submit Answer

Explanation Choice A is correct. Amenorrhea (absence of menstrual period) is a presumptive sign of pregnancy. Presumptive signs of pregnancy are symptoms and signs that the patient experiences. Presumptive signs may resemble pregnancy signs and symptoms but may also be caused by other etiologies. While amenorrhea is a presumptive sign of pregnancy, missing a period can also result from other conditions such as stress, hypothyroidism, and anorexia. Choices B and D are incorrect. A positive cardiac activity on ultrasound (Choice B) and auscultation of fetal heart tones (Choice D) would be a positive sign of pregnancy. Positive signs of pregnancy are signs that cannot, under any circumstances, be mistaken for other conditions. Positive signs confirm that the pregnancy has occurred. Choice C is incorrect. An enlarged uterus would be detected by the examiner and would be a probable sign of pregnancy. Probable pregnancy signs indicate pregnancy in most cases; however, there is still the chance they can be caused by conditions other than pregnancy. While an enlarged uterus is a probable pregnancy sign, other conditions such as uterine tumors, fibroids, and adenomyomas may also cause such a finding. Another example of a probable sign of pregnancy is a positive urine pregnancy test. NCSBN Client Need Topic: Pregnancy, Subtopic: Antepartum care, system-specific assessment Additional Info Source : Archer Review Last Updated - 16, Feb 2022

The postpartum nurse is monitoring a new mother for signs of illness following vaginal delivery of a newborn infant. Which of the following is an early sign of excessive blood loss? A. Heart rate change from 80 to 125 bpm B. Blood pressure change from 125/90 to 119/82 mmHg C. A decrease in respiratory rate from 22 to 16 breaths per minute D. Reports of perineal soreness Submit Answer

Explanation Choice A is correct. An early sign of illness involves an increase in the patient's heart rate. Tachycardia is a rapid response to hypovolemia. A heart rate change from 80 to 125 bpm warrants further investigation into a possible illness. Choice B is incorrect. A blood pressure change from 125/90 to 119/82 mmHg is not a significant drop enough to indicate hypovolemia or considerable hemorrhage. Shock is defined as mean arterial pressure less than 65 mmHg or systolic blood pressure less than 90 mmHg. Choice C is incorrect. A respiratory rate decrease from 22 to 16 breaths per minute is an expected finding after delivery and is not alarming. Moreover, significant bleeding or hemorrhagic shock is associated with tachypnea (increase in the respiratory rate), not a decrease. Choice D is incorrect. Some perineal soreness is normal after delivery and does not necessarily indicate impending hemorrhage. NCSBN client need Topic: Health Maintenance and Promotion, Postpartum Care Last Updated - 31, Jan 2022

Chadwick's sign is a prenatal assessment performed at the initial visit to verify pregnancy. The obstetric nurse knows that Chadwick's sign presents with: A. A blue to purplish hue of the cervix B. Softness to the uterine fundus felt through the abdomen C. A thinning and lengthening of the cervix D. The absence of menstruation at day 28 in a woman's cycle Submit Answer

Explanation Choice A is correct. Chadwick's sign presents as a blue to purple hue of the cervix and is considered a "probable" sign of pregnancy. Choice B is incorrect. A soft uterus is not used to verify pregnancy. Choice C is incorrect. The thinning and lengthening of the cervix is known as effacement and occurs just before or during labor. Choice D is incorrect. The absence of menstruation is not an assessment performed by the health care provider. Instead, it is an observation a woman will report. Additional Info Source : Archer Review Last Updated - 13, Feb 2022

A nurse is caring for a woman that just had a normal delivery an hour ago. The nurse understands that the patient is still at risk for uterine atony at this stage. All of the following interventions should be included in the care plan of the patient for detection of uterine atony, except: A. Checking for saturated perineal pads every shift B. Palpating the fundus at frequent intervals C. Weighing perineal pads once they are changed, noting the time it was changed and the saturation D. Checking vital signs frequently for signs of shock Submit Answer

Explanation Choice A is correct. Checking perineal pads every shift is an incorrect practice and therefore the correct answer to this question. The nurse should assess the perineal pad of the immediate post-partum woman every 30-minutes, not every turn. Perineal pads getting soaked with blood within 30 minutes should be a cause of concern for the nurse for this is a sign of continuous bleeding through the uterus due to uterine atony. Choice B is incorrect. Palpating the fundus frequently is correct practice. The nurse should palpate the patient's fundus frequently to make sure that it is firm and contracted. A firm and contracted uterus prevents blood loss. Choice C is incorrect. Weighing used perineal pads once they are changed is correct practice. The nurse should weigh the perineal pads after they are soaked to accurately assess the amount of blood lost by the patient through the perineum. One gram in weight is equivalent to 1 mL in plasma. Taking note of the time that the pads were changed would signify the frequency of pad changes, which is also essential in the assessment. Choice D is incorrect. Checking vital signs for signs of shock is correct nursing practice. The nurse should assess the patient frequently for signs of trauma. These include low blood pressure, weak, thready pulses, increased heart rate, and increased respiratory rate. Last Updated - 27, Jan 2022

Risk factors for preeclampsia include all of the following, except: A. Chronic hypotension B. Age C. Race D. Family history of preeclampsia

Explanation Choice A is correct. Chronic hypotension is not a risk factor for preeclampsia; therefore, this is the correct answer to the question. Instead, a history of high blood pressure is a risk factor. This hypertension is defined as a blood pressure reading above 140/90 mmHg. Choices B, C, and D are incorrect. Older and very young pregnant women are at higher risk. African-American women are at higher risk than other races. A personal or family history of preeclampsia increases the risk of preeclampsia for a woman. NCSBN Client Need Topic: Reduction of Risk Potential, Sub-topic: Potential for Alterations in Body Systems; Antepartum Additional Info Source : Archer Review Last Updated - 10, Nov 2022

What intervention is appropriate for the nurse to teach her pregnant patient about relieving constipation? A. Increasing the consumption of fruits and vegetables B. Taking a mild over-the-counter laxative C. Lying flat on the back when sleeping D. Reduce the consumption of iron by at least ½ Submit Answer

Explanation Choice A is correct. Dietary roughage (or fiber) with sufficient fluids and exercise may help relieve constipation. Constipation in pregnant women is thought to occur due to hormones that relax the intestinal muscle and by the pressure of the expanding uterus on the intestines. Relaxation of the intestinal muscle causes food and waste to move slower through your system. Sometimes iron tablets may contribute to constipation. Choice B is incorrect. Over-the-counter medications should be avoided during pregnancy. Choice C is incorrect. The supine position can place additional pressure on the aorta and vena cava, leading to vena cava syndrome. Choice D is incorrect. A reduction of iron supplements during pregnancy may reduce hemoglobin production and result in a less than effective immune system. NCSBN Client Need Topic: Physiological Integrity, Subtopic: Basic Care and Comfort Last Updated - 28, Oct 2021

A 35-week pregnant client comes into the emergency room concerned that she had not felt her baby kick for 3 days now. She also complains to the nurse that she had black stools in the morning. The nurse notices bruising in the client's arm and elbows. The nurse should suspect: A. Disseminated intravascular coagulation (DIC) B. Abuse C. Abruptio placentae D. Sepsis Submit Answer

Explanation Choice A is correct. Disseminated intravascular coagulation (DIC) is a maternal condition in which the clotting cascade is activated, resulting in clots in the microcirculation. The patient states that she has not felt her baby kick or move for three days. This could mean that her fetus has died in utero, and an intrauterine fetal death is a predisposing condition for DIC. Other symptoms like black stools and bruising are indicative of bleeding. The nurse should suspect DIC and initiate appropriate measures. Choice B is incorrect. All client symptoms are indicative of DIC; however, abuse should not be ruled out and requires further assessment by the nurse. Choice C is incorrect. Signs of an abruptio placentae include a contracted and firm uterus as well as a slowed fetal heart rate. The patient does not manifest any of these signs or symptoms. Choice D is incorrect. The client is manifesting signs of DIC; however, sepsis may arise from the intrauterine fetal death and should be removed through the delivery of the demised fetus. Additional Info Source : Archer Review Last Updated - 16, Oct 2021

The nurse is discussing sudden infant death syndrome (SIDS) with the parents of a newborn. Which of the following statements, if made by the parents, would require follow-up? A. "I have been keeping my baby warm with extra blankets while he sleeps." B. "I give my baby a pacifier at night while he sleeps." C. "I am keeping my baby up to date on his scheduled vaccinations." D. "I replaced my baby's sheepskin bedding with a firm mattress." Submit Answer

Explanation Choice A is correct. Environmental factors such as soft mattresses/bedding, blankets, bumper pads affixed to the crib, and accessories in the crib increase the risk for SIDS dramatically. The recommended sleeping position for an infant is supine, with no stuffed animals, blankets, or accessories. Choices B, C, and D are incorrect. Evidence has shown that a baby sleeping with a pacifier is not a risk factor for SIDS. Current vaccinations decrease the risk of SIDS. Soft mattresses such as sheepskin should not be used. It was appropriate for the parents to modify the infant's mattress to a firm one. Additional Info ➢ SIDS occurs higher in males and twins. ➢ The peak incidence is between two and four months. ➢ Other risk factors include exposure to cigarette smoke, prematurity, prone sleeping position, and overheating. ➢ Mitigating factors include a cool sleeping environment, supine sleeping, room-sharing (not bed-sharing), and sleeping on firm surfaces. Last Updated - 11, Dec 2022

The nurse is preparing a client who is prescribed continuous epidural analgesia. Which of the following pre-procedure prescriptions should the nurse be prepared to administer? A. Lactated Ringers B. Ondansetron C. Ketorolac D. Haloperidol Submit Answer

Explanation Choice A is correct. Epidural analgesia has a common adverse reaction of hypotension. It is routine for clients being prepared for epidural analgesia to receive a liter of isotonic fluids to preempt this adverse reaction. Choices B, C, and D are incorrect. While epidural analgesia may cause nausea and vomiting, it is not routine to administer pre-procedure antiemetics such as ondansetron. Ketorolac is an antiinflammatory indicated for mild to moderate pain. This is not a medication indicated for an epidural. Finally, haloperidol is a typical (first generation) antipsychotic and has no relevance to an epidural. Additional Info For lumbar epidural analgesia, the common site selected is between L3 to L4. Essential monitoring parameters include the area for signs of infection, blood pressure (may cause hypotension), and urinary output (may cause urinary retention). Last Updated - 06, Jun 2022

A woman in her 30th week of gestation was brought into the emergency department for falling down a flight of stairs. On evaluation, the physician notes a rigid, board-like abdomen. FHR is 167 bpm; with otherwise, stable vital signs. Which obstetric emergency must be anticipated considering a possible abdominal trauma? A. Abruptio placentae B. Ectopic pregnancy C. Placenta previa D. Massive uterine rupture Submit Answer

Explanation Choice A is correct. External trauma can lead to abruptio placentae, the complete or partial separation of the placenta from the uterine wall. A sign that concealed hemorrhage has occurred is the rapid increase in uterine size along with rigidity. Choices B, C, and D are incorrect. An ectopic pregnancy occurs when the embryo implants outside the uterine cavity. Placenta previa is a condition where the placenta is implanted in the lower segment of the uterus that either entirely or partially covers the cervical os. A massive uterine rupture happens during labor when the uterine contents are extruded through the uterine wall. Last Updated - 03, Feb 2022

Which of the following maternal deficiencies may result in neural tube defects in a fetus? A. Folic acid B. Vitamin B12 C. Vitamin E D. Iron

Explanation Choice A is correct. Folic acid is essential for the development of the neural tube and might prevent the defect or failure of the machine to close. Neural tube defects are one of the most common congenital disabilities, occurring in approximately one in 1,000 live births in the United States. A neural tube defect is an opening in the spinal cord or brain that occurs very early in human development. The first spinal cord of the embryo begins as a flat region, which rolls into a tube (the neural tube) 28 days after the baby is conceived. When the neural tube does not close completely, a neural tube defect develops. Neural tube defects develop before most women know they are even pregnant. Neural tube defects are congenital disabilities of the brain, spine, or spinal cord. They happen in the first month of pregnancy, often before a woman even knows that she is pregnant. The two most common neural tube defects are spina bifida and anencephaly. Neural tube defects are considered a complex disorder because they are caused by a combination of multiple genes and multiple environmental factors. Known environmental factors include folic acid deficiency, maternal insulin-dependent diabetes, and maternal use of certain anticonvulsant (antiseizure) medications. While only a few environmental factors have been characterized, many different studies provide evidence that NTDs have a genetic component in their development. Studies of twins with NTDs have shown both identical twins have NTDs more than both fraternal twins. Studies of families show that the chance of having a second family member born with an NTD after one child is born with an NTD is increased. For example, the general population's chance of having an NTD is approximately 0.1% (1 in 1000). However, once the couple has one child with an NTD, their chance of having a second child with an NTD is increased to approximately 2-5%. Further studies have shown evidence for a genetic pattern of inheritance for NTDs. NTDs are a feature (or symptom) of known genetic syndromes, such as trisomy 13, trisomy 18, specific chromosome rearrangements, and Meckel-Gruber syndrome. Choices B, C, and D are incorrect. None of these are associated with neural tube defects. NCSBN Client Need Topic: Physiological Integrity; Subtopic: Physiological Adaptation, Antepartum Last Updated - 20, Oct 2021

The nurse at a gynecology clinic speaks with a 25-year-old, 32-week pregnant client. The nurse assesses the client and the client's laboratory results. Which of the following findings should most concern the nurse? A. Glucose present in the urine B. The client reports +1 pedal edema at the end of the day C. The client reports increased vaginal discharge D. A hemoglobin level of 14 g/dL Submit Answer

Explanation Choice A is correct. Glucose in the urine may indicate gestational diabetes, as up to half of female clients have glucose in their urine at some time during pregnancy. Glucose in the urine may mean that a pregnant client has gestational diabetes. The nurse should promptly alert the health care provider (HCP) to allow further assessment into the cause of the glucose present in the client's urine. Choice B is incorrect. As pregnancy progresses, clients will often report the presence of slight pedal edema. This pedal edema occurs due to the pressure of the growing fetus on the vena cava, thus reducing venous return from the lower extremities. Choice C is incorrect. Increased vaginal discharge commonly occurs during pregnancy, especially as the client progresses toward the third trimester. As the client nears term, there is increased vascularity in the vagina and perineum. The nurse should inquire whether the discharge is foul-smelling, bloody, and/or abnormally colored, as these characteristics would be cause for concern and would need to be reported to the health care provider (HCP) for further evaluation. Choice D is incorrect. A hemoglobin level of 14 g/dL is an acceptable level for a pregnant client. Normal hemoglobin levels in pregnant women range from 11.5 to 14 g/dL. During pregnancy, the amount of blood in the client's body increases by about 20-30%, increasing the supply of iron and vitamins needed to make hemoglobin. Factors such as dehydration as well as overhydration can affect the test results. Learning Objective When reviewing the assessment and laboratory findings of a 25-year-old, 32-week pregnant client, identify glucose in the client's urine as the most concerning finding. Additional Info During routine prenatal care, clients undergo testing for gestational diabetes between 24 and 28 weeks of pregnancy. Approximately 6-10% of women in the United States develop gestational diabetes. Testing for gestational diabetes initially includes a glucose challenge test. If the client's blood glucose level exceeds the acceptable test range during the glucose challenge test, the client will subsequently undergo follow-up glucose tolerance testing to confirm or rule out a gestational diabetes diagnosis. Last Updated - 16, Dec 2022

A 35-week pregnant client arrives at an emergency department complaining of heavy, painful vaginal bleeding. The nurse should immediately suspect which condition? A. Abruptio placentae B. Placenta previa C. Prelabor rupture of the membranes D. Passage of the mucus plug Submit Answer

Explanation Choice A is correct. Here, the nurse should immediately suspect abruptio placentae, as this client presents with the classic signs of severe pain and vaginal bleeding. Abruptio placentae (called abruptio placentae, placental abruption, or premature separation of the placenta) is the premature separation of a normally implanted placenta from the uterus. Placental abruption presents a significant risk for both the woman and the fetus. Once the premature placental separation begins, the woman becomes at risk for hemorrhage and consequent hypovolemic shock and clotting abnormalities. Similarly, risks to the fetus include asphyxia, excessive blood loss, prematurity, and fetal demise. Choice B is incorrect. Placenta previa is the implantation of the placenta in the lower uterus. The classic sign of placenta previa is the sudden onset of painless uterine bleeding. In the above scenario, the client presents with "heavy, painful vaginal bleeding," which is not in congruence with a classic placenta previa client presentation. Choice C is incorrect. Prelabor rupture of membranes (PROM) is leakage of amniotic fluid before the onset of labor. Typically, unless complications occur, the only symptom of PROM is leakage or a sudden gush of amniotic fluid from the vagina. The above client's complaints do not align with PROM. Choice D is incorrect. Most women do not realize they have lost their mucus plug. If a woman does, a mucus plug will often appear similar to vaginal discharge in one's underwear and not contain blood. Learning Objective Correlate complaints of heavy, painful vaginal bleeding in a third-trimester pregnant woman with a tentative diagnosis of abruptio placentae. Additional Info Abruptio placenta is a true obstetric emergency. The severity of symptoms and signs depends on the degree of separation of the placenta from the uterus and the amount of blood loss. Abruptio placentae and other obstetric abnormalities increase the risk of morbidity or mortality for the woman, fetus, or neonate. Vaginal bleeding in placental abruption may be external or concealed (i.e., retained). Last Updated - 21, Aug 2022

The nurse is preparing to perform Leopold maneuvers on a client who is pregnant. The nurse understands that the purpose of this procedure is to A. determine the location and presentation of the fetus. B. prevent compression to the inferior vena cava. C. measure the involution progress of the uterus. D. assess whether the amniotic sac has ruptured. Submit Answer

Explanation Choice A is correct. Leopold maneuvers is a non-invasive technique that uses palpation through the abdominal wall to determine the presentation and location of the fetus. These maneuvers are typically done after 24 weeks but should be done in about 36 weeks for a more accurate assessment. Choices B, C, and D are incorrect. Maternal repositioning to the left (or right) side prevents compression to the inferior vena cava. Measuring the progress of involution (the uterus returning to the pre-pregnancy state) is not involved in Leopold maneuvers, as these are completed before delivery. The nitrazine test helps determine whether the amniotic sac has ruptured. Additional Info Leopold maneuvers is a series of tests that are done to determine fetal location and presentation. 1. Explain the procedure to the woman and the rationale for each step as it is performed. Tell her what is found at each step. Gives information, teaches the woman, and reassures her when the assessment findings are normal. 2. Ask the woman to empty her bladder if she has not done so recently. Have her lie on her back with her knees flexed slightly. Place a small pillow or folded towel under one hip. Decreases discomfort of a full bladder during palpation and improves ability to feel fetal parts in the suprapubic area. Knee flexion helps the woman relax her abdominal muscles to enhance palpation. Uterine displacement prevents aortocaval compression, which could reduce blood flow to the placenta. 3. Wash your hands with warm water. Wear gloves if contact with secretions is likely. Prevents transmission of microorganisms. Warm hands are more comfortable during palpation and prevent tensing of abdominal muscles. 4. Stand beside the woman, facing her head, with your dominant hand nearest her. The first three maneuvers are most easily performed in this position. 5. First maneuver—Palpate the uterine fundus. Use the flat palmar surface of hands, with fingers together using gentle, but firm pressure to perform palpation. The breech (buttocks) is softer and more irregular in shape than the head. Moving the breech also moves the fetal trunk. The head is harder and has a round, uniform shape. The head can be moved without moving the entire fetal trunk. Distinguishes between a cephalic and breech presentation. If the fetus is in a cephalic presentation, the breech is felt in the fundus. If the presentation is breech, the head is felt in the fundus. 6. Second maneuver—Hold the left hand steady on one side of the uterus while palpating the opposite side of the uterus with the right hand. With the right hand, start palpating at the uterine fundus and progress downward toward the symphysis pubis. Then hold the right hand steady while palpating the opposite side of the uterus with the left hand. The fetal back is a smooth, convex surface. The fetal arms and legs feel nodular, irregular, or protruding, and the fetus often moves them during palpation. Determines on which side of the uterus is the back and on which side are the fetal arms and legs ("small parts"). 7. Third maneuver (Pallach's maneuver or grip)—Palpate the suprapubic area. Attempt to grasp the presenting part gently between the thumb and middle finger. If the presenting part is not engaged, the grasping movement of the fingers moves it upward in the uterus. If the presenting part is engaged, it remains fixed and difficult to move. If a breech was palpated in the fundus, expect a hard, rounded head in this area. Confirms the presentation determined in the first maneuver. Determines whether the presenting part is engaged (widest diameter at or below zero station) in the maternal pelvis. 8. Omit the fourth maneuver if the fetus is in a breech presentation. Is performed only in cephalic presentations to determine whether the fetal head is flexed. Murray, S., McKinney, E. (032018). Foundations of Maternal-Newborn and Women's Health Nursing, 7th Edition. Last Updated - 11, Nov 2022

The nurse is caring for a client with placenta previa. Which of the following orders by the primary healthcare provider (PHCP) should the nurse question? A. Manual cervical exam B. Transabdominal ultrasound C. Nonstress test D. Transvaginal ultrasound Submit Answer

Explanation Choice A is correct. Manual cervical exams should be questioned if a client has placenta previa. The reasoning is that palpation of the placenta previa through a partially dilated cervix may cause severe hemorrhage. Choices B, C, and D are incorrect. These orders are appropriate for a client with placenta previa. Transvaginal and transabdominal ultrasounds are safe. Transvaginal ultrasound is the most accurate way to assess placenta previa. A nonstress test is often completed to determine fetal well-being. This is a non-invasive test usually conducted during the third trimester. Additional Info

An 11-week pregnant client is complaining to the nurse about her hemorrhoids. The nurse understands that hemorrhoids occur because of pressure on the rectal veins from the bulk of the growing fetus. All of the following are measures to alleviate hemorrhoid pain, except: A. Instruct the client to use mineral oil to soften her stools. B. Rest in a side-lying position daily. C. Increase the client's fiber and water intake. D. Apply a cold compress to the area.

Explanation Choice A is correct. Mineral oil is contraindicated in pregnancy as it decreases nutrient absorption in the mother. Choice B is incorrect. Sleeping in a side-lying position removes the weight of the fetus on the superior and inferior vena cava, promoting venous return and decreasing venous pressure. Choice C is incorrect. Increasing fiber and water intake promote the formation of bulkier stools. Preventing constipation and relieving rectal pain. Choice D is incorrect. Cold compresses relieve pain by vasoconstriction of the hemorrhoids. Last Updated - 23, Jan 2022

You are working in the intensive care nursery and are assigned to take care of an infant withdrawing from cocaine. At your first assessment, you appreciate the following: a high-pitched cry, no tremors, increased muscle tone, sleeping for 3 hours in between feedings, no congestion, respiratory rate = 42, excessive sucking on the pacifier, poor nutrition, no vomiting, and no loose stools. What is the neonatal abstinence score (NAS) for this patient? A. 7 B. 12 C. 2 D. 8

Explanation Choice A is correct. Neonatal abstinence syndrome (NAS) occurs due to sudden withdrawal of the fetus due to the discontinuation of substances used/abused by the mother during pregnancy. The Finnegan scoring system is commonly used to assess the severity of NAS. The NAS score uses 21 symptoms that are mostly seen in opiate-exposed infants. A numerical score is assigned to each sign and its corresponding severity. The total abstinence score is determined by the sum of the numerical score attached to each sign. This scoring can help guide initiation, monitoring, and cessation of treatment in the newborn. The initial treatment of NAS includes nonpharmacological methods. If improvement is not noted with nonpharmacological measures or if the infant develops severe withdrawal symptoms, pharmacological agents are used. NAS score helps in assessing the severity. The most common agent used in the treatment of NAS secondary to opioids is morphine. The infant may breastfeed unless the mother is involved in polysubstance abuse or has HIV. Each of these observations gives you a part of the NAS score for this infant: High pitched cry = 2 points No tremors = 0 points Increased muscle tone = 2 points Sleeping for 3 hours in between feedings = 0 points No congestion = 0 points RR = 42 = 0 points Excessive sucking on the pacifier = 1 point Poor feeding = 2 points No vomiting = 0 points No loose stools = 0 points This adds up to a total of 7 points for the NAS score for this patient. A score of 7 is in the moderate range. Less than five is mild, 5-8 is average, 8-12 is severe, and greater than 12 is very critical. For a score of 7, a breakthrough dose of morphine may not be appropriate, but the infant may not be ready to wean down on their methadone further. NCSBN Client Need: Topic: Health Promotion and Maintenance, Subtopic: Newborn Last Updated - 15, Feb 2022

At 25 weeks gestation, a pregnant client presents with a uterine growth size that is less than expected, decreased fetal movement, and an easily palpable fetus. Which of the following is this likely related to? A. Oligohydramnios B. Macrosomia C. Hydramnios D. Amniotic fluid embolism Submit Answer

Explanation Choice A is correct. Oligohydramnios results from a severe reduction in the amount of amniotic fluid. It results in less than expected fetal growth. Also, because of the low amount of amniotic fluid, the fetus will be more easily outlined and palpated. Choice B is incorrect. Macrosomia is defined as a newborn that is significantly larger than average. These babies have a birth weight of more than 8 lbs, 13 oz. Choice C is incorrect. Hydramnios is a condition in which excessive amounts of amniotic fluid accumulates during pregnancy. Choice D is incorrect. Amniotic fluid embolism is characterized by an acute collapse of mother and baby due to an allergic-type response to amniotic fluid entering the mother's circulatory system. NCSBN Client Need Topic: Health Promotion and Maintenance, Subtopic: Care of the Woman and Fetus at Risk Last Updated - 11, Feb 2022

A client at 32 gestational weeks reports the sudden onset of painless, bright red vaginal bleeding. The assessment showed a normal fetal heart rate and a non-tender uterus. The nurse understands that this client is at the highest risk of developing A. placenta previa. B. threatened abortion. C. placental abruption. D. uterine souffle. Submit Answer

Explanation Choice A is correct. Placenta previa may occur as early as 20 gestational weeks. The manifestations of painless, bright red vaginal bleeding coincide with this condition. Commonly, the presentation of placenta previa is a finding on routine ultrasound examination at approximately 16 to 20 weeks. Choices B, C, and D are incorrect. A threatened abortion may only occur before 20 gestational weeks. Thus, this condition is excluded. Placental abruption is highly serious and manifests with painful vaginal bleeding that causes the uterus to be firm and tender. Uterine souffle is a soft, blowing sound. This sound may be auscultated over the uterus. This is the sound of blood circulating through the dilated uterine vessels, and it corresponds to the maternal pulse. Additional Info Placenta previa is an implantation of the placenta in the lower uterus. As a result, the placenta is closer to the internal cervical os than to the presenting part (usually the head) of the fetus. The classic sign of placenta previa is the sudden onset of painless uterine bleeding in the last half of pregnancy. Many cases of placenta previa are diagnosed by ultrasound examination before any bleeding occurs. Last Updated - 08, Jul 2022

A 38-week pregnant woman comes into the emergency department complaining of vaginal bleeding. The client is not in obvious distress or pain. Which statement by the client would lead the nurse to suspect placenta previa? A. "I don't feel any pain at all. It's just the bleeding that concerns me." B. "I feel like I'm about to go into labor. My tummy is starting to contract." C. "I started bleeding when I picked up my 3-year-old son, who weighs 32 pounds." D. "I feel like I'm about to vomit." Submit Answer

Explanation Choice A is correct. Placenta previa typically manifests as painless vaginal bleeding after 20 weeks gestation. Choice B is incorrect. The client's statement's regarding contractions (including the implied pain associated with contractions) do not correlate with the symptoms of placenta previa. This client's comments would likely lead the nurse to assume this client is in early labor. Choice C is incorrect. With placenta previa clients, bleeding is not necessarily correlated with lifting heavy objects. Choice D is incorrect. Nausea is not typically associated with placenta previa. Typically, nausea is more common in early pregnancy than in late pregnancy. Learning Objective Associate a client's painless vaginal bleeding in the 38th week of pregnancy with a suspected diagnosis of placenta previa. Additional Info Placenta previa occurs when the placenta implants over or near the internal os of the cervix. In clients with placenta previa, the source of bleeding is maternal. Diagnosis is performed by ultrasonography. Treatment is modified activity if minor vaginal bleeding occurs before 36 weeks gestation, with cesarean delivery at 36 to 37 weeks, 6 days. Immediate cesarean delivery is indicated if the bleeding is severe or refractory or if the fetal status is nonreassuring.Source : Archer Review Last Updated - 30, Nov 2022

A prenatal client with gingivitis at her fourth-month clinic visit mentions that she has a tooth extraction planned for the following month and is wondering whether or not she can continue with the procedure. What information will you provide the prenatal client? A. The second trimester is the safest period for dental extractions. B. She will need to wait until after delivery to have the procedure performed. C. She should wait until the third trimester to have the procedure performed. D. She should take anti-viral medications before the procedure to prevent illness. Submit Answer

Explanation Choice A is correct. Pregnancy increases a woman's risk of developing gingivitis and cavities. The patient has gingivitis and it appears like her dentist recommended tooth extraction for her. There is some concern regarding undergoing dental extractions during pregnancy. However, there is no evidence that a pregnant woman will need to delay dental removal. Delaying dental care could be harmful to the mother and fetus. Emergency treatment can be done at any time during pregnancy. However, elective dental surgery should be deferred until the second trimester (weeks 14 through 20). By the second trimester, fetal organogenesis is complete, and the risk of adverse effects from procedure/medications is lower. The consequences of not treating a dental infection during pregnancy outweigh the possible risks of the medications. The patient has gingivitis that needs to be addressed soon. Also, the blood volume of the pregnant woman significantly increases in 3rd trimester and remains elevated until delivery. Although there is no risk to the fetus during this trimester, the pregnant woman may experience increased discomfort. There is also a risk of hypotension in the supine position, so, short dental appointments are advocated during the 3rd trimester. It is, therefore, better not to perform elective dental extractions during 3rd trimester. Choices B and C are incorrect. The prenatal client should not delay dental care. Choice D is incorrect. Anti-viral medications should not be given to a woman who will be undergoing a dental procedure. NCSBN client need Topic: Maintenance and Health Promotion, Ante / Intra /Postpartum Care Last Updated - 18, Nov 202

The nurse takes a call at the antepartum clinic from a client who is pregnant at 19 weeks gestation. The client reports feeling a flutter in her abdomen. The nurse understands that this client is likely experiencing A. quickening. B. lightening. C. ballottement. D. ambivalence. Submit Answer

Explanation Choice A is correct. Quickening is often described as a flutter sensation in the client's abdomen. This occurs during 16 to 20 weeks of gestation. The client is likely experiencing this sensation as she is at 19 weeks of gestation. Choices B, C, and D are incorrect. Lightening is the descent of the fetus toward the pelvic inlet before labor, and it occurs about 2 to 3 weeks before the natural onset of labor. Ballottement is an examination technique where a sudden tap on the cervix during the vaginal examination may cause the fetus to rise in the amniotic fluid and then rebound to its original position. Ambivalence is a natural occurring feeling by expected mothers during the first trimester when she feels conflicted about the pregnancy. Additional Info Quickening is usually first noticed by the expectant mother at 16 to 20 weeks of gestation and gradually increases in frequency and strength. Women often describe this initial sensation as a fluttering. Last Updated - 13, May 2022

A woman is in the labor and delivery suite at 37 weeks gestation. She has been under her obstetrician's care for preeclampsia. The labor nurse notices that the fetus is experiencing heart rate decelerations. You are part of the neonatal resuscitation team that responds to the call from the labor room nurse. The infant is born but does not respond to tactile stimulation. The group moves the infant to the warmer. You evaluate the infant and confirm he is still not breathing. You begin positive pressure blowing with room air. Another team member notes that the heart rate is 72 bpm and the newborn's chest is not moving with PPV on room air. The next appropriate action is to: A. Reposition the infant to open the airway B. Begin CPR C. Suction the infant with a bulb syringe D. Increase the oxygen concentration Submit Answer

Explanation Choice A is correct. Reposition the infant to open the airway while ensuring that you have a good seal with the mask on the newborn's face. Following that action, a team member should suction the infant's mouth and nose. Until the team establishes sufficient ventilation, there is no indication to increase oxygen concentration or begin CPR. The AHA and AAP focus on positive-pressure ventilation as the single most crucial step in the resuscitation of the newborn. Choices B, C, and D are incorrect. NCSBN Client Need Topic: Physiological Adaptation, Sub-topic: Alterations in Body Systems, Newborn Last Updated - 19, Jan 2022

A pregnant client who is Rh-negative is ordered an indirect Coomb's test. The nurse understands that the purpose of this test is to determine if A. antibodies are present from previous exposure to Rh-positive blood. B. the amount of time that it takes for fetal blood to clot. C. blood type, Rh factor, and antibody titer of the newborn D. the fetus has a risk of developing pernicious anemia later in life. Submit Answer

Explanation Choice A is correct. Rh-negative women should have an indirect Coombs' test to determine whether they are sensitized (have developed antibodies) as a result of previous exposure to Rh-positive blood. This testing is done at the first prenatal visit. Choices B, C, and D are incorrect. Coombs' testing (direct or indirect) does not measure clotting time (either maternal or fetus). The blood type, Rh factor, and antibody titer of the newborn are determined by the direct Coomb's test. These tests do not determine the probability of pernicious anemia, as this anemia is predicated on the lack of vitamin B12. Additional Info Rh-negative women should have an indirect Coombs' test to determine whether they are sensitized (have developed antibodies) due to previous exposure to Rh-positive blood. If the indirect Coombs' test is negative, it is repeated at 28 weeks of gestation to identify whether they have developed subsequent sensitization. In direct Coombs' testing, if the mother is Rh-negative, the umbilical cord blood is taken at delivery to determine the blood type, Rh factor, and antibody titer of the newborn. Last Updated - 09, Jul 20

The nurse is taking the history and physical of a woman who has just discovered that she is pregnant. This nurse knows that the purpose of asking a prenatal client about her history with rheumatic fever has the most to do with: A. Cardiac stress related to a possible valvular lesion. B. Preventing transmission of this teratogenic condition to her infant. C. Preparing to deliver preventative antibiotics during labor and post-birth. D. Monitoring lung sounds for reoccurrence of the disorder. Submit Answer

Explanation Choice A is correct. Rheumatic fever can cause the formation of valvular lesions, which can lead to cardiac stress during pregnancy. Choice B is incorrect. A prenatal client with a history of rheumatic fever will not be at risk for passing on rheumatic fever to her infant. Choice C is incorrect. Preventative antibiotics would not be needed in this circumstance. Choice D is incorrect. Lung sounds are not relevant to a woman with a history of rheumatic fever. NCSBN Client need Topic: Health Promotion and Maintenance / Ante / Intra and Postpartum Care Last Updated - 10, Feb 2022

A G1P0 client with a blood type A negative is at her 28th-week gestation and was advised a RhoGAM injection today. Which statement by the client indicates the need for further teaching about this therapy? A. "This shot is meant to prevent my baby from developing antibodies against my blood, right?" B. "I understand that if we find out my baby is Rh positive, then I'll need to get another one of these injections." C. "This shot should help to protect me in future pregnancies if this baby comes out Rh positive, like her dad." D. "This shot will prevent me from becoming sensitized to Rh-positive blood." Submit Answer

Explanation Choice A is correct. RhoGAM is administered to Rh-negative mothers to prevent them from producing antibodies against their Rh-positive fetus. "This shot is meant to prevent my baby from developing antibodies against my blood, right?" indicates that the client needs further teaching. Choices B, C, and D are incorrect. If the infant is Rh-positive, the mother needs to receive another dose after delivery to prevent maternal sensitization. This will also protect future pregnancies as the mother's blood will be free of antibodies against her fetus. RhoGAM prevents maternal sensitization of Rh-positive blood. Last Updated - 13, Jan 2022

At her first visit, a prenatal client is found to be suffering from mildly high blood pressure. The nurse should have her client reduce which dietary component? A. Salt B. Magnesium C. Potassium D. Calcium Submit Answer

Explanation Choice A is correct. Salt should be restricted in the client with mildly high blood pressure. A blood pressure that is considered moderately high is about 140/90 mmHg. These patients should begin treatment by reducing salt intake and assessing behavioral areas that may need adjustment, such as smoking cigarettes or failing to exercise. Choices B, C, and D are incorrect. Magnesium, potassium, and calcium do not need to be reduced when a patient presents with high blood pressure. NCSBN client need Topic: Maintenance and Health Promotion, Ante / Intra / Postpartum Care Last Updated - 21, Jan 2022

Which of the following best describes the reflex of a newborn, which includes: hand opening with abducted and extended extremities following a jarring motion? A. Moro reflex B. Grasp reflex C. Babinski reflex D. Rooting reflex Submit Answer

Explanation Choice A is correct. The Moro reflex occurs in response to a slight drop, sudden movement of the crib, or a loud noise, the newborn quickly makes a symmetrical abduction of the extremities and places the index fingers and thumbs into a "C" shape. The newborn's neurological system is immature at birth. The nurse may notice periodic jerking or twitching, which is considered normal. Tremors are not considered a normal finding in a newborn. The newborn's cry can provide information about the neurological status. A high-pitched scream can indicate an increase in intracranial pressure. When assessing the reflexes, the nurse needs to consider the gestational age, not the birth weight. Premature infants will have a reduced response to the reflex evaluation. The nurse should document and report the following warning signs: Tremors A high-pitched cry Abnormal pupil responses Hypertonic or hypotonic positions Absent newborn reflexes Choice B is incorrect. The grasp reflex occurs when the newborn wraps the fingers around the examiner's finger when it is placed in the newborn's palm. Choice C is incorrect. When the sole of the foot is stroked, the newborn's big toe moves upward toward the top surface of the foot and the other toes fan out. This is known as the Babinski reflex. Choice D is incorrect. The rooting reflex occurs as the newborn turns his/her head to the side on which the cheek is stroked. NCSBN Client Need Topic: Psychosocial Integrity, Subtopic: Newborn Reflexes Last Updated - 12, Jul 2021

Following a pregnant client's report of persistent nighttime leg cramps over the past week, it would be appropriate for the nurse to instruct the client to increase their dietary intake of: A. Whole grains, nuts, egg yolks B. Almonds, sweet potato, avocado C. Lentils, peas, nuts D. Carrots, tomatoes, squash Submit Answer

Explanation Choice A is correct. The client's complaint of nocturnal leg cramps may indicate inadequate magnesium and/or vitamin D intake. Whole grains and nuts are magnesium-rich foods, while egg yolks are a significant source of Vitamin D. The nurse should educate the client on these nutritional sources and how the consumption of these items will assist in alleviating the client's nocturnal leg cramping. Choice B is incorrect. Almonds, sweet potatoes, and avocadoes are foods rich in vitamin E. While the consumption of these foods will benefit the client, consuming foods will not resolve the client's primary complaint of nocturnal leg cramps. Choice C is incorrect. Lentils, peas, and nuts are folic acid-rich foods. Folic acid intake is vital in the early stages of pregnancy, as deficiency of this nutrient may cause spontaneous abortion and/or neural tube defects. However, folic acid does not aid in relieving nocturnal leg cramps during pregnancy. Choice D is incorrect. Squash, carrots, and tomatoes are high in beta-carotene. Although beta-carotene does not prevent nocturnal leg muscle cramps, this nutrient aids in fetal growth and cell differentiation. Learning Objective Correlate a pregnant client's nocturnal leg cramps with the client's need to increase the dietary intake of magnesium and vitamin D. Additional Info Legumes and seeds are also rich in magnesium. Vitamin D-rich foods include butter, soy products, and vitamin D fortified milk or orange juice. Painful contractions of the muscles of the legs occur in 25% to 50% of pregnant women. Typically, these cramps result from a lack of magnesium and/or vitamin D in the diet. Cramps often occur during sleep when the muscles are relaxed or when the woman stretches and extends her feet with the toes pointed. Last Updated - 30, Sep 2022

The nurse is assessing an infant with dark skin for jaundice. The nurse plans on assessing this client's A. hard palate of the mouth. B. lower back and sacrum. C. lower legs right below the knee. D. nail beds. Submit Answer

Explanation Choice A is correct. The correct technique when assessing an infant (or an adult) with dark skin for jaundice would be to examine the mucous membranes in the mouth, the hard palate, or the sclera. Choices B, C, and D are incorrect. These are not appropriate physical landmarks to assess for jaundice. Additional Info In infants with dark skin, assess the color of the palate and mucous membranes of the mouth and the conjunctivae. Determine the areas of the body affected by jaundice, and document carefully to use for comparison during future assessments. Jaundice begins at the head and moves down the body as the bilirubin level rises. Last Updated - 10, Jul 2022

You are on the team preparing to give positive-pressure ventilation to a newborn. You have selected the correct size mask and suctioned the infant's mouth and nose. You know that you should start positive-pressure ventilation with: A. 21% oxygen (room air) B. 28% oxygen C. 50% oxygen D. 100% oxygen Submit Answer

Explanation Choice A is correct. The current recommendation from the American Heart Association and the American Academy of Pediatrics is to start PPV with 21% oxygen. Research shows that beginning resuscitation with room air is as adequate as beginning with 100% oxygen and avoids the possible ill effects of using high levels of oxygen. The team should titrate the oxygen to achieve a specific level of oxygen saturation at particular times after birth. Therefore, the team must implement pulse oximetry early in the newborn resuscitation process. Choices B, C, and D are incorrect. NCSBN Client Need Topic: Physiological Adaptation, Sub-topic: Alterations in Body Systems, Newborn Last Updated - 18, Aug 2021

The nurse is caring for a newborn of a heroin-addicted mother. Which nursing intervention should the nurse implement in caring for this newborn? A. Decrease the newborn's sensory stimulation. B. Perform activities in one setting. C. Loosely wrap the neonate in a blanket. D. Place the newborn in a stimulating environment. Submit Answer

Explanation Choice A is correct. The drug-dependent newborn is irritable and very sensitive to environmental stimuli. He/she should have limited sensory input to allow extensive rest periods. Choice B is incorrect. The newborn should have procedures to him/her as tolerated. He/she should always have extended rest periods. Choice C is incorrect. The neonate should be wrapped tightly in a flexed position to promote rest. Choice D is incorrect. Increasing environmental stimuli can exacerbate the newborn's irritability and restlessness. Last Updated - 15, Feb 2022

You are planning an educational series of classes for young pregnant women. Which of the following needs should you include in a class related to nutrition during pregnancy? A. The need to increase caloric intake by about 340 calories during the second trimester of gestation. B. The need to increase caloric intake by about 370 calories during the second trimester of gestation. C. The need to increase caloric intake by about 340 calories during the third trimester of gestation. D. The need to increase caloric intake by about 370 calories during the third trimester of gestation. Submit Answer

Explanation Choice A is correct. The need that you should include in a class related to nutrition during pregnancy for a group of young pregnant women is the need to increase caloric intake by about 340 calories during the second trimester of gestation. This caloric increase is necessary to support the growth and development of the fetus. Similarly, a caloric increase of about 450 calories is indicated during the third trimester of gestation for the same reason. Choice B is incorrect. Although there is a need to increase caloric intake during the second trimester of gestation, this increase is less than 370 calories. Choice C is incorrect. Although there is a need to increase caloric intake during the third trimester of gestation, this increase is more than 340 calories. Choice D is incorrect. Although there is a need to increase caloric intake during the third trimester of gestation, this increase is more than 370 calories. Last Updated - 05, Dec 2021

The nurse is supervising a nursing student to teach a pregnant client about a scheduled chorionic villus sampling (CVS) test. Which statement, if made by the nursing student, would require follow-up? A. You will need to provide both a urine and blood sample for this test. B. Drink plenty of water prior to this test and do not empty your bladder. C. An ultrasound will be used during this procedure to guide the needle. D. It is okay to eat and drink on the day of the procedure. Submit Answer

Explanation Choice A is correct. This statement is untrue and requires follow-up. A CVS is a test utilized to determine the presence of chromosomal abnormalities and involves the aspiration of small samples of the placenta for prenatal genetic diagnosis. Maternal blood and urine specimens are not necessary for this test. Choices B, C, and D are incorrect. These statements are factual and do not require follow-up. The CVS uses ultrasound, and a full bladder allows for an acoustic window to ensure accurate imaging. No eating or drinking restrictions are in place during preprocedure. The client may eat and drink normally. Additional Info Chorionic villus sampling is a test that may be performed as early as ten gestational weeks to determine if the fetus has any chromosomal abnormalities. Chorionic villus sampling has drawbacks that preclude its use, including possible spontaneous abortion of the fetus and fetomaternal hemorrhage. Last Updated - 16, Dec 2022

The nurse is caring for assigned clients. The nurse should recognize that the patient at most significant risk for postpartum hemorrhage (PPH) is the client who has which of the following? A. Uterine atony and delivered with the assistance of forceps B. Postpartum urinary incontinence and diuresis C. An active outbreak of genital herpes and had a cesarean section D. Gestational diabetes and has postpartum hyperglycemia Submit Answer

Explanation Choice A is correct. Uterine atony is the most common cause of PPH. This is when the uterus fails to contract after delivery. Additionally, delivery with instruments such as forceps raises the risk of PPH because of the trauma the devices may cause. Choices B, C, and D are incorrect. Post-partum urinary incontinence is a common occurrence along with maternal diuresis. This occurs because fluids shift from the extravascular space to the intravascular space. Both findings are normal and do not place the patient at risk for PPH. An outbreak of herpes is not a risk factor for PPH. However, a cesarean section most certainly would raise the risk for PPH. Gestational diabetes may increase the risk of a macrosomia fetus, which would cause PPH; however, this is not indicated in the choice. Finally, post-partum hyperglycemia may occur but is not a risk factor for PPH. Insulin sensitivity may take time for it to normalize. Additional Info PPH is an obstetric emergency. Risk factors for PPH include - Uterine atony Prolonged labor Instrument assisted delivery Preeclampsia and eclampsia Fetal macrosomia Multifetal gestation Last Updated - 06, Dec 2022

A client who is a native of the Middle East is now on her 24th-week gestation. As part of her culture, she usually wears a long robe that covers her arms and body, with a shawl that covers her head and neck. Which supplement will the nurse most likely expect to give her? A. Vitamin D B. Vitamin C C. Calcium D. Zinc Submit Answer

Explanation Choice A is correct. Women from the Middle East are usually covered from head to foot. This causes them to receive little sun exposure. Unless the client's diet is rich in good sources of vitamin D, she needs to supplement it. Choices B, C, and D are incorrect. The situation has no data indicating the need for vitamin C, Calcium, or Zinc supplementation. Last Updated - 15, Nov 2021

While the nurse uses Nagele's rule to determine the prenatal clients' estimated due date (EDD), they know that their calculations are limited by the fact that they assume: A. Ovulation occurs on day 14 B. Pregnancy lasts 9 months C. Amenorrhea is the first sign of pregnancy D. Cycles are 30 days in length Submit Answer

Explanation Choice A is correct: Naegele's rule is limited in calculating the EDD as it assumes that all women ovulate around day 14 of their menstrual cycle. Women all vary biologically and may ovulate on varying days within their cycle. Naegele's rule also incorrectly assumes that all women have cycles that last 28 days. Choice B is incorrect: Pregnancy lasts 40 weeks. The term "month" assumes four weeks. 9 months would roughly equal 36 weeks. This is not how pregnancy is calculated. The average pregnancy lasts for 282 days which equals 40 weeks. Choice C is incorrect: Naegele's rule in factoring the estimated due date does not have to do with what the first sign of pregnancy was in the woman. Amenorrhea is not always the first symptom women experience when they become pregnant. Each individual client experiences different pregnancy symptoms. What is essential to know when calculating Naegele's rule is to understand the first day of the last menstrual period. Choice D is incorrect: Naegele's rule is based on a cycle that lasts 28 days, not 30 days. Learning Objective Understand, educate on, and provide antepartum, intrapartum, postpartum, and newborn care. Additional Info Source: Archer LibrarySource : Source: Archer Library Gestation refers to the amount of time from fertilization of an egg (ovum) until the estimated delivery date. This term holds true for all mammals. The typical gestation age for a human is 280-282 days. Naegele's rule calculates the estimated delivery date (EDD). The use of Naegele's rule requires that a woman's menstrual cycle only lasts 28 days. Naegele's rule works by subtracting three months from the start of the patient's last menstrual cycle and then adding seven calendar days. A year is then added, and the user is left with the estimated date of delivery. Here is an example: First day of last menstrual cycle: February 5th, 2022 Subtract three months: November 5th, 2021 Add seven days: November 12th, 2021 Add one year: November 12th, 2022.

The nurse is caring for a client with an ectopic pregnancy. Which findings does the nurse expect? Select all that apply. A. Pelvic pain B. Fever C. Vaginal bleeding D. Positive pregnancy test E. Dysuria Submit Answer

Explanation Choice A, C, and D are correct. An ectopic pregnancy is a gynecological emergency. If not treated, a rupture may occur that leads to intraperitoneal bleeding. Classic manifestations of an ectopic pregnancy include: Presumptive pregnancy signs (nausea, breast tenderness) Vaginal bleeding Increased human chorionic gonadotropin (hCG) levels causing a positive pregnancy test Unilateral abdominal pain that may be confined to the pelvic area Choices B and E are incorrect. An ectopic pregnancy (EP) is not an infectious process like appendicitis. So fever is not an expected finding. Finally, dysuria would be a manifestation associated with cystitis. Again, an EP is not infectious and would not cause dysuria. Additional Info An ectopic pregnancy (EP) is an extrauterine pregnancy. Almost all ectopic pregnancies occur in the fallopian tube, but other possible sites include cervical, interstitial, hysterotomy (cesarean) scar, ovarian, or abdominal. Manifestations of an EP include unilateral abdominal (pelvic) pain, vaginal bleeding, and a positive pregnancy test. Rapid management is needed because life-threatening intraabdominal bleeding may occur. For EP's that have not ruptured and the woman is stable, methotrexate may be used. If that is not the case, surgical management will be necessary. Last Updated - 14, Nov 2022

When caring for a client new to the general practice clinic, the nurse notes that the woman is "nulliparous." The nurse knows that the term "nullipara" describes: A. A woman who had one pregnancy loss at 25 weeks and no children are alive. B. A woman who has never given birth to a child. C. A woman who had three prior pregnancies. D. A woman who has never been pregnant. Submit Answer

Explanation Choice B is correct. "Nullipara" refers to a woman who has never given birth (live birth or stillbirth). A nulliparous woman may or may not have been pregnant before. It also includes women who have been previously pregnant but have not given birth because of spontaneous miscarriage or elective abortion before 20 weeks of gestation. A woman who experiences pregnancy loss beyond 20 weeks is not included under "nullipara." Choice A is incorrect. A woman who carries a viable pregnancy beyond 20 weeks is not considered nullipara even if she had a stillbirth or pregnancy loss. Choice C is incorrect. A woman who had multiple pregnancies is considered to be multigravida. Choice D is incorrect. A woman who has never been pregnant before is referred to as "nulligravida", not nullipara. NCSBN Client need Topic: Maintenance and Health Promotion, Ante / Intra /Postpartum Care Last Updated - 15, Feb 2022

You are caring for a pregnant woman with a baseline BMI of 22. You educate this client on the desirable weight gain during pregnancy with one baby for her is: A. 28 to 40 pounds B. 25 to 35 pounds C. 15 to 25 pounds D. 11 to 20 pounds

Explanation Choice B is correct. 25 to 35 pounds. The amount of optimal weight gain during pregnancy is determined based on the woman's body mass index (BMI) before pregnancy. BMI is a measure of body fat calculated from weight and height. Please use the following table to determine the recommended weight gain during pregnancy. A baseline BMI of 22 indicates that this woman's baseline is in the healthy range (Normal BMI = 18.5 to 24.9). The recommended weight gain for this client is 25 to 35 pounds. Weight gain during pregnancy is crucial to the health and well-being of the baby and the mother. Gaining too little weight can lead to premature birth and low infant birth weight. Gaining too much weight can also result in premature birth and obesity of the child in later life. Excessive weight gain can result in strenuous labor, the increased possibility of needing a caesarian section, and increased bleeding. Choice A is incorrect. This is the recommended weight gain during singleton pregnancy for an underweight woman. Choice C is incorrect. This is the recommended weight gain during singleton pregnancy for an overweight woman. Choice D is incorrect. This is the recommended weight gain during singleton pregnancy for an obese woman. NCSBN Client Need Topic: Reduction of Risk Potential; Sub-topic: System-Specific Assessment Last Updated - 13, Jun 2021

The nurse is teaching a client about a scheduled contraction stress test (CST). Which of the following statements should the nurse include? A. "You will need to consume a liquid with 50 grams of glucose." B. "You may need to stimulate your nipples during this test." C. "A positive result means your baby has had no late decelerations." D. "A negative result means your baby has had variable decelerations."

Explanation Choice B is correct. A CST is indicated for high-risk clients who are in the third trimester. CST requires the client to have contractions either through oxytocin administration or nipple stimulation. Choices A, C, and D are incorrect. The client consuming a liquid with concentrated glucose is not indicated for a CST. This is appropriate for a glucose tolerance test. The results for a CST are interpreted as follows - Positive (abnormal) indicates that late decelerations were present in the FHR in more than 50% of the contractions. Negative (normal) indicates that no late or variable decelerations were evident during the contractions. Learning Objective Understand that contractions during a contraction stress test can be induced by intravenous administration of oxytocin or by nipple stimulation. Additional Info Source : Archer Review A contraction stress test is indicated for clients with high-risk pregnancies. This test is completed in the third trimester. Relative contraindications to a CST include placenta previa and patients at risk for preterm delivery as this test may hasten delivery or at minimal cause rupture of the membranes. Last Updated - 15, Feb 2022

A nurse is caring for a client in the first trimester of pregnancy and notes that the client's serum potassium level is currently 2.9 mEq/L. Which of the following assessment findings is likely related to this lab finding? A. Alcohol consumption during pregnancy B. Hyperemesis gravidarum C. Lack of weight gain since the onset of pregnancy D. Food aversions Submit Answer

Explanation Choice B is correct. A serum potassium level of 2.9 mEq/L indicates hypokalemia. During pregnancy, hyperemesis gravidarum is strongly associated with hypokalemia. Choice A is incorrect. Alcohol exposure in utero increases the risk of spontaneous abortion, decreases birth weight, and can cause fetal alcohol syndrome, a constellation of variable physical and cognitive abnormalities. Choice C is incorrect. Various factors may lead to a woman's lack of weight gain during the first trimester, but this lack of weight gain is unlikely to lead to such significant hypokalemia. Choice D is incorrect. Food aversions may cause the client to feel weak and/or nauseous during the first trimester but is unlikely to lead to hypokalemia. Learning Objective Correlate a finding of hypokalemia with the clinical finding of hyperemesis gravidarum in a client currently in the first trimester of pregnancy. Additional Info Typically, potassium levels should remain consistent throughout pregnancy. Hypokalemia occurs in around 1% of pregnancies. Approximately 20% of pregnancy-related hospitalizations in clients with hypokalemia also included a diagnosis of hyperemesis gravidarum. In those without hypokalemia, the diagnosis of hyperemesis gravidarum occurred in less than 1% of clients. Source : Archer Review Last Updated - 20, Sep 2022

The nurse is caring for a client with suspected placenta previa. The nurse anticipates a prescription for which diagnostic test to confirm this finding? A. Manual cervical exam B. Transvaginal ultrasound C. Contraction stress test D. Nonstress test Submit Answer

Explanation Choice B is correct. A transvaginal ultrasound is a gold standard in diagnosing placenta previa. A transvaginal ultrasound is more accurate than transabdominal ultrasound in determining the location of the placenta. Please recognize that the vagal ultrasound probe can safely be inserted a few inches from the cervix to capture the placenta alteration. Any advancement of the probe into the cervix could cause bleeding. However, if done according to the recommendations, the appropriate angle between the cervix and the vaginal ultrasound probe prevents the probe from accidentally moving into the cervical canal. Choices A, C, and D are incorrect. A manual cervical examination is contraindicated in placenta previa because this may trigger bleeding. A manual cervical exam attempts to invade the cervical canal directly, whereas a transvaginal ultrasound probe is inserted away from the cervix. While placenta previa is not an absolute contraindication for a contraction stress test, it raises the risk of bleeding. A nonstress test is safe and often performed for individuals with placenta previa. However, this test does not confirm placenta previa; instead, it evaluates fetal wellbeing. Learning Objective Recognize that a transvaginal ultrasound is an accurate and safe test in diagnosing placenta previa. Additional Info Last Updated - 30, Nov 2022

A nurse is caring for a post-cesarean section client in the maternity ward. Which finding by the nurse would be most indicative of a common post-cesarean delivery complication? A. A distended bladder B. Soaked perineal pads with a soft fundus C. Shivering D. An elevated temperature Submit Answer

Explanation Choice B is correct. Although some vaginal bleeding is anticipated following delivery, soaked perineal pads and a soft fundus are cause for concern. Typically, if a client is utilizing a new perineal pad every 15 minutes, persistent, significant bleeding should become the primary concern. Following a quick assessment for restlessness, increased pulse, decreased blood pressure, skin coolness, clamminess, and color, the nurse should immediately notify the physician. Choice A is incorrect. Although a full bladder interferes with the ability of the uterus to contract and, if not corrected, eventually leads to uterine atony, a distended bladder is not the finding most indicative of a common complication post-cesarean delivery. Choice C is incorrect. Following birth, many mothers often experience tremors that resemble shivering, although they deny feeling cold. These tremors are thought to be due to a vasomotor response involving epinephrine during the birthing process. This shivering should spontaneously resolve within 20 minutes. Choice D is incorrect. An elevated temperature following a cesarean section is highly unlikely. Hypothermia occurs more commonly immediately following. Learning Objective Identify the symptom most indicative of a common post-cesarean delivery complication. Additional Info Most post-childbirth complications fall into one of the six following categories: Hemorrhage Mood Disorders Puerperal Infections Shock Subinvolution of the Uterus Thromboembolic Disorders Last Updated - 14, Aug 2022

The health care team is determining a prenatal client's estimated due date (EDD). Which of the following is the most accurate method used to determine the estimated due date? A. Nagele's Rule B. Embryonic ultrasound C. Early hCG levels D. Chadwick's sign Submit Answer

Explanation Choice B is correct. An early ultrasound is the most accurate way to determine the estimated due date. One study found that birth occurred within seven days of the estimated due date determined by ultrasound alone. Choice A is incorrect. Nagele's rule is not the most accurate way to determine a prenatal client's due date. Choice C is incorrect. hCG levels vary from woman to woman and are not accurate in predicting a due date. Choice D is incorrect. Chadwick's sign can be used as a probable sign of pregnancy, but it does not help determine a due date. NCSBN client need Topic: Maintenance and Health Promotion, Ante / Intra / Postpartum Care Last Updated - 13, Dec 2022

The nurse is caring for a client in labor experiencing early decelerations. Which of the following actions should the nurse take? A. Reposition the patient on her side B. Document the findings C. Discontinue oxytocin infusion D. Prepare for an amnioinfusion

Explanation Choice B is correct. Early decelerations are a reassuring finding caused by infant head compression, which is a normal part of labor. Choices A, C, and D are incorrect. Repositioning the client and discontinuing an oxytocin infusion would all be appropriate interventions for a client experiencing variable or late decelerations. Early decelerations are a reassuring pattern and do not require the nurse to intervene. Additional Info Source: Archer ReviewSource : Source: Archer Review ✓ Early decelerations are not associated with fetal compromise and do not require intervention. ✓ Early decelerations occur during contractions as the fetal head is pressed against the client's pelvis or cervix. Last Updated - 31, Dec 2022

The nurse is performing a home visit to the parents of a newborn. The nurse witnesses the father's direct eye contact with the infant and shows affection. The nurse should document this finding as A. binding-in. B. engrossment. C. entrainment. D. detachment.

Explanation Choice B is correct. Engrossment is used to describe the initial paternal bonding. This is characterized by the father having a face-to-face fascination with the newborn and the ability for the father to start the bonding process. Choices A, C, and D are incorrect. Binding-in (also known as claiming) is a standard psychosocial process where the mother links the newborn's facial features to other family members. Entrainment is when the newborn moves rhythmically in response to adult speech. Detachment is unexpected and occurs when the bonding process fails and either the infant or parent has no bond with the other. Additional Info Engrossment is a positive finding in the initial bonding phase between the infant and the father. Engrossment is reinforced when the infant awakens and responds to the father's voice. Last Updated - 29, Nov 2022

The nurse is caring for a client in labor who just received epidural analgesia. The nurse should monitor the client for which adverse effects? A. Hypertension B. Bladder distention C. Hypothermia D. Precipitous labor Submit Answer

Explanation Choice B is correct. Epidural analgesia may cause bladder distention. Bladder distention may cause pain that remains after initiation of the block and may interfere with fetal descent in labor. Choices A, C, and D are incorrect. Epidural analgesia is known to cause hypotension; thus, it is routine for a client to receive a preprocedural fluid bolus of isotonic lactated ringers. Hypothermia is unlikely with epidural placement. The fever associated with epidural analgesia is usually not caused by infection but may result from reduced hyperventilation and decreased heat dissipation. Epidural analgesia commonly slows the progression of the second stage of labor because it relaxes pelvic muscles. Additional Info The epidural space is entered at the L3-L4 interspace (below the end of the spinal cord), and a catheter is passed through the needle into the epidural space. The catheter allows continuous infusion or intermittent injection of medication to maintain pain relief during labor and vaginal or cesarean birth. The infusion of epidural medication also may be regulated by a patient-controlled epidural analgesia (PCEA) pump Last Updated - 10, Jul 2022

The nurse is counseling an adolescent who is pregnant and reports frequent eating at fast-food restaurants. The nurse should make which recommendation to help optimize her nutritional intake? A. Choose french fries over a baked potato B. Select a cheeseburger over a regular hamburger C. Pick sandwiches instead of wraps D. Breaded chicken is a better choice than broiled Submit Answer

Explanation Choice B is correct. Fast food is not desired during pregnancy because of the abundance of oils, dressings, and breading that supply a high degree of saturated fats, sodium, and calories. To optimize the client's nutritional intake, if the client insists on fast food, the nurse should recommend a cheeseburger because the cheese will add protein and calcium. Choices A, C, and D are incorrect. A baked potato is preferred over french fries because a baked potato's skin is dense with vitamins. Wraps are preferred over sandwiches because of the fewer calories in the breading of a sandwich. Grilled and broiled chicken is superior to breaded chicken because of the decrease in calories. Additional Info Teaching the adolescent about nutrition can be a challenge for nurses. It is essential to establish an accepting, relaxed atmosphere and show a willingness to listen to the teenager's concerns. The nurse should keep suggestions to a minimum and focus on only the most important changes. If an adolescent believes she must eliminate all her favorite foods, she is likely to rebel. Although not a popular choice for its nutritional value, strategic ways to optimize the intake of fast food include Add cheese to hamburgers to increase calcium and protein. Include lettuce and tomato for vitamins A and C. Avoid dressings on hamburgers because they tend to be high in calories and fat. To reduce fat and calories, choose broiled, roasted, and barbecued foods (e.g., chicken breast, roast beef). Avoid fried foods (e.g., French fries, fried zucchini, onion rings, fried cheese) because they are high in fat and the high heat may destroy some vitamins. Breaded foods such as chicken nuggets and breaded clams are high in calories and absorb more oil if they are fried. Try wraps instead of sandwiches to decrease calories. Baked potatoes with broccoli, cheese, and meat fillings provide better nutrition than French fries. Milk, milkshakes, and orange juice provide more nutrients than carbonated beverages, which are high in sodium and calories. Avoid pickles, olives, and other salty foods. Too much sodium may increase swelling of the ankles. Add only small amounts of salt to foods to prevent or decrease swelling. Last Updated - 31, Oct 2022

A 29-year-old is pregnant for the first time. She calls her family care provider and tells the nurse that the first day of her last period was May 5th. She used Nagele's rule to determine this. The nurse anticipates that her estimated due date will be on what date? A. February 5th B. February 12th C. January 28th D. April 12th Submit Answer

Explanation Choice B is correct. February 12th. Remember, the equation for Nagele's Rule is subtract three months from the first day of the woman's last menstrual period plus seven days. Choices A, C, and D are incorrect. Recalculate using the equation for Nagele's Rule. NCSBN client need Topic: Health Care Promotion and Maintenance, Ante / Intra / Postpartum Care Last Updated - 12, Feb 2022

The oncoming nurse is receiving a report on a pregnant patient with HELLP syndrome. This nurse knows that HELLP syndrome, a severe progression of preeclampsia stands for: A. Half Eclipsed Lipase Levels and Preeclampsia B. Hemolysis, elevated liver enzymes, and lowered platelets C. Hematocrit elevation, low lipase, and pancreatitis D. Hemoglobin, elevated lipids, and low plasma Submit Answer

Explanation Choice B is correct. HELLP syndrome stands for Hemolysis, elevated liver enzymes, and low platelets. HELLP syndrome is a condition in which hemolysis of the red blood cells occurs creating elevated liver enzymes and low platelets. Generally, complications are prevented by delivering the fetus as soon as symptoms develop. Choices A, C, and D are incorrect. These are not associated with HELLP syndrome. NCSBN client need Topic: Physiological Adaptations, alterations in body function Additional Info Last Updated - 28, Oct 2021

The nurse is caring for a client at 29 weeks gestation who is at risk for delivering preterm. Which of the following medications would the nurse anticipate the primary healthcare provider (PHCP) to prescribe? A. Methotrexate B. Indomethacin C. Oxytocin D. Folic acid Submit Answer

Explanation Choice B is correct. Indomethacin is a cyclooxygenase inhibitor indicated as a tocolytic in preterm labor. This medication relaxes the uterus and therefore decreases uterine contractions. Choices A, C, and D are incorrect. Methotrexate would be significantly contraindicated because of its teratogenic effects. This medication is often utilized in an ectopic pregnancy. Oxytocin would be contraindicated for preterm labor since it causes uterine contractions. Folic acid is useful during pregnancy; however, this medication does not suppress uterine contractions. Additional Info ✓ Indomethacin is a tocolytic agent indicated for the prevention of preterm labor. ✓ This medication is given orally and has maternal side effects such as gastritis and reflux. ✓ The adverse effects on the fetus include enterocolitis, cardiac defects, and intraventricular hemorrhage. ✓ This medication is contraindicated if the client has renal or peptic ulcer disease. ✓ Significant caution must be taken because indomethacin may cause cardiovascular defects in the fetus if given after 32 gestational weeks. ✓ If indomethacin is given to the client, it is generally used for less than 48 hours. Last Updated - 31, Dec 2022

The mother of a 2-month old infant tells the nurse that her mother-in-law said to her that picking her baby up immediately when she cries, "spoils her baby". What would be the nurse's best response? A. "You can let your baby wait a while before picking her up." B. "Babies need to be cuddled and comforted; this does not spoil your child." C. "You need to feed her right away because crying means that she is hungry." D. "You can just let your baby cry; she will stop once she gets tired." Submit Answer

Explanation Choice B is correct. Infants need to have their security needs met by being held and cuddled. Choice A is incorrect. Not picking up the baby after she has cried does not meet the baby's need for security. Choice C is incorrect. Infants cry for many reasons. Assuming that the child is hungry and feeding each time they cry may cause overfeeding problems such as obesity. Choice D is incorrect. Letting the baby cry to sleep does not meet the baby's security needs. Last Updated - 12, Feb 2022

The nurse is caring for a newborn with erythroblastosis fetalis. The nurse understands that this disease is characterized by A. excessive red blood cell production that requires therapeutic blood donation. B. incompatibility between maternal and fetal blood. C. an excessive amount of circulating white blood cells (WBC). D. erythrocytes become shaped like a sickle and sensitive to hypoxia. Submit Answer

Explanation Choice B is correct. Infants with erythroblastosis fetalis are anemic from the destruction of RBCs. Severely affected infants may develop hydrops fetalis, a severe anemia resulting in heart failure and generalized edema. This hemolysis stems from maternal-fetal blood incompatibility. Choices A, C, and D are incorrect. Polycythemia vera is characterized by excessive red blood cell production that requires therapeutic blood donation. Excessive WBCs would be leukocytosis which is a non-specific indicator of possible infection or inflammation. Sickle cell anemia fits the description of erythrocytes becoming shaped like a sickle and sensitive to hypoxia. Additional Info The use of Rho(D) immune globulin (RhoGAM) to prevent the mother from forming antibodies against Rh-positive blood has greatly decreased the incidence of erythroblastosis fetalis. Last Updated - 18, Dec 2022

The nurse is caring for a newborn with erythroblastosis fetalis. The nurse understands that this disease is characterized by A. excessive red blood cell production that requires therapeutic blood donation. B. hemolysis of fetal erythrocytes resulting from incompatibility between maternal and fetal blood C. inability to metabolize amino acid phenylalanine, causing high levels of phenylalanine. D. erythrocytes become shaped like a sickle and sensitive to hypoxia. Submit Answer

Explanation Choice B is correct. Infants with erythroblastosis fetalis are anemic from the destruction of RBCs. Severely affected infants may develop hydrops fetalis, which is a severe anemia that results in heart failure and generalized edema. This hemolysis stems from maternal-fetal blood incompatibility. Choices A, C, and D are incorrect. Polycythemia vera is characterized by excessive red blood cell production that requires therapeutic blood donation. The inability to metabolize amino acid phenylalanine, causing high levels of phenylalanine characterizes phenylketonuria. Sickle cell anemia fits the description of erythrocytes becoming shaped like a sickle and sensitive to hypoxia. Additional Info The use of Rho(D) immune globulin (RhoGAM) to prevent the mother from forming antibodies against Rh-positive blood has greatly decreased the incidence of erythroblastosis fetalis. Last Updated - 10, Jul 2022

The nurse performs a physical assessment on a newborn and observes fine, downy hair on the cheeks and forehead. The nurse analyzes this finding as A. milia. B. lanugo. C. vernix caseosa. D. mongolian spot. Submit Answer

Explanation Choice B is correct. Lanugo is the soft, down hairs present on newborns' shoulders, back, and forehead. It is theorized that this assists in keeping the newborn warm. Choice A is incorrect. Milia are tiny white bumps that commonly appear on newborns' foreheads. These dermal cysts of keratin disappear during the first month of life. Choice C is incorrect. Vernix caseosa is a white, creamy, naturally occurring biofilm covering the fetus's skin during the last trimester of pregnancy. This is often washed off during the first bath. Choice D is incorrect. Mongolian spots are a type of gray-blue birthmark in appearance and are commonly found on the lumbar and sacral-gluteal region. Black and Asian babies are widely affected by these benign spots that disappear by six years of age. Additional Info Lanugo is the soft, down hairs present on newborns' shoulders, back, and forehead. It is theorized that this assists in keeping the newborn warm. Lanugo is commonly found with vernix caseosa, a white, creamy, naturally occurring biofilm covering the fetus's skin. Last Updated - 05, Nov 2022

A primigravida patient in her 2nd trimester calls the OB office to report a dark line on her skin in the middle of her abdomen. The phone triage nurse would recognize this as which of the following? A. Chloasma B. Linea nigra C. Goodell Sign D. Striae gravidarum Submit Answer

Explanation Choice B is correct. Linea nigra refers to the linear hyperpigmentation of the midline of the abdomen (from sternal notch to pubis). This is a frequent change that occurs during the 2nd trimester. Choice A is incorrect. Chloasma refers to a butterfly-shaped pigmentation on the face that is seen during the 1st trimester. Choice C is incorrect. Goodell sign describes the softening of the cervix due to increased vascularity, congestion, and edema. This change occurs during the 1st trimester of pregnancy. Choice D is incorrect. Striae gravidarum describes the presence of stretch marks due to weight gain, commonly seen on the breast and abdomen during pregnancy. NCSBN Client Need Subject: Topic: Reproductive, Subtopic: Pathophysiology, antepartum care Additional Info Last Updated - 10, Oct 2021

The nurse is caring for a client two weeks postpartum with reports of flu-like symptoms, headache, and tenderness to the left breast. On examination, the nurse assesses enlarged axillary lymph nodes. The client is demonstrating manifestations of A. Endometritis B. Mastitis C. Pelvic inflammatory disease D. Cystitis

Explanation Choice B is correct. Mastitis commonly occurs 2-4 weeks postpartum. The client often experiences flu-like symptoms (fever, malaise, and axillary lymphadenopathy). The affected breast usually is tender, has erythema, and is swollen. The client's manifestation classically coincides with this infection. Choices A, C, and D are incorrect. These manifestations are not consistent with these disorders. If the client were to have any of these infections, the pain or discomfort would be localized to this area which is not in the client's complaint. Additional Info Mastitis is often caused by Staphylococcus aureus, methicillin-resistant Staphylococcus aureus (MRSA), E. coli, and streptococci. The bacteria are most often carried on the skin of the mother or in the mouth or the nose of the newborn. The organism enters through an injured area on the nipple, such as a crack or blister. The primary medical treatment is antibiotics and continued emptying of the breast. Comfort measures during mastitis include applying moist heat or ice packs, breast support, bed rest, fluids, and analgesics. Last Updated - 30, Jul 2022

The client in the delivery room has just delivered her third child. The physician ordered methylergonovine (Methergine) for the client and it was promptly administered. Which manifestation would indicate to the nurse that the medication is having its intended effect? A. The client reports a decrease in pain. B. The nurse palpates a firm uterus on the client. C. The client states that she wants to empty her bladder. D. The client's blood pressure increases. Submit Answer

Explanation Choice B is correct. Methylergonovine promotes vasoconstriction and uterine contraction. A firm and contracted uterus is a sign that the medication is having its desired effect. Choice A is incorrect. Methylergonovine does not control pain. It is an ergot alkaloid that promotes vasoconstriction and uterine muscle constriction. Choice C is incorrect. Methylergonovine does not promote urine production nor stimulate urination. Choice D is incorrect. An increase in blood pressure is a side effect of methylergonovine. Its primary indication/effect is uterine contraction and vasoconstriction, which leads to a rise in blood pressure. Last Updated - 12, Feb 2022

The nurse cares for a client at 30 weeks gestation at risk of delivering preterm. Which of the following medication would the nurse anticipate the primary healthcare provider (PHCP) to prescribe? A. Penicillin G B. Nifedipine C. Oxytocin D. Misoprostol Submit Answer

Explanation Choice B is correct. Nifedipine is a calcium channel blocker indicated as a tocolytic in preterm labor. This medication relaxes smooth muscle and reduces uterine contractions. Choices A, C, and D are incorrect. Penicillin G is indicated in treating maternal infections such as syphilis or infective endocarditis. Oxytocin would be contraindicated for preterm labor as it causes uterine contractions. Misoprostol is another medication that causes uterine contractions and is frequently utilized in miscarriages. This medication would not inhibit preterm labor. Additional Info ✓ Nifedipine is a tocolytic agent indicated for the prevention of preterm labor. ✓ This medication is given orally and has very few adverse fetal effects. ✓ Maternal blood pressure should be monitored before and after this therapy because of the risk of maternal hypotension. ✓ Considering this medication may cause dizziness, the client should be advised to change positions slowly.

A nurse is caring for a client receiving an intravenous oxytocin infusion for the induction of labor. The nurse notes the client's fundus has been contracting continuously for the past five minutes. An assessment of the fetal heart rate reveals 95 beats per minute. Which of the following should be the nurse's initial action? A. Place the client in a Trendelenburg position B. Stop the oxytocin infusion C. Administer oxygen via facemask D. Administer intravenous fluids at a high rate Submit Answer

Explanation Choice B is correct. Oxytocin is used to induce labor at or near full-term gestation and to enhance labor when uterine contractions are weak and ineffective. Here, when the nurse notices the client's fundus has been contracting continuously for the past five minutes, and an assessment of fetal heart rate reveals 95 beats per minute, the nurse should immediately recognize these symptoms as evidence of fetal distress. The priority action is for the nurse to stop the oxytocin infusion. Choice A is incorrect. The nurse should place the client in a side-lying position to aid in improving fetal circulation. Placing a client in a Trendelenburg position is used in situations of cord prolapse. Choice C is incorrect. Although the nurse should start oxygen to aid in fetal circulation, this is not the priority intervention. Choice D is incorrect. The infusion of intravenous fluid benefits clients in need of hydration; however, in this case, hydration is unnecessary as the cause of this client's problem stems from the infusion of oxytocin. Learning Objective Identify the signs and symptoms of fetal distress, correlate the fetal distress to the oxytocin infusion, and recognize the need to stop the oxytocin infusion immediately. Additional Info Following the discontinuation of the oxytocin infusion, immediately notify the physician of the fetal distress and the discontinuation of the medication. Oxytocin is also commonly referred to under the brand name Pitocin. Oxytocin is also used to prevent or control uterine bleeding after delivery, to induce completion of an incomplete abortion (including miscarriages), and to promote milk ejection during lactation. Last Updated - 23, Aug 2022

Upon gathering the lab results from your prenatal client's recent blood draw, the nurse notes that the patient's red blood cell levels have decreased since before pregnancy. The nurse believes that physiological anemia of pregnancy is likely occurring. This results from which of the following? A. Decrease in circulating red blood cells B. Increase in plasma C. Increase in iron demands from the body D. Decrease in heart size Submit Answer

Explanation Choice B is correct. Physiological anemia of pregnancy occurs when there is an increase in plasma in the blood, thus "outweighing" the number of otherwise normal red blood cell levels. Physiological anemia is normocytic (normal red cell size), whereas anemia caused by iron deficiency would be "microcytic." Choice A, C, and D are incorrect. In physiological anemia, there is no decrease in the circulating red blood cells; instead, there is an increase in plasma volume, causing dilutional anemia. While pregnancy does call for increased iron demands, this does not lead to physiological anemia; instead, iron deficiency leads to pathological microcytic anemia. Finally, pregnancy causes an increase in heart size, not a decrease. NCSBN client need Topic: Health Promotion and Maintenance: Intrapartum care Last Updated - 13, Jan 2022

The nurse just finished receiving the shift report from the night nurse. Which of the following newborns should the nurse assess first? A. A 3-hour old newborn weighing 6 pounds B. A 4-hour old newborn delivered at 42 weeks C. A 6-hour old newborn that is 21 inches long D. An 8-hour old newborn delivered at 40 weeks Submit Answer

Explanation Choice B is correct. Post-maturity refers to any baby born at or beyond 42 weeks gestation (42 0/7 weeks) or at or beyond 294 days from the first day of the mother's last menstrual period (LMP). Post-maturity is also referred to as prolonged pregnancy, post-term, and post-dates pregnancy. At about 40-42 weeks, placental insufficiency ensues due to the aging placenta. Therefore, the infants rely on their subcutaneous fat reserves to sustain them after 40 to 42 weeks since the aging placenta is unable to provide the necessary nutrition. Due to these depleted subcutaneous fat reserves, the post-term infant is at risk for hypoglycemia and hypothermia. In at-risk infants, the incidence of neonatal hypoglycemia is highest in the first few hours after birth. In this case (Choice B), a 4-hour old infant delivered at 42 weeks is at-risk. Additionally, the risk of meconium aspiration is high in the post-term fetuses and can cause respiratory distress when the baby is born. The nurse should prioritize and assess this post-term infant first. Choice A is incorrect. According to the World Health Organization (WHO), the average birth weight for a full-term baby is around 7.5 lb. However, a birth weight range between 5.5 lb. (2.5 kg) and 8.2 lb. (4.0 kg) is considered normal. Small for gestational age (SGA) is defined as a birth weight of less than 10th percentile for gestational age. Large for gestational age (LGA) refers to a birth weight equal to or more than the 90th percentile for a given gestational age. Macrosomia refers to a birth weight greater than 4000 to 4500 grams ( 4 to 4.5 kg), regardless of gestational age. The infant weighing 6 pounds (Choice A) is within the normal weight range for a newborn; the nurse does not need to see this infant first. Choice C is incorrect. The average length of full-term babies at birth is 20 inches (50 cm). The normal range is between 18 to 22 inches. Macrosomia is defined based on the birth weight, not the birth length. Twenty-one inches is a bit longer than the average for most infants, but this is not a relevant finding that causes complications soon after birth. The nurse does not need to see this infant first. Choice D is incorrect. The risk of neonatal hypoglycemia is highest in the first few hours after birth in some "at-risk" infants. These "at risk" infants include those who are post-term infants (at or beyond 42 weeks gestation), late preterm (34-36.6 weeks gestation), term infants who are small for gestational age, infants of diabetic mothers, and large for gestational age infants. An infant born at 40 weeks (Choice D) is referred to as a term infant, and there are no reported problems from the previous shift. No risk factors have been mentioned. The American Academy of Pediatrics (AAP) guidelines do not recommend routine screening and monitoring of blood glucose in healthy term infants after a normal pregnancy and delivery. The nurse does not need to assess this newborn first. Last Updated - 04, Feb 2022

Your pregnant client has been hospitalized with hyperemesis gravidarum. She is given ondansetron to treat this illness. What serious side effects should the hospital nurses be watching for? A. Continued nausea and vomiting B. Prolonged QT interval C. Respiratory distress D. Constipation Submit Answer

Explanation Choice B is correct. Prolonged QT intervals have been noted as a severe side effect of ondansetron. This medication is used to treat hyperemesis gravidarum when the patient is losing weight and or unable to cope with pregnancy-related nausea. Choice A is incorrect. While the nurse should monitor for the resolution of nausea and vomiting, the continuation of emesis is not a side effect of ondansetron. Choice C is incorrect. Respiratory distress is not generally associated with ondansetron. Choice D is incorrect. Constipation is not a severe side effect of ondansetron. NCSBN client need Topic: Physiological Integrity: Pharmacological and Parenteral Therapies Additional Info Source : Archer Review Last Updated - 14, Jan 2022

A 28-week pregnant client is advised by her physician to receive an Rh immune globulin (RhoGAM) injection. The client asks the nurse the reason for the medication. Which of the following is the most appropriate response from the nurse? A. "Rh immune globulin (RhoGAM) prevents measles during pregnancy." B. "This injection prevents you from forming antibodies against your baby." C. "Rh immune globulin (RhoGAM) prevents jaundice in your baby." D. "This type of injection prevents autosomal abnormalities." Submit Answer

Explanation Choice B is correct. Rh immune globulin (RhoGAM) is administered to the unsensitized Rh-negative woman at 28 weeks of gestation to prevent sensitization, which may occur from small leaks of fetal blood across the placenta. Rh immune globulin (RhoGAM), a commercial preparation of passive antibodies against Rh factor, is repeated after birth if the woman delivers an Rh-positive infant. Choice A is incorrect. Rh immune globulin (RhoGAM) has nothing to do with the measles vaccination. The measles vaccination is contained within the MMR (measles mumps rubella) vaccine. The MMR vaccination cannot be administered to pregnant women until after delivery, including breastfeeding women. Choice C is incorrect. Rh immune globulin (RhoGAM) does not prevent neonatal jaundice. Choice D is incorrect. Rh immune globulin (RhoGAM) does not prevent autosomal abnormalities. Learning Objective Correlate the administration of Rh immune globulin (RhoGAM) injections to pregnant women to prevent the formation of antibodies against the fetus. Additional Info As a precaution, women who have Rh-negative blood are given an injection of Rh antibodies at each of the following times: At 28 weeks of pregnancy Within 72 hours after delivery of a baby who has Rh-positive blood, even after a miscarriage or an abortion After any episode of vaginal bleeding during pregnancy After amniocentesis or chorionic villus sampling Sometimes, when large amounts of the fetus's blood enter the woman's bloodstream, additional injections are needed. Last Updated - 28, Aug 2022

The nurse is taking vital signs on a pregnant client in active labor. When she inflates the blood pressure cuff, she looks at the fetal monitor and notices that the fetal heart rate increases above baseline and then returns to baseline about 15 seconds later. What is the priority nursing action? A. Notify the healthcare provider B. Document and continue to monitor C. Place the mother on her left side D. Administer 100% FiO2 via face mask Submit Answer

Explanation Choice B is correct. Since the nurse has noted a reassuring sign of the fetal heart rate, it is appropriate for her to document the finding and continue to monitor the mother. If the nurse had noticed a non-reassuring sign, other interventions would be necessary. Choice A is incorrect. The nurse has observed an acceleration in the fetal heart rate, which is an increase in fetal heart rate by 15 bpm above the baseline. An acceleration lasts about 10-15 seconds and then the heart rate returns to baseline. Accelerations are a reassuring sign observed on a fetal heart rate monitoring strip. The nurse does not need to report this to the healthcare provider. Choice C is incorrect. Accelerations are a reassuring sign observed on a fetal heart rate monitoring strip and do not require repositioning of the mother to the left side. Repositioning the mother to the left-lateral position will be needed if a non-reassuring sign (e.g. fetal bradycardia, late decelerations) is noted. Choice D is incorrect. Accelerations are a reassuring sign observed on a fetal heart rate monitoring strip and do not require administration of 100% FiO2 via face mask. NCSBN Client Need: Topic: Effective, safe care environment, Subtopic: Coordinated care; Problems with Labor and Delivery Additional Info Source : Archer Review Last Updated - 13, Feb 2022

A G1P0 client in the first trimester of pregnancy informs the clinic nurse that she has replaced coffee with hot tea at breakfast. Her hemoglobin level was 10 g/dL today. She tells the nurse that she is taking her iron supplements twice per day. Which response by the nurse would be most appropriate? A. "You're off to a great start! Tea has much less caffeine than coffee." B. "A little lemon would be a great addition to your cup of tea, as this will help the absorption of your iron pill." C. "Right now, your iron levels are low. Please eliminate all caffeine from your diet, including tea and coffee." D. "That's alright. Drinking coffee or tea will not affect the fetus." Submit Answer

Explanation Choice B is correct. Tannins are compounds found in coffees and teas, which often inhibit or decrease the body's absorption of iron. Specifically, concerning tea intake, research has shown that adding lemon juice (which is high in vitamin C) appears to cancel the inhibitory effect of tannins on iron absorption. Choice A is incorrect. There is also no evidence that caffeine itself affects iron absorption. Still, when consumed in large amounts during pregnancy, it may increase the risk of spontaneous abortion, preterm birth, and small-for-gestational-age (SGA) newborns. Additionally, large quantities of caffeine consumption also affect calcium and zinc in terms of absorption and excretion. Choice C is incorrect. There is also no evidence that complete caffeine elimination will remedy iron-deficiency anemia. A further discussion with the client's health care provider (HCP) should occur to clarify this instruction. Choice D is incorrect. The "[d]rinking coffee or tea will not affect the fetus" by the nurse is inappropriate and not based on scientific research. This comment could lead to pregnancy complications and, therefore, should not be made by the nurse. Learning Objective Identify that adding lemon to a cup of tea will help the absorption of an iron pill. Additional Info Pregnant clients require higher amounts of iron per day than non-pregnant clients. The National Institutes of Health recommends pregnant clients consume 27 mg of iron daily. Discuss herbal teas or coffee with your health care provider (HCP) prior to adding these items to your diet during pregnancy. Last Updated - 21, Oct 2022

A G1P0 client in the first trimester of pregnancy informs the clinic nurse that she has replaced coffee with hot tea at breakfast. Her hemoglobin level was 10 g/dL today. She tells the nurse that she is taking her iron supplements twice daily. Which response by the nurse would be most appropriate? A. "You're off to a great start! Tea has much less caffeine than coffee." B. "A great addition to your cup of tea would be a little lemon. It's going to help you absorb your iron pill better." C. "Right now your iron levels are low. Please eliminate all caffeine." D. "That's alright. Drinking coffee or tea won't affect the fetus." Submit Answer

Explanation Choice B is correct. Tannins are polyphenolic compounds found in plants, wood, leaves, fruits, and tea. The tannin that is present in tea decreases the absorption of iron. But adding lemon juice, which is high in vitamin C, seems to cancel the inhibitory effect of tannins on iron absorption. Choices A, C, and D are incorrect. There is also no evidence that caffeine affects the absorption of iron, but when consumed in large amounts during pregnancy, it may increase the risk of spontaneous abortion, preterm birth, and small-for-gestational-age newborns. It also affects calcium and zinc in terms of absorption and excretion. Last Updated - 20, Jul 2022

While working in the newborn nursery, you are called to L&D and asked to assign the APGAR score after birth. When you evaluate the infant at 1 minute of life, you find the following: cyanotic trunk and extremities, HR is 30 bpm, slight withdrawal when you pinch her foot, floppy muscles, and RR is 10 and irregular. What APGAR score do you assign? A. 1 B. 3 C. 5 D. 7 Submit Answer

Explanation Choice B is correct. The APGAR score is 3. The infant gets 0 points for blue skin color all over, 1 point for an HR below 100 bpm, 1 point for a minimal response to stimulation, 0 points for absent muscle tone, and 1 point for a slow and irregular respiratory rate. This APGAR score indicates severe distress; therefore, the baby needs immediate action. Choices A, C, and D are incorrect. NCSBN Client Need: Topic: Physiological Integrity, Subtopic: Physiological adaptation, Newborn Last Updated - 10, Feb 2022

You are a nurse in the L&D department of the local hospital. You are caring for a newborn born at term with APGAR scores of 8 and 10. Before discharge from the hospital, you should ensure that the newborn has received: A. Hep A (hepatitis A) vaccine B. Hep B (hepatitis B) vaccine C. RV (Rotavirus) vaccine D. DTaP (diphtheria, tetanus, and pertussis) vaccine

Explanation Choice B is correct. The Hepatitis B vaccine is given in three doses; the first dose is administered at the time of birth, the second dose at two months, and the third dose at six months of age. The Centers for Disease Control and Prevention (CDC) makes recommendations for vaccines and reviews special situations in vaccinations. Choice A is incorrect. Hep A vaccine is not given until the child is one year old. Choice C is incorrect. Rotavirus vaccine is given in 2 or 3 doses. The first dose is given at 2 months. Choice D is incorrect. Finally, the DTaP vaccine is given in 5 treatments with the first dose administered at two months. NCSBN Client Need Topic: Health Promotion and Maintenance, Sub-topic: Health Promotion/Disease Prevention; Newborn Last Updated - 14, Jan 2022

The nurse is assessing a client in the labor and delivery department. She notes that the client's abdomen remains hard between contractions and that the fetal heart rate is 99 beats per minute (bpm). Which nursing diagnosis should take priority? A. Fluid and electrolyte imbalance B. Risk for fetal demise C. Ineffective breathing problem D. Alteration in comfort Submit Answer

Explanation Choice B is correct. The client is exhibiting signs of abruptio placentae. Diagnosis of abruptio placentae is predominantly based on clinical findings. The client may exhibit a sudden onset of abdominal pain or back pain. There is a sudden onset of mild to moderate vaginal bleeding in many cases. However, blood may get trapped inside the uterus in some cases, and there may not be any vaginal bleeding despite a severe abruption. Uterine contractions often come one right after another. The uterine tone increases, and therefore, the uterus feels hard/ rigid even in between the contractions. Based on the current international guidelines, the normal fetal baseline heart rate ( FHR) ranges from 110 beats per minute (bpm) to 160 bpm A heart rate of 99 bpm is significantly lower than the normal FHR. The patient is showing signs of abruptio placentae and an accompanying decreased fetal heart rate, indicating a fetus in compromise. If unaddressed, this issue will lead to fetal death. The concern for impending fetal demise should be prioritized. Choice A is incorrect. Pregnant women have fluid retention, resulting in electrolyte imbalances; this does not take priority over fetal demise. Choice C is incorrect. The fetus may experience altered gas exchange. There are no manifestations to suggest that the client is having any difficulty breathing; therefore, this would not be considered a problem for her. Choice D is incorrect. Pain relief for the mother should be a priority, but it should not overpower the potential death of the fetus. Last Updated - 15, Nov 2021

The nurse is teaching a postpartum client about caring for her episiotomy. Which of the following statement by the client would indicate a correct understanding of the teaching? A. "I can expect to have pain and urgency with urination." B. "I should increase my fluid and fiber intake." C. "I will clean the area with hot, soapy water." D. "I should wipe in a continuous motion using a washcloth." Submit Answer

Explanation Choice B is correct. The client should be instructed to increase their fluid and fiber intake to prevent constipation because constipation may cause a client to experience significant pain. If the client is still experiencing constipation as they recover from an episiotomy, the primary healthcare provider (PHCP) may prescribe a stool softener. Choices A, C, and D are incorrect. Pain and urgency with urination are concerning symptoms of cystitis. The client should report this to the PHCP. Stinging with urination may occur but should not be painful or have an increased urgency. The client should be instructed to clean the area with a peri bottle and pat it dry. For the first 12 hours following an episiotomy, the nurse should advise the client to apply a cold compress for the first 12 hours, followed by a warm compress afterward. The client should not use a continuous motion when wiping, nor should they use a washcloth which may cause trauma to the area. Patting dry with toilet paper or using a sitz bath or peri bottle could be helpful. Additional Info ✓ An episiotomy is an incision into the perineum right before birth ✓ Indications for an episiotomy include resolution of shoulder dystocia, breech delivery, macrosomic fetus, and birth assisted with a vacuum or forceps. ✓ Infection is the main risk following an episiotomy. ✓ Fever should be reported along with foul-smelling drainage. ✓ Cold applications are applied for at least the first 12 hours, followed by warm perineal applications after 12 hours. ✓ The client should increase their fluids and fiber to prevent constipation which may aggravate their pain. ✓ It is also recommended that the client pat dries with toilet paper and irrigate the peri area with warm water. Last Updated - 25, Jan 2023

The nurse is in the postpartum unit assessing a client who gave birth 2 hours ago. The nurse notes that the client's fundus is soft and boggy. Her perineal pads have been changed twice over the past 2 hours. What is the initial action of the nurse? A. Apply pressure on the fundus. B. Massage the fundus until it is firm. C. Notify the physician. D. Elevate the client's legs. Submit Answer

Explanation Choice B is correct. The initial action of the nurse when she finds that the fundus is soft and boggy is to massage it until it is firm. Massaging the fundus stimulates it to contract and expels blood clots. Choice A is incorrect. Applying pressure on the fundus that is not contracting can lead to an inversion of the uterus and result in massive hemorrhaging. Choice C is incorrect. Notifying the physician is an inappropriate action. The nurse should implement nursing interventions first before informing the physician. Choice D is incorrect. Elevating the client's legs can help in shunting blood from the extremities to the vital organs. However, it does not help in managing uterine atony in this client. Last Updated - 01, Feb 2022

A nurse has received orders to administer a RhoGAM injection IM to a postpartum patient. Which situation is NOT a contraindication for administration of this injection? A. Administration to a patient who has a history of a systemic allergic reaction to preparations containing human immunoglobulins. B. Administration of the injection within 72 hours after delivery. C. Administration to an Rh-positive female patient. D. Administration to a patient with an elevated temperature. Submit Answer

Explanation Choice B is correct. The injection should be given within 72 hours after birth. RhoGAM is administered intramuscularly within 72 hours after birth to prevent sensitization to the Rh factor in an Rh-negative woman with an infant who is Rh-positive. This injection will prevent hemolytic disease in subsequent pregnancies. Each vial of RhoGAM is cross-matched to a specific woman. The nurse must do all appropriate checks for patient identification to avoid an error in administration. Choices A, C, and D are incorrect. Any history of a systemic allergic reaction to human immunoglobulins is a contraindication for the RhoGAM injection. RhoGAM is administered to an Rh-negative female. The injection should be withheld in a patient who has an elevated temperature. NCSBN Client Need Topic: Safe and Effective Care Environment, Subtopic: Coordinated Care, Postpartum Procedures Last Updated - 29, Dec 2021

A client in her second trimester presents to the maternity clinic expressing concern that the dark, verticle line present on the midline of her abdomen may pose a danger to her baby. Which of the following would be the most appropriate action for the nurse to take in response to the client's concern? A. Refer the client to a dermatologist for assessment. B. Educate the client that this is a common occurrence in pregnancy called linea nigra, which usually disappears after childbirth. C. Ask the client what types of foods she has been ingesting. D. Educate the client that this is a common occurrence in pregnancy called linea nigra, which typically remains following childbirth.

Explanation Choice B is correct. The linea alba—the line that marks the longitudinal division of the midline of the abdomen—darkens to become the linea nigra due to hormone changes during pregnancy. This dark line of pigmentation may extend from the symphysis pubis to as high as the top of the fundus and becomes darker as pregnancy progresses. This hyperpigmentation typically disappears after childbirth. Choice A is incorrect. There is no need for further assessment or evaluation by a dermatologist, as this is a common occurrence of pregnancy. Choice C is incorrect. There is no connection between food intake and the appearance of the linea nigra. Choice D is incorrect. Hormonal changes during pregnancy cause temporary pigmentation darkening making the linea nigra more visually prominent in some women. This hyperpigmentation is not permanent and will typically disappear following childbirth. Learning Objective Recognize that a linea nigra is a temporary hyperpigmentation that appears vertically down the abdomen due to hormone changes during pregnancy. Additional Info Those with darker complexions tend to have a more pronounced linea nigra than those with fair complexions. In most individuals, the linea nigra darkens enough to be visible in the second trimester. Following delivery, fading is gradual, occurring over several weeks or months. The exact cause of linea nigra is unknown, but the hypothesis is that the melanocyte-stimulating hormone created by the placenta causes melanin to rise during pregnancy. Source : Archer Review Last Updated - 25, Aug 2022

Your new client presented with a positive home pregnancy last night. She has abdominal pain, some vaginal bleeding. and you note an adnexal mass on palpation. You order a progesterone level, which returns as 13 ng/mL. Your initial impression is: A. Early normal pregnancy B. Possible ectopic pregnancy C. Abnormal intrauterine pregnancy D. Incorrect home pregnancy test Submit Answer

Explanation Choice B is correct. The nurse should suspect a possible ectopic pregnancy. Abdominal pain, vaginal bleeding, and an adnexal mass are the classic triad for an ectopic pregnancy. The developing chorion produces progesterone. A normal progesterone level is > 15 ng/mL. A lower than normal progesterone level is uncommon in normal pregnancies but is very common in an ectopic pregnancy. Further testing will usually be done to confirm the diagnosis. Choices A, C, and D are incorrect. NCSBN Client Need Topic: Reduction of Risk Potential, Sub-topic: Potential for Alterations in Body Systems; Antepartum Last Updated - 31, Jan 2022

The obstetric nurse is reading the prenatal client's chart. The nurse notes that the patient is suffering from preeclampsia and knows to observe for which complications in the newborn? A. Shaking and agitation [23%] B. Low birth-weight [53%] C. Abnormal kidney function [13%] D. Blurred vision

Explanation Choice B is correct. The nurse with this patient should expect an infant born with low birth weight. Preeclampsia often results in blood being shunted away from the fetus; growth restriction is commonly found in infants born to these women. Choice A is incorrect. Shaking and agitation aren't commonly connected with preeclampsia. These symptoms may be related to drug abuse or gestational diabetes. Choice C and D are incorrect. Blurred vision and abnormal kidney function affect mothers who are suffering from preeclampsia, not their infants. NCSBN client need Topic: Health Promotion and Maintenance Last Updated - 07, Feb 2022

A pregnancy-related spinal change that can alter mobility is known as: A. Ankylosing spondylosis B. Lordosis C. Scoliosis D. Kyphosis Submit Answer

Explanation Choice B is correct. The spinal change that is common in pregnancy is lordosis. This is the result of the increasing weight of the enlarging uterus and the effect of gravity. As a fetus grows, a variety of changes appear in a pregnant woman's body. The thoracic and lumbar spine curvature change, pain in the low back, and pelvic region can increase, and the balance and gait pattern also changes. Some studies report that the center of gravity of pregnant women moves towards the abdomen, resulting in an increase in lumbar lordosis, posterior tilt of the sacrum, and movement of the head to the back to compensate for the increased weight as the fetus grows. Choice A is incorrect. Ankylosing spondylosis is a form of arthritis that primarily affects the spine, although other joints can become involved. It causes inflammation of the spinal bones (vertebrae) that can lead to severe, chronic pain, and discomfort. Choice C is incorrect. Scoliosis is a sideways curvature of the spine that occurs most often during the growth spurt just before puberty. Choice D is incorrect. Kyphosis is an exaggerated, forward rounding of the back. It can occur at any age but is most common in older women. Age-related kyphosis is often due to weakness in the spinal bones that causes them to compress or crack. Other types of kyphosis can appear in infants or teens due to malformation of the spine or wedging of the spinal bones over time. NCSBN Client Need Topic: Physiological Integrity, Subtopic: Basic Care and Comfort, Musculoskeletal Changes in Pregnancy Last Updated - 12, Feb 2022

The nurse is evaluating the lab test results of one of her prenatal clients. She is eight weeks along and has a hematocrit level of 36% and hemoglobin of 11.7 gm/dL. These numbers are down from her pre-pregnancy H and H levels. The priority action of the nurse would be to: A. Call the client and request that she have her levels redrawn. B. Record these normal findings and confirm that the client is on a prenatal vitamin during her next visit. C. Report this abnormal finding to the doctor immediately. D. Notify the lab that these results are not normal and need to be re-assessed. Submit Answer

Explanation Choice B is correct. These results are typical and should be recorded as such. A drop from pre-pregnancy values is an expected phenomenon if they remain within or close to the normal range. Most women see a decrease in their hemoglobin and hematocrit levels during pregnancy. This phenomenon is known as physiological anemia and occurs as a result of increased plasma volume in the maternal bloodstream. It is essential to confirm that the client is taking prenatal vitamins. Demand for iron increases during pregnancy. Folic acid supplementation is necessary to prevent fetal neural tube defects. Prenatal vitamins will serve to address those needs. Normal hemoglobin in a pregnant client is > 11 g/dL. Normal hematocrit in a pregnant client is > 33%. Choices A, C, and D are incorrect. This mother will not need emergent care, and thus the doctor does not need to be notified regarding this expected drop from pre-pregnancy values. No lab values will need to be redrawn at this time. NCSBN Client need Topic: Maintenance and Health Promotion, Ante / Intra / Postpartum Care Last Updated - 26, Oct 2021

A client who is 31 weeks gestation arrives to the clinic reporting headache, as well as swelling in her hands and face. The nurse should take which action? A. Determine the fundal height B. Obtain vital signs C. Perform Leopold Maneuvers D. Insert an indwelling urinary catheter Submit Answer

Explanation Choice B is correct. This client has symptoms of preeclampsia. She has a headache and swelling in her hands/face. These are symptoms of preeclampsia that may get overlooked. Preeclampsia can develop after 20 gestational weeks. Clinical features of preeclampsia include: blood pressure above 140/90 mmHg, proteinuria, and swelling (usually in the hands and face). The nurse should obtain vital signs to determine the client's overall stability and help establish this potential diagnosis. The nurse should report: BP > 140/90, > 1+ proteinuria, weight gain of more than 2 lb./wk., and facial swelling. These are concerning findings for preeclampsia. Choices A, C, and D are incorrect. The client is exhibiting manifestations of preeclampsia; assessing the fundal height, performing Leopold maneuvers, and inserting an indwelling urinary catheter are unnecessary. A urine specimen is necessary to help confirm the diagnosis of preeclampsia, but it does not need to be obtained by inserting an indwelling urinary catheter. Learning Objective Understand that preeclampsia can develop anytime up to 6 weeks postpartum. Recognize the signs that need immediate attention. Additional Info Signs of Preeclampsia Source : Archer Review

A prenatal client is worried about her fetus' activity, so she performs a "kick count". She informs the nurse that while laying down, she felt ten kicks in one hour. The nurse should tell this client that: A. She'll need to come into the clinic and have a non-stress test performed. B. Ten kicks in an hour is a reassuring finding. C. She is dehydrated and should drink more water before re-trying the kick count. D. She should get up and walk for ten minutes and then re-try the test. Submit Answer

Explanation Choice B is correct. This is a reassuring finding. Ten kicks noticed during a 1 - 2 hour period are considered normal. Choices A, C, and D are incorrect. These interventions are not necessary. NCSBN client need Topic: Maintenance and Health Promotion, Ante / Intra / Postpartum Care Last Updated - 01, Dec 2021

A pregnant woman is admitted to the ER with an initial diagnosis of placenta previa. The nurse carries out orders to start an IV infusion, administer oxygen, and extract blood for laboratory tests. The client is getting anxious and asks the nurse what is happening. The nurse tells her not to worry and that everything is under control. What is the best description of the nurse's statement? A. Incorrect, the doctor should be the one to offer information and assurances. B. Questionable, because the patient has the right to understand the type of treatment and the reason for the treatment. C. Effective, because the response lowers the client's anxiety. D. Adequate, because the preparations are routine and need no explanation.

Explanation Choice B is correct. This is a violation of the client's rights. The client has the right to accurate and complete explanations about any procedures to be performed. Choice A is incorrect. In the Patients' Bill of Rights, the patient has the right to be informed by healthcare staff about any procedure that will be done to her. Choice C is incorrect. The nurse has the responsibility to inform the client regarding the procedure that is going to be performed on her. Choice D is incorrect. The procedure may be routine work for the nurse, but it is not routine for the client and should be explained to her. Last Updated - 08, Feb 2022

While performing a cardiovascular assessment on an infant at 2 hours of life, you note the following: Normal sinus rhythm HR = 178 Systolic murmur +1 pedal pulses +3 radial pulses 5 second capillary refill No edema What is the priority nursing action after this assessment? A. Continue to monitor B. Notify the health care provider C. Administer PRN acetaminophen D. Re-evaluate the patient in one hour Submit Answer

Explanation Choice B is correct. This patient is displaying signs and symptoms of congenital heart disease; specifically coarctation of the aorta. Even if you did not know which congenital heart disease they may have, you would be expected to know that the healthcare provider needs to be notified of these symptoms. Your patient is in normal sinus rhythm and has a normal heart rate for the newborn age group. The systolic murmur, the gradient in peripheral pulses, and 5 second capillary refill are all abnormal. The murmur indicates that there is an opening somewhere in the heart where there should not be. This could be an ASD, VSD, or one of the bypasses in fetal circulation (the ductus arteriosus or foramen ovale) may not have closed on their own. The gradient in pulses indicates that there is more blood flow in the top half of the body than in the lower half - this is what points to coarctation of the aorta. A capillary refill time of 5 seconds is the last abnormal sign for this patient. Capillary refill should be less than 3 seconds in a newborn - delayed capillary refill indicates poor perfusion and must be addressed quickly. It is important to recognize that these are abnormal signs and symptoms and need to be reported to the health care provider for prompt intervention. Choices A, C, and D are incorrect. These are not the priority action. NCSBN Client Need: Topic: Physiological Integrity, Subtopic: Physiological adaptation, Newborn Last Updated - 14, Feb 2022

The nurse is caring for a client in labor. The following tracing was on the fetal heart rate monitoring strip. The nurse recognizes that this tracing is a See the exhibit. A. variable deceleration. B. late deceleration. C. early deceleration. D. normal variability pattern.

Explanation Choice B is correct. This strip indicates a late deceleration. Late decelerations are visually apparent and usually symmetric in shape, with a gradual decrease and return of the fetal heart rate (FHR) to baseline. Late decelerations are caused by decreased perfusion to the fetus. Maternal repositioning is an effective intervention for this nonreassuring pattern. Other interventions include oxygen administration and the administration of intravenous isotonic fluids. Choice A is incorrect. This strip does not show a variable fetal deceleration. This strip indicates the presence of another fetal heart pattern other than variable fetal deceleration. Choice C is incorrect. This strip does not show early fetal deceleration. This strip indicates the presence of another fetal heart pattern other than initial fetal deceleration. Choice D is incorrect. This strip does not have a normal variability pattern. This strip indicates a non-reassuring finding. Additional Info Source : Archer Review Late decelerations become more concerning when they are recurrent (present with 50% or more of the uterine contractions in a 20-minute period). Late decelerations are caused by placental insufficiency. Interventions for late decelerations include - • Maternal repositioning (either lateral or hands and knees) • Intravenous fluid boluses • Administering oxygen (via nonrebreather 10 L/min) • Discontinuing oxytocin infusion (if applicable) Last Updated - 01, May 2022

The nurse is providing a 5-month pregnant woman with her options regarding birthing locations. The nurse would be most correct in suggesting which possibility to a woman who would like freedom of movement with drug-free labor and birth but is not comfortable with a home-birth? A. The nearest hospital to her home B. A birthing center C. She should continue with a home-birth if she is low risk D. A clinician's office with her OB/GYN

Explanation Choice B is correct. This woman should consider a birthing center. Birthing centers are generally drug-free, allow women to roam around the facility to relieve discomfort, and provide a home-like environment. Choice A is incorrect. The nearest hospital may not be the best location for a pregnant woman concerned about her freedom of movement and drug-free labor. Choice C is incorrect. A woman not comfortable with a home birth should not have one. Home births are more successful when the woman is confident about her birth choice. Choice D is incorrect. Most births do not occur at a clinician's office. NCSBN Client need Topic: Maintenance and Health Promotion, Ante / Intra / Postpartum Care Last Updated - 28, Jan 2022

When assessing a postpartum client, a nurse notes that the client has soaked three perineal pads in the three hours since delivery. The nurse also notes a soft fundus. The initial action for the nurse would be which of the following? A. Insert vaginal packing B. Massage the client's fundus C. Apply an ice pack over the client's perineal area D. Administer packed red blood cells Submit Answer

Explanation Choice B is correct. Uterine atony results from the inability of the uterine muscle to contract adequately following birth, leading to vaginal bleeding and/or postpartum hemorrhage. By performing a fundal massage, the nurse will attempt to stimulate the client's uterus to contract. Choice A is incorrect. Only a physician should insert vaginal packing. Choice C is incorrect. The application of an ice pack over the client's perineum would do nothing to alleviate or address the client's issue. Choice D is incorrect. The administration of blood products such as packed RBCs is incorrect for numerous reasons. First, there is no indication that a physician's order to administer RBCs exists. Second, there is no indication that this client has a low hemoglobin level. Third, although one may assume this client was typed and screened upon admission and likely signed a consent to receive blood products, one should not make such assumptions, instead taking the time to verify for oneself (in the absence of a life-threatening emergency). Learning Objective Correlate findings of vaginal bleeding and soft fundus in a postpartum client to the symptoms of uterine atony and subsequently utilize a nursing intervention of fundal massage to stimulate the client's uterus to contract. Additional Info Uterine atony is a principal cause of postpartum hemorrhage, an obstetric emergency. Although not an inclusive list, some common risk factors for uterine atony include: Prolonged labor Precipitous labor Fibroid uterus Prolonged use of oxytocin Retained placental tissue Placental disorders (such as morbidly adherent placenta, placenta previa, and abruption placentae) Body mass index (BMI) above 40 Last Updated - 29, Aug 2021

The patient has just arrived for her initial physical examination of her new pregnancy. She received a positive pregnancy test two days ago and is three days late for her period. She asks about the following tests and procedures. She is wondering when they will be performed. The nurse would be correct in explaining which assessment will likely not be completed at this time? A. Calculation of body mass index (BMI) B. Evaluation of areas prone to edema such as the hands, face, and ankles C. Fetal doppler assessment D. Pelvic examination Submit Answer

Explanation Choice C is correct. A fetal doppler assessment will not be performed this early in the pregnancy. If this woman is only three days late for her period, she is between 4 and 5 weeks pregnant. Fetal heart tones cannot be heard with the doppler until about 10 - 12 weeks. Choices A, B, and D are incorrect. Calculation of body mass index, the evaluation of areas prone to swelling, and a pelvic examination are performed at the first prenatal appointment to determine the mother's baseline health status and to develop the best plan of care. NCSBN client need Topic: Health Promotion and Maintenance, Ante / Intra / Post Partum Care Last Updated - 26, Oct 2021

The nurse is working in the NICU for the morning shift. While assessing four neonates less than 6-hours old, which neonate warrants additional attention from the nurse? A. A neonate with a molded head and overriding sutures. B. A neonate with cyanotic hands and feet that has not passed meconium. C. A neonate that is spitting up excessive mucus, with a temperature of 36.1 °C (97°F), and is dusky in appearance. D. A neonate with abdominal respirations and intermittent tremors of the extremities. Submit Answer

Explanation Choice C is correct. A neonate is expected to be pinkish in appearance. Saliva should be minimal and the normal temperature for a newborn is from 36.5 °C to 37 °C. These signs need to be evaluated by the nurse to determine whether the baby needs further assessment. Choice A is incorrect. Molding and overriding sutures in a neonate are normal and may persist for a few days. Choice B is incorrect. Acrocyanosis in the newborn may be present for 2 to 6 hours. Meconium is expected to be passed within 24 hours after delivery. Choice D is incorrect. Tremors in the neonate are common. There is no need to worry about this sign. Last Updated - 15, Jan 2022

The nurse is caring for a client in labor and discovers the client has a completely prolapsed umbilical cord. The nurse should take which action? A. Encourage the client to push at the next contraction B. Administer 2 liters/minute oxygen via nasal cannula C. Position the client knee to chest D. Obtain a prescription for oxytocin Submit Answer

Explanation Choice C is correct. A prolapsed umbilical cord is a serious finding that may lead to fetal hypoxia. The nurse must act quickly if this is suspected. Nursing and medical care will overlap, but one of the earliest interventions is to reposition the client either knee to chest, Trendelenburg, or hips elevated with pillows with a side-lying position maintained. The goal of repositioning is to position the woman's hips higher than her head to shift the fetal presenting part toward her diaphragm. Choices A, B, and D are incorrect. The client should not push at the next contraction. Pushing will progress labor, and the goal is to provide immediate care to restore perfusion to the fetus. Most cases of prolapsed umbilical cord result in an emergent cesarean section. Administering oxygen is an early priority, but giving 2 liters via nasal cannula is insufficient. The clinical standard is administering oxygen 8-10 liters via a face mask. Obtaining a prescription for oxytocin would be inappropriate. This is because the more uterine contractions, the more fetal perfusion will be demanded, which cannot be supplied because of the prolapse in the umbilical cord. Additional Info ✓ A prolapsed umbilical cord is a medical emergency ✓ The prolapse may be hidden or complete ✓ This condition should be suspected if the fetal heart monitor should show sustained bradycardia, variable decelerations or prolonged deceleration ✓ The nurse should reposition the client with the intent to position the woman's hips higher than her head to shift the fetal presenting part ✓ Acceptable positions include knee-chest, Trendelenburg, or hips elevated with pillows, with side-lying position maintained ✓ The nurse should shout for help, pause oxytocin infusion, and provide oxygen via face mask at 8-10 liters/minute ✓ Provide and maintain vaginal elevation of the presenting part using a gloved hand ✓ An emergency cesarean section is likely Last Updated - 15, Feb 2023

A nurse in the nursery is assessing a newborn in the unit. Which finding would necessitate further investigation? A. A soft spot just above the newborn's head B. Greasy, white substances that resemble cheese on the baby's neck, back, and thighs C. A single crease on the palm D. Acrocyanosis Submit Answer

Explanation Choice C is correct. A single crease on the palm is also called a Simian crease and would indicate that the child has Down's syndrome. The nurse should further assess this finding to confirm Down's syndrome in the child. Choice A is incorrect. The soft spot on the newborn's head is the anterior fontanel. They allow the bony plates of the baby's skull to flex so the baby's head can get through the birth canal. They do not close until 9-18 months. Choice B is incorrect. This is the vernix caseosa, which is a greasy, cheese-like substance on the newborn that occurs naturally to provide insulation on the newborn. Choice D is incorrect. Acrocyanosis is cyanosis of the newborn's extremities. This is a regular occurrence during the first hours of the newborn's life. Last Updated - 13, Oct 2021

A primigravida patient begins labor and is visibly upset that her family is unavailable. Which is the most appropriate approach for the nurse to take to help meet the client's needs at this time? A. Assure her that the nursing triage team will stay with her at all times. B. Encourage the client regarding her own abilities to cope and maintain a sense of control. C. Ask the client if there is someone else who wants to be her support person. D. Tell the client that they will try to locate her family. Submit Answer

Explanation Choice C is correct. Allow the client to select another individual to give support. This allows her to have someone with her until her family can be with her. Women and families have different expectations during childbearing. These expectations are shaped by their experiences, knowledge, belief systems, and social as well as family backgrounds. In most cases, a childbirth companion (or social support during birth) has been found to improve the whole birth experience. Research shows that women who receive good social support during labor and childbirth tend on average to have shorter labors, control their pain better, and often have less need for medical intervention. With these things in mind, the nurse should put forth an effort to help find a support person for the laboring mother. Keep in mind, while nursing staff and non-medical staff can offer support, this is a very emotional time for the mother. Asking the mother's preference regarding who an alternate support person should be would give her the ability to feel like she still has some control over the situation and may prevent worsening stress. Choice A is incorrect. The triage team is responsible for the initial assessment of the client. While the nursing staff and ancillary personnel will check on the mother throughout the labor process, it is unlikely that staff will be with her "at all times." Choice B is incorrect. Although the client should be encouraged, this is an emotional time, and having the support of friends/loved ones is important. Choice D is incorrect. It is most appropriate to ask the client if there is someone specific she would like to have contacted on her behalf. It is not up to the nurse to determine who should be contacted. NCSBN Client Need Topic: Psychosocial Integrity, Subtopic: Emotional Support During Labor Last Updated - 12, Jan 2022

The nurse is educating clients that are attending a prenatal class. Which of the following statements should the nurse include? A. "Chorionic Villous Sampling (CVS) may detect neural tube defects." B. "Maternal serum alpha-fetal protein (MSAFP) may determine gender." C. "Amniocentesis may be used to assess for chromosomal abnormalities." D. "A biophysical profile (BPP) assesses six variables such as fetal glucose." Submit Answer

Explanation Choice C is correct. Amniocentesis is a widely used antepartum test that may determine the gender of a fetus, the presence of neural tube defects, chromosomal abnormalities, and fetal lung maturity. This test may also be used therapeutically for polyhydramnios as it may remove some excessive amniotic fluid volumes. Choices A, B, and D are incorrect. CVS is a test that may be performed as early as week ten to determine if the fetus has any chromosomal abnormalities - not neural tube defects. MSAFP is a test that assesses neural tube defects, not gender. A BPP considers five variables, and fetal glucose is not one of these variables. Additional Info Amniocentesis is utilized to detect fetal neural tube defects and chromosomal abnormalities. This test can also be used later in pregnancy to determine fetal lung maturity. The client will need to consent before the procedure, and if the client is Rh-negative, she will need to receive prescribed RhoGAM. The client should be instructed to report post-procedure fever, decreased fetal movement, leaking of amniotic fluid, or bleeding. Last Updated - 06, Dec 2022

A client presents to the emergency department (ED) with a suspected ectopic pregnancy. The nurse anticipates which diagnostic test will confirm this finding? A. Nonstress testing B. Abdominal radiograph (x-ray) C. Transvaginal ultrasound D. Doppler transducer Submit Answer

Explanation Choice C is correct. An ectopic pregnancy (EP) is a medical emergency. The imaging of choice is a transvaginal ultrasound, as this type of ultrasound may visualize an extrauterine gestational sac with a yolk sac or embryo (with or without a heartbeat). Choices A, B, and D are incorrect. Non-stress testing is utilized in the third trimester, where an EP is typically recognized around six or eight gestational weeks. Radiography does not provide a sufficient ability to visualize an embryo. Doppler transducer for fetal heart rate would simply discern if there is a fetal heart rate and not where the embryo is located. Additional Info An ectopic pregnancy (EP) is an extrauterine pregnancy. Almost all ectopic pregnancies occur in the fallopian tube, but other possible sites include cervical, interstitial, hysterotomy (cesarean) scar, ovarian, or abdominal. Manifestations of an EP include unilateral abdominal (pelvic) pain, vaginal bleeding, and a positive pregnancy test. Rapid management is needed because life-threatening intraabdominal bleeding may occur. For EP's that have not ruptured and the woman is stable, methotrexate may be used. If that is not the case, surgical management will be necessary. Last Updated - 26, Jun 2022

The 8 to 10-week ultrasound verifies all of the following, except: A. Estimated Due Date (EDD) B. Pelvic shape C. The absence of fetal abnormalities D. Confirm pelvic health assessed at the first prenatal appointment Submit Answer

Explanation Choice C is correct. At the 8 to 10-week ultrasound; the radiologist will not be able to rule out all fetal abnormalities. Choices A, B, and D are incorrect. Ultrasounds performed during this period do help determine the due date as well as verify the pelvic shape and health. NCSBN Client need Topic: Maintenance and Health Promotion, Ante / Intra / Postpartum Care Last Updated - 13, Nov 2021

The nurse is caring for a neonate with a decreased cardiac output. If noted in this patient, which of the following is not a sign of decreased cardiac output? A. Oliguria B. Difficulty breastfeeding C. Bradycardia D. Hypotension Submit Answer

Explanation Choice C is correct. Bradycardia is not a typical symptom of decreased cardiac output in neonates. Instead, a decreased cardiac output generally results in tachycardia as the heart pumps faster to compensate. Typical signs of decreased cardiac output in an infant include oliguria, difficulty feeding, hypotension, irritability, restlessness, pallor, and decreased distal pulses. Choice A is incorrect. Oliguria is an expected finding in an infant with a decreased cardiac output. As the kidneys are perfused less efficiently in an infant with decreased cardiac output, urination reduces or ceases altogether. Choice B is incorrect. Difficulty breastfeeding may be seen in infants with low cardiac output. Feeding is increasingly difficult for babies with poor circulation. Choice D is incorrect. Hypotension is an expected finding in an infant with low cardiac output. Normal cardiac output is required to keep blood pressure regulated. Last Updated - 06, Feb 2022

The nurse is attending to a client who is 20 weeks pregnant and has completed patient education. Which of the following statements by the client indicates that she has a good understanding of her baby's development? A. "My baby is able to breathe now." B. "My baby can open his eyes." C. "My baby is about 7 ½ inches long." D. "My baby has fully grown fingernails." Submit Answer

Explanation Choice C is correct. By 20 weeks gestation, the fetus is approximately 20 cm long or 7 ½ inches. This statement reflects a proper understanding of the mother regarding fetal development and does not require further teaching. Choices A, B, and D are incorrect. There are several stages of fetal development. Pregnancy comprises the first, second, and third trimesters. In the first trimester, critical events include changes to the fertilized cell and the development of major organs and structures. During the second trimester, the organs and structures continue to develop and the woman becomes more aware of the growing fetus. During the third trimester, the fetus gains weight, matures, and prepares for life outside of the uterus. Fetal lungs do not begin the movements of respiration until 24 weeks. The placenta provides oxygen to the fetus, and the developmental function of the lungs for breathing does not occur until birth; therefore, the statement in choice A needs further teaching. The fetus can open its eyes at 28 weeks gestation, not at 20 weeks; therefore, the statement in choice B needs further teaching. Fingernails begin to grow at ten weeks gestation but are not complete until 38 weeks; therefore, the statement in choice D needs further teaching. NCSBN Client Need Topic: Health Promotion and Maintenance, Subtopic: Human Reproduction and Fetal Development Last Updated - 14, Feb 2022

While assessing a newborn infant in the nursery, you observe bounding +3 radial pulses and faint +1 pedal pulses. You also notice that the feet are cold and pale, while the hands are warm and pink. Which cardiac defect do you suspect this infant has? A. Tetralogy of Fallot (TOF) B. Hypoplastic left heart syndrome C. Coarctation of the aorta (COA) D. Transposition of the great arteries Submit Answer

Explanation Choice C is correct. Coarctation of the aorta is a narrowing of the aorta near the ductus arteriosus. Because of this narrowing, there is increased blood flow to the upper extremities and decreased blood flow to the lower extremities. That is what causes the symptoms described in the question: bounding upper pulses, faint lower pulses, and overall better perfusion to the upper extremities. Choice A is incorrect. In tetralogy of Fallot, there is a combination of four defects - an overriding aorta, pulmonary stenosis, hypertrophy of the right ventricle, and a VSD. At birth, the nurse would appreciate a murmur and mild to severe cyanosis depending on the case. The described symptoms do not fit tetralogy of Fallot. Choice B is incorrect. In hypoplastic left heart syndrome, there is underdevelopment of the left side of the heart. The nurse would note cyanosis and murmur at birth, but the described symptoms do not fit hypoplastic left heart syndrome. Choice D is incorrect. In transposition of the great arteries, the pulmonary artery leaves the left ventricle and the aorta leaves the right ventricle. These infants are severely cyanotic at birth and need surgery early in life, but the described symptoms do not fit the transposition of the great arteries. NCSBN Client Need: Topic: Physiological Integrity, Subtopic: Physiological Adaptation, Cardiovascular Last Updated - 12, Feb 2022

The nurse is caring for a client who is six weeks pregnant and inquires about her prescribed antidepressant. Which statement, if made by the client, would indicate effective understanding? A. "I will need to stop my antidepressant until after I have delivered." B. "I will not need an antidepressant while pregnant because pregnancy causes depression remission." C. "I may have to switch to a new antidepressant while I am pregnant." D. "If I continue my antidepressant medication, I must undergo a cesarean section at delivery." Submit Answer

Explanation Choice C is correct. Depression and anxiety can still be effectively treated while a client is pregnant. SSRIs (citalopram, sertraline, fluoxetine) can be effectively continued while a client is pregnant to cause mood stabilization and prevent depressive episodes. The only SSRI likely to be switched is paroxetine because of its slight risk for teratogenicity. Thus, the client may have to be switched to a safer agent like sertraline if they take paroxetine. Even electroconvulsive therapy (ECT) is safe for a pregnant client. Choices A, B, and D are incorrect. These statements are false. Not treating antenatal depression and anxiety disorders may put the client at risk for destabilizing their mood, which at worse, could lead to self-harm. Thus, depression may be effectively managed with the vast majority of SSRIs. Pregnancy does not specifically put depression into remission. Depression may actually worsen with pregnancy because of the fluctuating hormone levels. SSRI usage does not trigger an automatic cesarean section at delivery. Additional Info ✓ SSRIs are generally safe to continue during pregnancy, except for paroxetine. ✓ A client with unipolar depression and anxiety disorders should continue treatment as prescribed. ✓ If a client has ADHD, amphetamines are preferred over methylphenidate for their safety profile. Last Updated - 26, Jan 2023

While assessing a laboring mother during a contraction, the nurse notes a decrease in fetal heart rate from 150 to 120 bpm. The heart rate slows for about 10 seconds and increases back to 150 bpm as the contraction ends. Which of the following correctly classifies this observation? A. Late deceleration B. Moderate variability C. Early deceleration D. Marked variability Submit Answer

Explanation Choice C is correct. Early decelerations occur when the fetal heart rate decreases at the same time as a contraction. In this question, the nurse noted a decrease from 150 to 120 bpm with the contraction and then a return to baseline. This occurs due to the pressure of the head of the fetus on the pelvis or soft tissue, and no intervention is required by the nurse after an early deceleration. Choice A is incorrect. Late decelerations are a decrease in the fetal heart rate that occurs after a contraction. They are a non-reassuring sign on a fetal heart rate strip. In this question, the nurse noticed an early deceleration because it occurred with a contraction, not after. Choice B is incorrect. Variability on a fetal heart rate is defined as the fluctuations in the fetal heart rate from baseline. A moderate amount of variability is what is expected and is considered a reassuring sign. This question does not mention the variability of the fetal heart rate; instead, it notes an early deceleration. Choice D is incorrect. Variability on a fetal heart rate is defined as the fluctuations in the fetal heart rate from the baseline. Marked variability is a dramatically increased amount of these fluctuations. This question does not mention the variability of the fetal heart rate; instead, it notes an early deceleration. NCSBN Client Need: Topic: Physiological Integrity, Subtopic: Physiological adaptation; Problems with Labor and Delivery Additional Info Source : Archer Review Last Updated - 19, Nov 2021

The nurse notes that the 39-week pregnant client is experiencing placenta previa. Knowing the contexts surrounding this condition, the nurse refrains from performing which of the following standard procedures? A. Ultrasonography of the uterus B. Palpating the uterus to determine fetal arrangement C. Checking the cervix for dilation D. Placing the patient on the left side Submit Answer

Explanation Choice C is correct. If the prenatal client has a current case of placenta previa, the cervix should not be assessed for dilation. Placenta previa arises when the placenta develops in a problematic spot, close to or over the cervical os. To prevent bleeding or premature labor, women with placenta previa shouldn't have their cervix checked manually. Instead, an ultrasound may be performed. Choice A is incorrect. Ultrasounds may be used safely in women with placenta previa. Ultrasounds are the safest way to assess cervical dilation in a woman with this issue. Choice B is incorrect. If the physician requires, the nurse may safely palpate the abdomen and thus the uterus of a woman whose pregnancy is difficult because of placenta previa. Choice D is incorrect. Laying on her left side is an often-used position for pregnant women, including those with placenta previa. This position increases circulation to the fetus and is often a comfortable position for laboring women. Additional Info Last Updated - 09, Dec 2021

A nurse is doing an assessment on a client who is 6-hours postpartum after delivering a full-term infant. The client verbalized feeling dizzy and faint. Which is the most appropriate nursing action? A. Place the client in Trendelenburg's position B. Review the pre-delivery hemoglobin and hematocrit C. Instruct the client to get out of bed slowly and ask for help when ambulating D. Inform the nursery nurse to delay rooming-in until the client is stable Submit Answer

Explanation Choice C is correct. In the first 8 hours postpartum, orthostatic hypotension is a regular occurrence that may be manifested by feeling faint or dizzy. The nurse should reassure the client that this is normal and focus on the client's safety. The client should always be instructed to get help when getting out of bed and ambulating until the symptoms subside. Choice A is incorrect. Placing the client in Trendelenburg's position is not the most appropriate intervention. Trendelenburg's position is appropriate when there is evidence of hypovolemic shock. While this patient could have lost blood during delivery, there is no information to suspect hypovolemic shock. The most likely reason for the client's dizziness is orthostatic hypotension, thus choice C is most appropriate. Choice B is incorrect. Reviewing the pre-delivery hemoglobin and hematocrit is incorrect. Hemorrhagic shock may manifest with blood-loss anemia and low blood pressure. However, there's no suggestion in the question that severe intrapartum bleeding has occurred. Even if such intrapartum bleeding has occurred, reviewing pre-delivery hemoglobin is not reliable because examining the values obtained before the hemorrhagic event is of no help. Choice D is incorrect. Informing the nursery nurse to delay rooming-in until the client is stable is not necessary. The symptoms the client is experiencing are a regular occurrence in the first 8 hours postpartum and do not require delayed rooming-in. Keeping mother and baby together by rooming-in is recommended as a healthy birth practice. It promotes early breastfeeding and encourages maternal-infant bonding. Last Updated - 31, Jan 2022

The nurse is assessing a newborn that was delivered 8 hours ago. The nurse notices hyperactivity, a persistent shrill cry, and jitteriness. The nurse suspects which condition? A. Sepsis B. Hypoglycemia C. Drug dependence D. Hypothermia

Explanation Choice C is correct. Neonates born to drug-dependent mothers exhibit jitteriness, hyperactivity, and a shrill cry. These signs usually appear within 24 hours of being delivered. Choice A is incorrect. Signs of sepsis include varying unstable temperatures and tachycardia. The symptoms presented do not indicate sepsis. Choice B is incorrect. Hypoglycemia in the neonate is manifested by low body temperature, diaphoresis, muscle twitching, and respiratory distress. The signs presented in the question are not related to hypoglycemia. Choice D is incorrect. Signs of hypothermia in the neonate include cool extremities, acrocyanosis, bradycardia, poor feeding, and respiratory depression. Last Updated - 03, Jan 2022

A primigravida patient at 38 weeks arrives at the hospital maternity triage unit reporting a "sudden gush of clear fluid" from her vagina shortly before arrival. The maternity triage nurse suspects the patient has experienced a prelabor rupture of membranes (PROM). Which of the following is the most appropriate method for the nurse to use to confirm her suspicion? A. An internal vaginal examination B. Ultrasound C. Nitrazine paper test D. A Gram stain Submit Answer

Explanation Choice C is correct. Nitrazine paper is used to measure the pH of the fluid to determine if the amniotic membrane has ruptured. Once the fluid comes into contact with nitrazine paper, the pH of the fluid will cause the paper's color to change. The resulting color will correlate to a pH allowing the nurse to decipher whether the fluid is amniotic fluid (i.e., PROM) or vaginal fluid. Choice A is incorrect. An internal vaginal examination is done when the client is in labor to determine cervical dilatation and effacement. Here, the membranes are suspected of having ruptured, but the client is not exhibiting any active signs of labor, making an internal vaginal examination an inappropriate assessment method. Choice B is incorrect. An ultrasound cannot be used to determine whether a prelabor rupture of membranes has occurred. Choice D is incorrect. Gram staining is performed to detect the presence of bacterial infections. Learning Objective Recognize when a client presents with a possible rupture of membranes, a nitrazine paper is typically used to assess the pH level of the fluid to identify whether the fluid is amniotic. Additional Info Prelabor rupture of membranes (PROM) is leakage of amniotic fluid before the onset of labor. Diagnosis of PROM is clinical. Delivery is recommended when gestational age is ≥ 34 weeks and is generally indicated for infection or fetal compromise regardless of gestational age. Prelabor rupture of membranes (PROM) may occur at term (≥ 37 weeks) or earlier (called preterm PROM if < 37 weeks). Source : Archer Review Last Updated - 13, Feb 2022

The nurse is collecting the health history from a pregnant patient. Which of the following conditions would not put this patient at an increased risk of developing preeclampsia? A. Obesity B. Chronic hypertension C. Frequent urinary tract infections D. Multifetal gestation Submit Answer

Explanation Choice C is correct. Preeclampsia is a dangerous complication in pregnancy that is characterized by high blood pressure and protein in the urine. A history of frequent urinary tract infections would not put this patient at increased risk for developing preeclampsia. Choice A is incorrect. Obesity is associated with an increased risk of preeclampsia in pregnancy. Choice B is incorrect. Chronic hypertension is associated with an increased risk of preeclampsia in pregnancy. Choice D is incorrect. Multifetal gestation is associated with an increased risk for preeclampsia in pregnancy. NCSBN Client Need Topic: Maternal and newborn health, Subtopic: Antepartum care, alterations in body systems, pathophysiology Additional Info Source : Archer Review Last Updated - 10, Nov 2022

The nurse is providing education to a student nurse assigned to care for a pregnant mother with preeclampsia. The nursing student would not need to be corrected if they said which of the following about this condition? A. "If a woman develops high blood pressure at her first prenatal appointment she likely has preeclampsia." B. "The kidneys cause fluid overload leading to preeclampsia." C. "High blood pressure is one of the findings in preeclampsia and occurs after 20 weeks gestation." D. During preeclampsia episodes, the mother experiences frequent nausea and vomiting, sometimes preventing weight gain. Submit Answer

Explanation Choice C is correct. Preeclampsia occurs when a woman develops high blood pressure after 20 weeks of gestation. Choice A is incorrect. High blood pressure that presents before 20 weeks, usually is preexisting hypertension unrelated to pregnancy. Choice B is incorrect. Kidney function can be affected by high blood pressure but is not the defining factor of preeclampsia. Choice D is incorrect. Frequent nausea and vomiting, which sometimes limit weight gain during pregnancy, are known as hyperemesis gravidarum, not preeclampsia. NCSBN Client need Topic: Maintenance and Health Promotion, Ante / Intra /Postpartum Care Additional Info Source : Archer Review Last Updated - 10, Nov 2022

A woman in her 37th week of gestation is wary about recognizing the signs of labor. She asks the nurse how she will know when it is time to go to the labor and delivery unit. The best response by the nurse is: A. "When the mucus plug comes out." B. "When you feel a heaviness in your bladder." C. "When you see a large gush of fluid coming out of your vagina." D. "When you feel nauseated and vomit simultaneously."

Explanation Choice C is correct. Prelabor rupture of membranes (PROM) may occur at term (≥ 37 weeks) or earlier (called preterm PROM if < 37 weeks). Upon the prelabor rupture of membranes, amniotic fluid is expelled from the vagina, typically as a sudden gush of fluid from the vagina (unless complications are present). Some women experience membrane rupture as the first sign of labor onset. If this occurs, the woman should go to the birth center for evaluation. Choice A is incorrect. Although the passage of the mucus plug is a sign that the cervix is beginning to dilate, in some cases, the mucus plug is passed weeks prior to the onset of labor. Choice B is incorrect. The mother may experience bladder pressure and frequency as the fetus settles into the pelvis during the final weeks of pregnancy. Choice D is incorrect. The symptoms of nausea and vomiting are not indicative of actual labor. Learning Objective Identify prelabor rupture of membranes as the best response when a client asks how to recognize the onset of labor. Additional Info Prelabor rupture of membranes (PROM) may occur at term (≥ 37 weeks) or earlier (called preterm PROM if < 37 weeks). Preterm PROM predisposes to preterm delivery. PROM at any time increases the risk of the following: Infection in the woman (intra-amniotic infection [chorioamnionitis]), neonate (sepsis), or both Abruptio placentae Abnormal fetal presentation Intraventricular hemorrhage in neonates (which may result in neurodevelopmental disabilities (e.g., cerebral palsy)) Compression of the fetal umbilical cord Last Updated - 21, Oct 2022

Upon entering the room of a prenatal client, the nurse sees that her chart reads primigravida. As a competent health care practitioner, the nurse knows that this means that the client: A. Is experiencing a miscarriage B. Has been pregnant at least once before C. Is experiencing her first pregnancy D. Is a premium candidate for a vaginal delivery Submit Answer

Explanation Choice C is correct. Primagravida indicates that this woman is experiencing her first pregnancy. Choice A is incorrect. Miscarriage is charted as spontaneous abortion. Choice B is incorrect. If she has been pregnant before, this woman's chart will be considered multigravida. Choice D is incorrect. Primagravida has nothing to do with whether or not this woman will have a vaginal birth. NCSBN client need Topic: Maintenance and Health Promotion, Ante / Intra / Postpartum Care Last Updated - 16, Jan 2022

The nurse is taking care of a client in the fourth stage of labor. She notes that her fundus is firm but she is still bleeding profusely. What should be the nurse's first action? A. Document the findings. B. Massage the client's fundus. C. Notify the physician. D. Put the client in Trendelenburg position. Submit Answer

Explanation Choice C is correct. Profuse bleeding may indicate a laceration of the birth canal or cervix, which needs the attention of a doctor to initiate appropriate interventions. Choice A is incorrect. The nurse should document the findings; however, the nurse should notify the physician first. Choice B is incorrect. The nurse should massage the client's fundus if it is still soft to stimulate contraction. Massaging the fundus if it is already firm does not help in controlling the bleeding. Profuse bleeding may indicate a laceration of the birth canal or cervix, which needs the attention of a physician. Choice D is incorrect. Putting the client in the Trendelenburg position compromises the patient's respiratory and cardiac functioning. Last Updated - 09, Nov 2021

Which of the following maternal infections may increase the risk of developing congenital heart defects in the fetus? A. Parainfluenza B. Adenovirus C. Rubella D. Measles Submit Answer

Explanation Choice C is correct. Rubella is a maternal infection that is known to increase the risk that the fetus will have a congenital heart defect. All mothers should be tested for rubella, and if found to be positive, should have a fetal echocardiogram performed to evaluate the fetus' heart more closely. Choice A is incorrect. Parainfluenza is not known to affect the risk for congenital heart disease. Choice B is incorrect. Adenovirus is not known to affect the risk for congenital heart disease. Choice D is incorrect. Measles is not known to affect the risk for congenital heart disease. NCSBN Client Need Topic: Effective, safe care environment; Subtopic: Maternal/Fetal infection control and safety Last Updated - 30, Jan 2022

Which of the following does the Apgar scoring system indicate? A. Heart rate, cyanosis, and edema B. Heart rate, seizure activity, respiratory effort C. Irritability, heart rate, respiratory effort, muscle tone, and color D. Reflex, respiratory rate, and bleeding Submit Answer

Explanation Choice C is correct. The APGAR score is a systematic method of assessing a newborn's physical condition at birth. It is used to determine the newborn's heart rate, muscle tone, response to stimuli, and color rating by assigning a score of 0 to 2 for each category. The newborn is assessed 1 minute after birth and again at 5 minutes after birth. A score between 7 and 10 indicates that the newborn is adjusting to extrauterine life. A score below 7 indicates that medical or nursing interventions may be needed to improve the newborn's cardiorespiratory status. If the 5 minute APGAR score is below 7, the newborn should be assessed every 5 minutes until the count is 7 or higher. Choice A is incorrect. Assessment for edema is not part of the APGAR scoring system. Choice B is incorrect. Seizure activity is not part of the APGAR scoring system. Choice D is incorrect. Bleeding assessment is not part of the APGAR scoring system. NCSBN Client Need Topic: Safe and Effective Care Environment, Subtopic: Safety and Infection Control, APGAR Score Last Updated - 05, Nov 2021

A woman was admitted to the obstetric unit in active labor and has had a frank rupture of membranes. A fetal scalp electrode and intrauterine pressure catheter were inserted promptly. The woman had progressed to 8-cm dilation when the nurse noticed abrupt decreases in the fetal heart rate of 15-20 bpm that quickly returned to baseline. The changes in fetal heart rate occurred with and without contractions. At this point, the nurse should prepare to initiate a client teaching about the possibility of which procedure? A. High forceps delivery B. Oxytocin induction C. Amnioinfusion D. Cesarean birth

Explanation Choice C is correct. The fetus is experiencing variable decelerations of heart rate in the setting of ruptured membranes. Amnioinfusion refers to the infusion of a warmed isotonic solution into the uterine cavity through the IUPC. It is mostly used as a treatment to correct fetal heart rate changes caused by umbilical cord compression, indicated by variable decelerations seen on cardiotocography. It can help cushion the cord and relieve pressure when the membranes have ruptured. Choice A is incorrect. A high forceps delivery is contraindicated because it is associated with fetal and maternal tissue damage. The situation does not meet the criteria for assisted delivery with forceps. Per ACOG, appropriate forceps deliveries are indicated as in the following: 1. Outlet forceps (the scalp is visible at the vaginal opening), 2. Low Forceps (when the leaning edge of the fetal skull is at station 3), 3. Mid Forceps (when the leaning side of the fetal head is between stations 0 and 2 ). Choice B is incorrect. The fetal heart rate findings in this case of ruptured membranes describe variable decelerations caused by cord compression. Oxytocin infusion could increase the pressure on the umbilical cord. It is also not recommended when labor is progressing adequately. Choice D is incorrect. Even when there is cord compression, the situation states that compression is relieved, as indicated by the quick return of the heart rate to baseline. If the patient is stable, there is no need for immediate delivery or cesarean delivery. Last Updated - 23, Jan 2022

The maternal health nurse is taking the history and physical for a pregnant woman in her second trimester. She has several mysterious bruises on her arms and appears isolative. The nurse would provide vigilant surveillance for which common occurrence in pregnancy? A. Chronic depression B. Physiological anemia C. Domestic violence D. Acute insomnia Submit Answer

Explanation Choice C is correct. The incidence of domestic violence intensifies while a woman is pregnant. Signs of domestic violence while pregnant include the late onset of prenatal care, unexplained bruising, and depression. Other symptoms include problems adhering to prenatal care, missed appointments, and drug or alcohol abuse. Choice A is incorrect. Depression may occur during pregnancy; however, unexplained bruising is not an anticipated finding in a depressed patient. Choice B is incorrect. While the occurrence of physiological anemia rises during pregnancy, it is not associated with a late onset of prenatal care. Choice D is incorrect. Insomnia does sometimes occur during pregnancy but is unrelated to the unexplained bruising or late onset of prenatal care. NCSBN client need Topic: Psychosocial Integrity. Abuse/Neglect Last Updated - 26, Oct 2021

The nurse is caring for a client who is six hours post-partum. The client informs the nurse that they have changed their peri-pad four times in the last six hours. The nurse should take which action? A. Document the finding as expected B. Massage the client's fundus C. Assess the client for hemorrhage D. Ambulate the client to the bathroom Submit Answer

Explanation Choice C is correct. The initial action is to assess the client for postpartum hemorrhage. The normal pad count after birth is one pad every two hours. If the client should exceed this, then the client should be assessed for postpartum hemorrhage. Choices A, B, and D are incorrect. Documenting the findings as expected would be inappropriate. This client needs to be assessed for potential postpartum hemorrhage. Massaging the client's fundus and encouraging the client to void would be appropriate measures if the hemorrhage is confirmed by the nurse. This can only come through assessment. Additional Info Source : Archer Review Postpartum hemorrhage (PPH) is serious and a significant contributor to maternal death in morbidity worldwide Risk factors and causes of PPH include Multiple gestation Uterine atony Macrosomia birth (increased risk of lacerations) Hydramnios (large amniotic fluid volume making uterine contraction difficult) Retained placenta (it will now allow the uterus to fully contract) Manual removal of the placenta Clotting disorders Lacerations Any delivery that was assisted with a tool or instrument (increased risk of lacerations) Manifestations of a client experiencing PPH include excessive lochia, uterine tenderness, and unstable vital signs (tachycardia, hypotension) Treatment includes prompt recognition and activation of a PPH protocol (each facility is required to have a protocol that streamlines treatment). If uterine atony is the cause, a fundal massage is necessary. Blood product replacement, intravenous oxytocin, and/or intramuscular (IM) methylergonovine. Last Updated - 10, Nov 2022

The client is undergoing labor in the delivery room. The fetal monitor shows that there are late decelerations. What is the initial action of the nurse? A. Call the doctor immediately. [13%] B. Let the client deep-breathe slowly and relax. [3%] C. Let the client lie on her left side. [79%] D. Prepare for Cesarian delivery.

Explanation Choice C is correct. The left lateral position improves placental blood flow and oxygen supply to the fetus. This should be the nurse's first intervention. Choice A is incorrect. Late decelerations occur due to a lack of oxygen supply to the fetus. The nurse should first initiate positional changes to increase the blood supply of the fetus before calling the doctor. Choice B is incorrect. The nurse's priority when there is a late deceleration is to increase blood supply to the fetus. Deep breathing may help decrease the mother's anxiety, but restoring fetal blood flow is a priority. Choice D is incorrect. The nurse should prepare for an emergency C-section, but the initial action should be to restore blood flow to the fetus. Last Updated - 10, Jan 2022

The nurse comes into the client's room to check on her and her newborn child. The client tells the nurse that another nurse just came and took the baby back to the nursery. What would be the initial action of the nurse? A. Alert security personnel about an infant abduction and call a code. B. Ask the mother what the nurse who took her baby looked like. C. Call the nursery to ask if the baby was returned to the nursery. D. Ask the mother if she asked the nurse for a code word. Submit Answer

Explanation Choice C is correct. The nurse should always confirm first whether another staff member returned the baby to the nursery. The nurse should not cause a false alarm in the institution. A Code Pink notifies all hospital staff of a possible infant abduction. Choice A is incorrect. When the nurse is sure that the infant is not in the nursery, a Code Pink can be started. This notifies all hospital staff of a possible infant abduction. Choice B is incorrect. This will be done if the infant was not returned to the nursery, but this is not the initial action of the nurse. Choice D is incorrect. There are many safety precautions to prevent infant abductions. For example, most facilities have a code word that is changed daily. The mother must ask anyone who wants to take the infant out of the mother's room for the code word. This is not the nurse's first intervention. Last Updated - 06, Feb 2022

The nurse reviews clinical data for a client 24 hours postpartum following a vaginal delivery. Which of the following findings would require follow-up by the nurse? A. Hematocrit 46% B. Creatinine 1.0 mg/dL C. Platelets 90,000 mm3 D. White blood cell 17,000 mm3 Submit Answer

Explanation Choice C is correct. The platelet count is low and requires follow-up and notification to the primary healthcare provider (PHCP). The normal platelet count is 150-400 mm3. Causes of postpartum thrombocytopenia include exposure to certain medications, HELLP syndrome, preeclampsia, or disseminated intravascular coagulopathy. Choices A, B, and D are incorrect. These postpartum findings are normal. This hematocrit level is on the higher end of normal (the range for a female is 37-47%). Following pregnancy, the woman undergoes diuresis, which assists in normalizing the hematocrit levels within 4 to 6 weeks. The creatinine is normal and does not require follow-up (0.6-1.2 mg/dL is the expected range). Leukocytosis is a common and benign postpartum finding. The white blood cell count may be as high as 30,000/mm3 during labor and usually returns to normal limits by 6 days postpartum. Additional Info Normal postpartum findings include - ✓ A urinary output of up to 3000 mL/day may occur, especially on days 2 through 5 postpartum. ✓ Diaphoresis is a common postpartum finding ✓ White blood cell (WBC) count increases to as high as 30,000/mm3 during labor and the immediate postpartum period ✓ The hematocrit returns to normal limits 4 to 6 weeks postpartum ✓ The first stool usually occurs within 2 to 3 days postpartum. Constipation is common during the first bowel movement. ✓ Approximately 4.5 to 5.8 kg (10 to 13 lb) is lost during childbirth. This includes the weight of the fetus, placenta, and amniotic fluid and blood lost during the birth ✓ An additional 2.3 to 3.6 kilograms (kg) (5 to 8 lb) are lost as a result of diuresis and 0.9 kg to 1.4 kg (2 to 3 lb) from involution and lochia by the end of the first week Murray, S., McKinney, E. (032018). Foundations of Maternal-Newborn and Women's Health Nursing, 7th Edition. Last Updated - 13, Feb 2023

While working in the neonatal intensive care unit, the nurse assesses the client receiving continuous nasogastric feeding. In the gastrointestinal assessment, the nurse notes: Hypoactive bowel sounds The abdominal girth of 32 cm increased from 30 cm at the previous assessment. Soft abdomen, tender to palpation. No stool x 2 days. What is the priority nursing action? A. Notify the health care provider B. Continue to monitor C. Pause the infant's feeds D. Re-evaluate the abdominal girth at your next assessment Submit Answer

Explanation Choice C is correct. The priority nursing action is to pause the infant's feeds. Next, the nurse should notify the healthcare provider. Infants in the neonatal intensive care unit (NICU) have a high risk of developing necrotizing enterocolitis (NEC). One of the first signs of NEC is the inability of the infant to tolerate feedings. The feeding intolerance is evidenced by abdominal distension and bilious vomiting. This patient is showing potential signs of early necrotizing enterocolitis: hypoactive bowel sounds, increased abdominal girth, and no stool. One of the significant complications of NEC is bowel perforation. If the patient's bowel perforates, it is a medical emergency that can lead to sepsis and death. To reduce this risk, the priority nursing action is to pause the feeds to avoid further damage. Pausing the feeds is an independent nursing action in this urgent scenario, and after that, the nurse can proceed to notify the health care provider. Choices A, B, and D are incorrect. Notifying the health care provider, monitoring the patient, and reassessing the abdominal girth for worsening distension are all necessary actions. However, pausing the feeds to prevent further complications takes priority over the other listed actions. If the NEC is recognized in its early stages, feedings can be discontinued, the stomach can be decompressed by inserting an orogastric tube, and antibiotics may be started to address possible sepsis. Learning Objective Recognize the early signs of necrotizing enterocolitis ( NEC) and institute immediate nursing interventions. Discontinuation of feedings is a priority intervention once an NEC is suspected. Last Updated - 11, Dec 2021

A woman in her 37th-week of gestation is wary about complications and labor signs. She asks the nurse, how would she know if it was time to go to the labor and delivery unit? The best response is: A. "When the mucus plug is out." B. "When you feel a heaviness in your bladder." C. "When you see a large gush of fluid coming out of your vagina." D. "When you feel nauseated and vomit altogether." Submit Answer

Explanation Choice C is correct. The rupture of membranes causes the amniotic fluid to be expelled in large amounts. If the fetus has not engaged, the umbilical cord may prolapse along with the fluid; this poses a danger to both the fetus and the mother. The mother should then promptly arrive to the labor and delivery unit. Choice A is incorrect. The mucus plug may be passed several weeks before the onset of actual labor. Choice B is incorrect. The mother may experience bladder pressure and frequency when the fetus settles into the pelvis and this may occur a few weeks before labor. Choice D is incorrect. Prodromal signs of labor include nausea and vomiting but are not indicative of actual labor. Last Updated - 20, Jul 2022

The nurse reviews clinical data for a client 24 hours postpartum following a vaginal delivery. Which of the following findings would require follow-up by the nurse? A. White blood cell count 14,000 mm3 B. BUN 18 mg/dL C. Capillary blood glucose 258 mg/dL D. Urinary output 60 mL/hr Submit Answer

Explanation Choice C is correct. This blood glucose is greater than 250 mg/dL and is clinical hyperglycemia. Regardless if the client has a history of diabetes mellitus, this CBG requires follow-up because it is the only abnormal clinical data. Choices A, B, and D are incorrect. Leukocytosis is a common and benign postpartum finding. The white blood cell count may be as high as 30,000/mm3 during labor and usually returns to normal limits by six days postpartum. A BUN of 18 mg/dL is within normal limits (10-20 mg/dL is normal). Urinary output of 60 mL/hr is fine as postpartum diuresis commonly occurs. A urinary output of up to 3000 mL/day may occur, especially on days 2 through 5 postpartum. While this client does have hyperglycemia, this finding would be expected in the absence of hyperglycemia. Additional Info Normal postpartum findings include - ✓ A urinary output of up to 3000 mL/day may occur, especially on days 2 through 5 postpartum. ✓ Diaphoresis is a common postpartum finding ✓ White blood cell (WBC) count increases to as high as 30,000/mm3 during labor and the immediate postpartum period ✓ The hematocrit returns to normal limits 4 to 6 weeks postpartum ✓ The first stool usually occurs within 2 to 3 days postpartum. Constipation is common during the first bowel movement. ✓ Approximately 4.5 to 5.8 kg (10 to 13 lb) is lost during childbirth. This includes the weight of the fetus, placenta, and amniotic fluid and blood lost during the birth ✓ An additional 2.3 to 3.6 kilograms (kg) (5 to 8 lb) are lost as a result of diuresis and 0.9 kg to 1.4 kg (2 to 3 lb) from involution and lochia by the end of the first week Murray, S., McKinney, E. (032018). Foundations of Maternal-Newborn and Women's Health Nursing, 7th Edition. Last Updated - 08, Feb 2023

A client who is pregnant and is attending a prenatal class. Which statement, if made by the client, requires further teaching? A. "Since my body mass index is normal, I should be gaining 25-35 pounds." B. "It will be okay for me to continue using sugar substitutes, such as sucralose." C. "Since I am pregnant, I will have to abandon my vegan diet." D. "I will need to keep my caffeine intake less than 200 mg/day."

Explanation Choice C is correct. This statement is false and requires follow-up. A vegan diet may be continued during pregnancy if the woman is methodical in her food choices. The concern with vegan diets is the consumption of complete proteins. However, evidence has indicated that plant proteins can meet pregnancy needs. Choices A, B, and D are incorrect. These statements are true and do not require follow-up. For a woman with a normal BMI, the average weight during pregnancy should be 25-35 pounds. Sugar substitutes are permitted in moderation. The current recommendation for daily caffeine intake is not to exceed 200 mg/day. Additional Info Individuals who follow a vegan diet avoid all animal products and may have the most difficulty meeting their nutrient needs. Through careful consideration of foods and supplemental vitamins, it is entirely possible for a woman who follows the vegan diet to have a successful pregnancy. Last Updated - 09, Jul 2022

A diabetic client has just given birth to a male neonate. Which assessment finding in the newborn would warrant nursing intervention? A. Crying B. Restlessness C. Twitchiness D. Yawning Submit Answer

Explanation Choice C is correct. Twitching or jitteriness is a sign of seizures in the newborn. The nurse should inform the physician. Choices A, B, and D are incorrect. Crying, restlessness, and yawning are all normal for the newborn. Last Updated - 14, Dec 2021

A client visits an antepartum clinic for her check-up and is prescribed iron supplements. Which instruction should the nurse provide to the client regarding her iron supplement therapy? A. You must take the iron supplements with milk. B. Consume the iron supplements with meals. C. Take the iron supplement with orange juice. D. Avoid consuming large amounts of water when taking the iron supplement, as this can cause nausea. Submit Answer

Explanation Choice C is correct. Vitamin C increases the absorption of iron. Due to the vitamin C concentration in orange juice, consuming the iron supplement with orange juice would aid in the absorption of the iron supplement. Choice A is incorrect. Products containing calcium decrease the absorption of iron in the gastrointestinal tract. Therefore, avoid taking iron supplements with milk. More specifically, if possible, avoid the intake of milk, calcium, calcium supplements, and/or any antacids two hours before or after taking an iron supplement. Choice B is incorrect. Many of the nutrients contained within a meal will hinder, at minimum, a portion of the iron supplement's absorption within the gastrointestinal tract. Choice D is incorrect. Iron supplements often cause constipation in clients. If this occurs, the client may find it necessary to increase water intake. Learning Objective Identify the correct instructional statement when educating a client who was newly prescribed iron supplements. Additional Info Iron is responsible for oxygen distribution to hemoglobin and myoglobin. Iron is absorbed the best on an empty stomach. However, iron supplements can cause stomach cramps, nausea, and diarrhea in some individuals. Some clients may need to take iron with a small amount of food to circumvent these problems. Last Updated - 24, Aug 2022

The nurse is caring for a neonate experiencing cold stress. The nurse should also assess the neonate for A. hyperglycemia. B. increased muscle tone. C. hypoglycemia. D. metabolic alkalosis. Submit Answer

Explanation Choice C is correct. When a neonate develops hypoglycemia, norepinephrine is released, causing tachycardia which causes an increase in glucose metabolism. This increase in glucose metabolism depletes the neonate's reserve of glucose. If a neonate is experiencing cold stress, the nurse should warm the neonate by applying warm clothes, removing the neonate from any drafts, and ensuring the neonate is dry. The nurse should assess the neonate for hypoglycemia via heel stick once the neonate's temperature stabilizes. Choices A, B, and D are incorrect. Cold stress would deplete glucose stores because of the sympathetic response. Thus, hyperglycemia would not occur. Manifestations of hypoglycemia include poor muscle tone and a weak, jittery cry. Metabolic acidosis would develop due to cold stress because the lack of glucose would cause fat to be the fuel source, causing metabolic acidosis. Additional Info Last Updated - 27, Nov 2022

A client at 32 weeks gestation arrives at the maternity unit stating she has not felt her baby move for nearly six hours. An external fetal monitor is attached for a nonstress test (NST). The nurse attempts to reassure the client by telling her she has a reactive nonstress test. Which of the following would be indicative of a reactive nonstress test? A. Numerous decelerations lasting between 10 and 20 seconds in duration, all ranging between 15 and 20 beats/minute under the baseline heart rate. B. Fetal heart rate (FHR) of 120 - 160 beats/minute throughout the entire 20-minute monitoring session. C. Three accelerations in a 15-minute period, all ranging between 17 and 21 beats/minute over the baseline heart rate. D. One acceleration in a 20-minute period, lasting 15 seconds at 15 beats/minute over the baseline heart rate. Submit Answer

Explanation Choice C is correct. When undergoing a nonstress test (NST), results are considered reactive (reassuring) if there are a minimum of two accelerations of 15 beats/minute above the baseline, each lasting a minimum of 15 seconds over the 20-minute testing period. Choice A is incorrect. Variable decelerations occur commonly in nonstress tests. These do not provide any reassurance regarding fetal condition. Choice B is incorrect. The fetal heart rate does not provide any data regarding fetal tolerance to stress or the neurological integrity of the fetus. Choice D is incorrect. In order for a nonstress test to be deemed reactive, the results must have at least two accelerations of a minimum of 15 beats/minute above the baseline, each lasting a minimum of 15 seconds over the 20-minute testing period. A nonstress test cannot be deemed positive if there are less than two accelerations. Here, there was only one acceleration, making this test nonreassuring. Learning Objective Identify the reactive nonstress test (NST) consisting of a minimum of two accelerations of at least 15 beats/minute over the 20-minute testing period. Additional Info Source : Archer Review A nonstress test records fetal heart rate and uterine contractions using external electronic monitors and correlates the heart rate with fetal movements (reported by the mother). A nonstress test is typically performed for 20 minutes (occasionally, the exam may be extended to 40 minutes if needed). Results are considered reactive (reassuring) if there are two accelerations of 15 beats/minute over the 20-minute duration of the exam. An absence of accelerations is considered nonreactive (nonreassuring). The presence of late decelerations suggests hypoxemia, potential for fetal acidosis, and/or the need for intervention.

The nurse is caring for a married 31-year-old second-time pregnant female. Her first child is 4-years-old who was born vaginally and is considered to be a healthy preschooler. The patient explains that she's worried that her newborn child will be born with cystic fibrosis because her sister's daughter has it. What information about this genetic condition should the health care provider share? A. Her child is at an extremely high risk for passing cystic fibrosis to her child. B. Her child is not at risk for developing cystic fibrosis. C. While it is unlikely that this child will be born with cystic fibrosis, genetic testing may relieve some of the distress. D. She needs to adjust her diet in order to prevent passing on the medical condition.

Explanation Choice C is correct. While it is unlikely that this child will have cystic fibrosis, genetic testing can provide more information. Carrier testing may be recommended for people with a family history of this genetic disorder. Cystic fibrosis is an inherited autosomal recessive disorder that only appears in an infant whose parents have the mutated gene. Choice A is incorrect. There is not enough evidence to tell the patient that her child is at high risk for developing cystic fibrosis. Choice B is incorrect. There is not enough information to say that this child will not be born with cystic fibrosis. Choice D is incorrect. Her diet is irrelevant in regards to whether or not she will pass cystic fibrosis to her child. Last Updated - 11, Nov 2021

A breastfeeding mother is struggling to care for her infant with lactose intolerance. Which of the following foods should the mother avoid? A. Leafy greens B. Red meats C. Yogurt D. Wheat rolls

Explanation Choice C is correct. Yogurt is a dairy product and therefore contains lactose. Breastfeeding mothers with infants who are lactose intolerant should avoid dairy products such as cheese, milk, and yogurt. Choice A is incorrect. Leafy greens do not contain lactose and do not need to be avoided by the mother nursing a lactose intolerant infant. Choice B is incorrect. Red meat does not contain lactose and does not need to be avoided by the mother nursing a lactose intolerant infant. Choice D is incorrect. Wheat rolls generally do not contain lactose and do not need to be avoided by the mother nursing a lactose intolerant infant. NCSBN client need Topic: Physiological Integrity, Basic Care and Comfort Last Updated - 19, Feb 2021

A woman in her 38th-week gestation was given MgSO4 for pregnancy-induced hypertension (PIH). The nurse knows that this client should be monitored for: A. Blurring of vision B. Tachypnea C. Pain in the epigastrium D. Respiratory depression Submit Answer

Explanation Choice D is correct. A common side effect of this medication is respiratory depression, not tachypnea, and the client is carefully monitored for this. Choices A, B, and C are incorrect. Blurring vision and epigastric pain are indicators that pregnancy-induced hypertension (PIH) has become more severe and may precede the eclamptic phase. Last Updated - 22, Nov 2021

Which of the following clinical manifestations should the nurse document as a positive sign of pregnancy? A. Amenorrhea B. Uterine soufflé C. Positive pregnancy test D. Fetal heartbeat

Explanation Choice D is correct. A fetal heartbeat can be detected with a doppler as early as 10-12 weeks of pregnancy and is considered a positive or diagnostic sign of fertility. Signs of pregnancy can be possible, probable, or definite. Because likely signs of pregnancy may also occur when other conditions are present, the nurse needs to know what each possible indicator of pregnancy means. Choice A is incorrect. Amenorrhea, the absence of menses, is considered a possible sign of pregnancy. It is a more helpful sign when more than one cycle has been missed. Choice B is incorrect. Uterine soufflé is the sound heard on auscultation over the uterus that is caused by blood flow through the placenta. It is considered a probable sign of pregnancy, but not a definite sign of pregnancy since other conditions like uterine myomas or ovarian tumors can cause it. Choice C is incorrect. A positive pregnancy test is based on the detection of human chorionic gonadotropin. It is present during pregnancy, but other conditions can cause it to be elevated, so it is considered a probable sign of fertility. NCSBN Client Need Topic: Health Promotion and Maintenance, Subtopic: Physical and Psychological Changes of Pregnancy Additional Info Source : Archer Review Last Updated - 07, Feb 2022

The nurse is caring for a newborn immediately after delivery. Which of the following actions would be appropriate? A. Perform APGAR assessment at five and ten minutes B. Suctions the nose then the mouth C. Administer RhoGAM intramuscularly D. Place the infant skin to skin with a parent

Explanation Choice D is correct. A newborn is at risk of cold stress during the first few hours of post-intrauterine life. The nurse should dry the newborn thoroughly and place the newborn skin-to-skin with a parent. Choices A, B, and C are incorrect. The APGAR assessment is completed at one and five minutes. If suctioning is indicated, the nurse should suction the mouth, then the nose, not the nose, then the mouth. RhoGAM is indicated for Rh-negative mothers - not newborns. Medications administered to a newborn include intramuscular Vitamin K and erythromycin eye ointment. Additional Info ✓ When caring for a newborn, the nurse should perform an APGAR assessment at one and five minutes. ✓ The higher the score, the more stable the infant. Scores 7 to 10 are reassuring. ✓ The assessment evaluates a newborn's color, heart rate, reflexes, muscle tone, and respiration. ✓ Suctioning is only performed if indicated. ✓ If indicated, the correct sequence for suctioning a newborn is the mouth, then the nose. Last Updated - 31, Dec 2022

The nurse is on her shift in the nursery. Which of the following newborns would warrant further investigation and intervention from the nurse? A. A 1-hour old newborn with lanugo B. A 6-hour old newborn with a respiratory rate of 50 C. A crying 12-hour old newborn that is turning red D. A 1-day old newborn that has not yet passed meconium Submit Answer

Explanation Choice D is correct. A newborn who has not passed meconium 24 hours after birth must be assessed for intestinal obstruction or congenital abnormalities. This may be caused by an imperforate anus or several other possibilities. This newborn must be evaluated immediately. Choice A is incorrect. A newborn with lanugo is typical. This would not warrant immediate intervention. Choice B is incorrect. A standard respiratory rate for a newborn is 30 - 60; this would not warrant immediate intervention. Choice C is incorrect. A newborn turning red when crying is ordinary; this would not warrant immediate intervention. Last Updated - 04, Feb 2022

A 38-week pregnant client is scheduled to undergo a nonstress test (NST). While speaking with the nurse, the client inquires regarding the purpose of this type of testing. The most appropriate response by the nurse would be which of the following? A. "This test determines whether you are ready for labor induction." B. "A nonstress test assesses your blood sugar control." C. "This testing provides an accurate determination of fetal age." D. "A nonstress test assesses the fetal condition in the third trimester." Submit Answer

Explanation Choice D is correct. A nonstress test (NST) is a non-invasive test performed in pregnancies over 28 weeks gestation. During the procedure, fetal heart rate and uterine contractions are recorded using external electronic monitors and correlated with fetal movements as reported by the mother. This test determines the fetus's condition during the third trimester of pregnancy. Choice A is incorrect. Cervical assessment estimates whether the cervix is favorable for labor induction. More specifically, the Bishop scoring system assesses cervical readiness for labor via five factors: cervical dilation, effacement, consistency, position, and fetal station. Choice B is incorrect. Blood sugar control can be assessed during pregnancy by random blood glucose assessments or, for a larger assessment, by glycated hemoglobin (HbA1c). Additionally, during pregnancy, several tests are used to identify gestational diabetes. The first, called the Glucose Challenge Screening, is a preliminary screening test performed between 26 and 28 weeks. If a woman tests positive during this screening test, the second test, called the Oral Glucose Tolerance Test (OGTT), may be performed. Choice C is incorrect. A nonstress test does not determine fetal age. Ultrasound measurement of the embryo or fetus in the first trimester is the most accurate method to establish or confirm gestational age. Learning Objective Correctly identify a nonstress test (NST) as a non-invasive procedure that determines fetal condition during the third trimester of pregnancy. Additional Info Source : Archer ReviewSource : Archer ReviewSource : Archer ReviewSource : Archer Review Last Updated - 22, Nov 2022

A client presented to the clinic stating that her home pregnancy test was positive. The nurse understands that this is a A. positive sign of pregnancy. B. presumptive sign of pregnancy. C. possible sign of pregnancy. D. probable sign of pregnancy. Submit Answer

Explanation Choice D is correct. A pregnancy test (urine or serum) is a probable sign of pregnancy. Elevated HCG levels may be caused by other reasons such as ectopic pregnancy, user error, or recent abortion/miscarriage. Choices A, B, and C are incorrect. Positive signs of pregnancy include visualization of the embryo or fetus via ultrasound, fetal movements detected by the examiner or auscultation of fetal heart sounds. Presumptive signs of pregnancy include amenorrhea, nausea and vomiting, and an increase in urinary frequency. Possible signs of pregnancy are not a category utilized. Additional Info Presumptive signs of pregnancy Amenorrhea Nausea and vomiting Fatigue Urinary frequency Quickening (slight fluttering movement usually between 16-20 weeks gestation) Probable signs of pregnancy Goodell's sign (softening of the cervix) Chadwick's sign (bluish appearance of the cervix) Hegar's sign (softening of the isthmus of the cervix) Ballottement (sudden tap on the cervix during the vaginal examination may cause the fetus to rise in the amniotic fluid and then rebound to its original position) Braxton hicks contractions Positive pregnancy test Palpation of fetal outline Positive signs of pregnancy Fetal movements detected by an examiner Auscultation of fetal heart sounds Visualization of embryo or fetus Last Updated - 28, Oct 2022

The nurse is caring for a prenatal client with some vaginal bleeding. The nurse knows that this client could be experiencing a spontaneous abortion or miscarriage if it is occurring before ________ weeks of gestation. A. 14 B. 16 C. 18 D. 20 Submit Answer

Explanation Choice D is correct. A spontaneous abortion or miscarriage occurs before 20 weeks. Choices A, B, and C are incorrect. NCSBN client need Topic: Maintenance and Health Promotion, Ante / Intra / Postpartum Care Last Updated - 01, Dec 2021

You are caring for a newborn born at term. On your assessment. You note that central cyanosis is present and persistent at five minutes after birth. You attach a pulse oximeter to the newborn. When determining whether or not the infant requires supplemental oxygen, you know that the expected oxygen saturation at 5 minutes after birth is: A. 65-70% B. 70-75% C. 75-80% D. 80-85%

Explanation Choice D is correct. At five minutes after birth, the expected SpO2 is in the 80-85% range. Regardless of the cyanosis, if the oxygen saturation is within this range, the infant probably does not need supplemental oxygen at this point. The American Heart Association and American Academy of Pediatrics suggest the following table for Target Pre-ductal Oxygen Saturation levels following birth. Choices A, B, and C are incorrect. NCSBN Client Need Topic: Physiological Adaptation, Sub-topic: Hemodynamics, Newborn Last Updated - 02, Sep 2021

The nurse is caring for a pregnant client with heart disease undergoing labor. All of the following are appropriate nursing interventions, except: A. Attach the client to a cardiac monitor and place an external fetal monitor. B. Manage pain early in labor. C. Use controlled pushing efforts. D. Encourage ambulation. Submit Answer

Explanation Choice D is correct. Bed rest should be maintained to conserve energy and decrease cardiac stress. Choice A is incorrect. The client and the fetus should be monitored frequently to assess for fetal distress and cardiac stress. Attaching the client to a cardiac monitor and placing an external fetal monitor ensures that the staff closely follows them. Choice B is incorrect. Pain increases cardiac stress in labor. The client should have adequate pain control; an epidural might be prescribed to control pain. Choice C is incorrect. Controlled pushing efforts decrease cardiac stress and conserve the client's energy. Last Updated - 27, Jan 2022

Upon noticing fetal bradycardia, the labor and delivery nurse performs a vaginal examination on her client in labor. She discovers a pulsatile mass. What is the initial action of the nurse? A. Prepare for a Cesarean section. B. Tell the client not to push when contractions arrive. C. Escort the father out of the room. D. Place the client in Trendelenburg position. Submit Answer

Explanation Choice D is correct. Cord prolapse is seen when the umbilical cord descends before or with the fetal presenting part. It should be suspected when there is a non-reassuring fetal heart rate tracing and absent membranes. A digital vaginal exam or external inspection will help the nurse confirm the suspicion of cord prolapse. The diagnosis is confirmed by palpating a pulsatile mass in the vaginal vault. In this condition, the presenting fetal part puts pressure on the prolapsed cord, compromising the fetal blood supply. Additionally, fetal blood flow is further compromised by umbilical vasospasm that occurs due to exposure to a cold atmosphere. Compromise of fetal blood supply results in fetal hypoxia and a non-reassuring fetal heart rate pattern (fetal bradycardia or recurrent, variable decelerations). The Trendelenburg position makes use of gravity to pull the embryo back into the uterus, relieving pressure off the umbilical cord from the presenting part. Cord prolapse is an obstetric emergency. The nurse should suspect it if fetal bradycardia or variable decelerations occur, especially immediately after the rupture of membranes. The nurse should: Call for help Avoid handling the cord since it can cause further vasospasm and worsen outcomes. Manually lift the presenting part off the cord by vaginal digital exam. Do not push the cord back. Place the client in the left-lateral, Trendelenburg position, with the head down and a pillow placed under the left hip. Prepare for immediate delivery (usually via emergency Cesarean section). If delivery is not immediately available and fetal decelerations persist, consider tocolysis (i.e., terbutaline) while awaiting the Cesarean section. Tocolytics relax the uterus, stop contractions, and, therefore, relieve some pressure off the cord. Choice A is incorrect. With the fetus in distress, the nurse should prepare for an emergency C-section; however, this is not the first action of the nurse. Choice B is incorrect. In cord prolapse, the primary goal of the nurse is to ensure that the fetal blood supply and fetal oxygenation are restored. Telling the client not to push during contractions is an inappropriate action. Choice C is incorrect. The nurse may ask the client's husband to leave, especially if they are disruptive. However, this is not the primary concern at this moment. Here is a short 2-minute video on "Dos and Don'ts" of umbilical cord prolapse : Additional Info To relieve compression of the prolapsed cord, lift the presenting part off the cord. In the image above, the fetal head is gently being lifted off the prolapsed cord. Do not push the cord back. Last Updated - 27, Jan 2022

The client had just given birth and is resting in the postpartum unit when suddenly she feels a sharp pain in the chest and is having difficulty breathing. Upon assessment by the nurse, she has a heart rate of 120 and a respiratory rate of 24. She is suspected of having a pulmonary embolism. What should be the initial action of the nurse? A. Start an IV line. B. Monitor the client's blood pressure. C. Draw up morphine sulfate. D. Give oxygen via face mask at 8-10 liters per minute. Submit Answer

Explanation Choice D is correct. During a pulmonary embolism, circulation in the pulmonary bed is altered, thus affecting the oxygenation of the patient. Oxygen should be started immediately at 8-10 liters per minute to decrease hypoxia. Choice A is incorrect. Starting an IV line is necessary but it is not the priority. Choice B is incorrect. Checking the client's blood pressure is a necessary action, but it should not be the first action for the nurse to take. Choice C is incorrect. Morphine sulfate may be given for pain management; however, the nurse's initial action would be to provide oxygen. Additional Info Last Updated - 19, Dec 2022

The nurse is educating a pregnant woman with an above-average BMI about her risk factors. Which of the following issues does not correlate with an above-normal BMI pre-pregnancy? A. Gestational diabetes B. Preeclampsia C. Swelling D. Frequent UTI Submit Answer

Explanation Choice D is correct. Frequent urinary tract infections are not associated with maternal above-average body mass index. Choices A, B, and C are incorrect. The development of gestational diabetes, preeclampsia, and swelling are positively correlated with maternal above-average BMI. Other issues include increased C-section rates, stillbirth, and poor wound healing. NCSBN client need Topic: Maintenance and Health Promotion, Ante / Intra / Postpartum Care Last Updated - 31, Jan 2022

A pregnant woman with preexisting hypertension is being seen in the clinic. Her blood pressure continues to rise despite attempting first-line therapy with anti-hypertensives. Which of the following medications will be used for the prenatal patient resistant to other blood pressure-lowering medications? A. A calcium channel blocker B. Methyldopa C. Labetalol D. Hydralazine

Explanation Choice D is correct. Hydralazine is the second-line therapy for high blood pressure in prenatal patients who are not seeing any results from other medications. Choice A, B, and C are incorrect. Calcium channel blockers, methyldopa, and labetalol are all common first-line anti-hypertensives for treating prenatal clients with high blood pressure. NCSBN client need Topic: Pharmacologic and Parenteral Therapies: Parenteral/Intravenous Therapies Last Updated - 05, Jan 2022

A G3P3 client in labor tells the nurse, "I would like to breastfeed, but my breasts got so engorged last time. I could not take it. Do I have to go through that again?" Which of the following responses is most appropriate? A. "Keeping your baby on an every 4-hours schedule would help slow the milk production and lessen the engorgement." B. "You can feed your baby formula milk until your milk comes in. This will reduce stimulation and prevent engorgement." C. "You can take Parlodil to stop your milk production and prevent engorgement." D. "You need to feed your baby as soon as possible. Also, feeding your baby often would prevent breast engorgement."

Explanation Choice D is correct. Immediate and frequent breastfeeding is the key to decreasing breast engorgement in breastfeeding women. Also, the first step in treating engorgement is encouraging the mother to immediately breastfeed and continue to do so every 2 hours. The most common causes of engorgement include: A missed session of feeding or breast milk expression. Feeding the baby on a strict schedule. Less feeding by the baby due to illness/sickness. Rapid weaning from breast milk. The mother should be educated to slow down the weaning process if she experiences engorgement during weaning. Choices A and B are incorrect. Feeding and emptying the breasts less often (Choice A) and substituting it with formula (Choice B) increases the risk of engorgement. Having a strict feeding schedule increases the risk of breast engorgement. The amount of milk that can be stored in the breasts without causing engorgement varies from person to person. Therefore, following a fixed schedule of feeding/expression may predispose the mother to breast engorgement and mastitis because of inadequate milk drainage. Choice C is incorrect. Parlodel (Bromocriptine) completely stops milk production; it also has serious side effects including stroke, when given to postpartum women. Last Updated - 20, Jul 2022

During the early stages of labor, a G3P2 client informs the labor nurse: "I would like to breastfeed, but my breasts became so engorged last time, I could not take it. Do I have to go through that again?" Which of the following responses by the nurse is most appropriate? A. "Keeping your baby on a routine feeding schedule occurring every four hours will help slow the milk production and lessen the engorgement you previously experienced." B. "You can feed your baby formula milk until your milk comes in. This will reduce stimulation and prevent engorgement." C. "Speak to your health care provider (HCP) regarding taking bromocriptine to prevent or suppress your milk production to prevent engorgement." D. "You need to feed your baby as soon as possible. Also, feeding your baby frequently will help prevent breast engorgement." Submit Answer

Explanation Choice D is correct. Immediate and frequent breastfeeding is the key to decreasing breast engorgement in breastfeeding women. Also, the initial step in treating breast engorgement is encouraging the mother to immediately breastfeed following birth and continue to do so every two hours. Choice A is incorrect. Feeding and emptying the breasts every four hours increases the risk of engorgement, as does adhering to a strict feeding schedule. The amount of milk capable of being stored in the breasts without causing engorgement varies from person to person. Therefore, following a fixed schedule of feeding/expression may predispose the client to breast engorgement and mastitis due to inadequate milk drainage. Choice B is incorrect. Substituting feedings with formula significantly increases the risk of breast engorgement. Choice C is incorrect. This statement by the nurse is entirely inaccurate and contains misinformation. The medication's previous indication for the prevention of postpartum lactation was withdrawn voluntarily by the manufacturer due to safety issues in 1994. Severe and life-threatening adverse events such as hypertension, myocardial infarction, psychosis, seizures, and stroke had been (and continue to be) reported in postpartum clients when used to inhibit lactation. Bromocriptine is not a preferred medication when treatment is needed for use in breastfeeding clients with an overabundant milk supply (also known as hypergalactia). Learning Objective Identify that immediate and frequent breastfeeding is the key to decreasing breast engorgement in breastfeeding clients. Additional Info The most common causes of engorgement include: A missed session of feeding and breast milk expression. Decreased feeding by the baby due to illness. Rapid weaning from breast milk. The mother should be educated to slow the weaning process if engorgement occurs during weaning. Last Updated - 21, Oct 2022

A pregnant client is brought into an emergency department by her husband. The client reports she is currently at 37 weeks gestation and began experiencing severe abdominal pain and bright red vaginal bleeding which "runs down my legs" thirty minutes prior to arrival. She currently rates her abdominal pain 10/10. Based on this information, which assessment method should the emergency room nurse refrain from performing? A. External fetal heart rate monitoring B. Abdominal palpation C. Measurement of vital signs D. Internal vaginal examination Submit Answer

Explanation Choice D is correct. In the presence of vaginal bleeding, an internal vaginal examination is contraindicated unless performed inside an environment prepared to perform an emergent vaginal delivery or cesarean section (i.e., such as a labor and delivery unit). Additionally, this emergency room nurse is likely not permitted under hospital or emergency department policy to perform an internal vaginal examination on a full-term pregnant woman with vaginal bleeding and severe abdominal pain. Additionally, this emergency department nurse is likely not trained to perform an internal vaginal examination on a full-term pregnant woman, as this skill is not routinely performed or practiced in an emergency department. Choice A is incorrect. External fetal heart rate monitoring should be performed to determine the current status of the fetus. Continued fetal monitoring should continue to observe for any signs and symptoms to allow for early intervention. Choice B is incorrect. Abdominal palpation should be performed, as this can provide the nurse with valuable information regarding uterine contractions and abdominal tenderness, potentially assisting in determining the cause of the vaginal bleeding. Choice C is incorrect. The nurse should assess a complete set of vital signs on the client to ascertain the current physiological functioning of the client. Learning Objective Identify that as an emergency department nurse, an internal vaginal examination cannot be performed on a pregnant woman who is 37 weeks gestation complaining of severe abdominal pain and bright red vaginal bleeding. Additional Info Of note, while many emergency departments are capable of performing emergent vaginal and/or cesarean section deliveries in the emergency department, emergency department staff prefers to avoid such deliveries in the department. Preferably, emergency department staff prefers to allow the obstetrics staff who specializes in this type of care to provide care for these types of clients, resulting in improved outcomes for all involved. Last Updated - 24, Aug 2022

A nurse is assigned to care for four clients who are each one day postpartum. Following the nurse performing an initial assessment on each client, which finding would prompt further evaluation by the nurse? A. A client complaining of mild pain B. A client with a pulse rate of 65 bpm C. A client with colostrum discharge from both breasts D. A client with red, foul-smelling lochia Submit Answer

Explanation Choice D is correct. Lochia is vaginal discharge following childbirth. At one day postpartum, the client's discharge is expected to be dark red or red-brown, similar to a heavy menstrual discharge, and should have a fleshy, earthy odor. A client with foul-smelling lochia should prompt further evaluation by the nurse, as lochia which smells foul or fishy could indicate infection. Additionally, the nurse may need to elaborate further on the discharge color rather than simply using the term "red," as the anticipated discharge color at this point in the postoperative period should likely be darker in color. Choice A is incorrect. It is reasonable for a client who is one day postpartum to have mild pain; therefore, further assessment is not required. Choice B is incorrect. A pulse rate of 65 beats per minute is within normal limits and is not cause for concern. Choice C is incorrect. Colostrum discharge from the client's bilateral breasts is anticipated in a client who is one day postpartum and, therefore, does not warrant further assessment. Learning Objective Recognize that a finding of foul-smelling lochia in a postpartum client warrants additional evaluation by a nurse. Additional Info Lochia consists of the contents from the uterus accumulated during the nine months of pregnancy. The lochia will have an odor similar to a typical menstrual period. Because estimating the amount of lochia on a peripad (perineal pad) is difficult, nurses frequently document lochia in terms that are difficult to quantify, such as scant, moderate, or heavy. Typically, lochia flow is greater immediately after delivery and gradually decreases. Lochia flow is less following a cesarean birth, as some of the endometrial lining is removed during surgery. The lochia of a client who had a cesarean birth will go through the same phases as that of a client who had a vaginal birth, with the only exception of the amount being reduced. Last Updated - 27, Jan 2022

The nurse is preparing to measure the fundal height of a client at 16 gestational weeks. The nurse should prepare the client for this assessment by instructing the client to A. lay in a side-lying position with the knees bent. B. prepare for the insertion of an intravenous (IV) catheter. C. not to eat or drink two hours after this assessment. D. empty their bladder. Submit Answer

Explanation Choice D is correct. Measuring the fundal height is a painless and noninvasive way to evaluate fetal growth patterns and confirm gestational age. For this assessment, the client should empty their bladder to prevent elevation of the uterus. Choices A, B, and C are incorrect. The client should be instructed to lay supine with their knees bent. This is a noninvasive examination and does not require the client to get an intravenous catheter or refrain from eating or drinking after the assessment. Additional Info ➢ To measure the fundal height, the client should empty their bladder and lay supine with her knees slightly flexed ➢ The nurse should take a tape measure and measure the height from the top of the symphysis pubis, over the abdominal curve, to the top of the fundus ➢ At 16 gestational weeks, the fundus can be found approximately halfway between the symphysis pubis and the umbilicus ➢ At 20-22 gestational weeks, the fundus can be found at the umbilicus ➢ At 36 weeks, the fundus is at the xiphoid process Last Updated - 25, Dec 2022

The nurse is caring for a 48-hour old newborn. Based on the previous shift's report, the newborn has not yet passed stools since it was delivered. The nurse would suspect which condition? A. Celiac disease B. Cystic fibrosis C. Intussusception D. Hirschsprung's disease Submit Answer

Explanation Choice D is correct. Most healthy infants will pass meconium (first stool) by 24 hrs, and almost 100% of normal full-term neonates will pass meconium by 48 hours. Failure to pass meconium within the first 48 hours of life may indicate Hirschsprung's disease, a congenital anomaly resulting in mechanical obstruction due to inadequate motility (aganglionosis) in an intestinal segment. Choice A is incorrect. Celiac disease is a disease in which the small intestine is hypersensitive to gluten, leading to difficulty digesting food. Celiac disease is characterized by diarrhea, malabsorption, and iron deficiency anemia. Not passing stools for the first 24 hours of life is not a manifestation of Celiac disease. Choice B is incorrect. Cystic fibrosis is a generalized dysfunction of the exocrine glands leading to increased mucus secretions, particularly in the pancreas and lungs. Meconium ileus (small bowel obstruction) by thickened meconium is one of the earliest manifestations of cystic fibrosis. Meconium ileus must be differentiated from other conditions that cause the delayed passage of meconium (e.g. Hirschsprung's disease). Often, meconium ileus presents within few hours of birth with bilious emesis after the first feeding is initiated. Certainly, these infants can also present with delayed passage of meconium, but more often, obstructive ileus symptoms are present within hours after birth rather than at 48 hours. Choice C is incorrect. Intussusception is the invagination of one portion of the intestine into another. A characteristic manifestation of this condition is severe abdominal pain. Last Updated - 03, Feb 2022

The nurse is interviewing a client who reports frequent urination and nausea. The client is concerned that she may be pregnant. The nurse understands that these manifestations are A. a possible sign of pregnancy. B. probable sign of pregnancy. C. positive sign of pregnancy. D. presumptive sign of pregnancy. Submit Answer

Explanation Choice D is correct. Nausea and urinary frequency are manifestations associated with presumptive signs of pregnancy. Choices A, B, and C are incorrect. Positive signs of pregnancy include visualization of the embryo or fetus via ultrasound, fetal movements detected by the examiner or auscultation of fetal heart sounds. Presumptive signs of pregnancy include amenorrhea, nausea and vomiting, and an increase in urinary frequency. Possible signs of pregnancy are not a category utilized. Additional Info Presumptive signs of pregnancy Amenorrhea Nausea and vomiting Fatigue Urinary frequency Quickening (slight fluttering movement usually between 16-20 weeks gestation) Probable signs of pregnancy Goodell's sign (softening of the cervix) Chadwick's sign (bluish appearance of the cervix) Hegar's sign (softening of the isthmus of the cervix) Ballottement (sudden tap on the cervix during the vaginal examination may cause the fetus to rise in the amniotic fluid and then rebound to its original position) Braxton hicks contractions Positive pregnancy test Palpation of fetal outline Positive signs of pregnancy Fetal movements detected by an examiner Auscultation of fetal heart sounds Visualization of embryo or fetus Last Updated - 04, May 2022

You are working in the delivery room. The physician has inserted an endotracheal tube (ETT) in a newborn who did not respond to initial treatment. The most reliable method for confirming the placement of the ETT is: A. Observe for the rise and fall of the chest with ventilations B. Observe for increased heart rate C. Auscultate for bilateral breath sounds D. Observe for CO2 exhalation using a CO2 detector Submit Answer

Explanation Choice D is correct. Observe for CO2 exhalation using a CO2 detector. A CO2 sensor is the most reliable indicator of successful intubation. Following intubation, the team should connect a CO2 detector to the ETT. Within 8 to 10 breaths, the sensor should begin to detect exhaled CO2. If an indicator is not available, the team should observe for an increased heart rate. Choices A and C are incorrect. Observing for the rising and fall of the chest and auscultation for breath sounds are less reliable methods of confirming ETT placement. Choice B is incorrect. A rising heart rate is a positive sign that the infant is receiving oxygen with good saturation. NCSBN Client Need Topic: Basic Care and Comfort, Sub-topic: Assistive Devices; Newborn Additional Info Last Updated - 11, Dec 2022

The nurse participates in a committee reviewing the hospital security plan regarding infant abduction. Which of the following recommendations should the nurse make to the committee? A. Rearrange rooms so that the crib is near the door. B. Carry infants in the hallway instead of using the bassinet. C. Issue staff identification badges without a photo. D. Take photographs of all visitors.

Explanation Choice D is correct. Photographing all visitors and requiring visitors to sign in is fundamental to preventing infant abduction. This creates a record of the visitor, and the photograph is helpful if an abduction should occur. Choices A, B, and C are incorrect. Cribs should be located away from doors to prevent rapid abduction. Infants should always be transported in the hallway with bassinets and not carried. Carrying an infant in the hallway should raise suspicion as this is not a standard mode of transport. Staff identification badges should be unique, contain a recent photo of the employee, and have the ability to be deactivated if it is lost. Additional Info Strategies to prevent infant abduction include - Annual staff training on prevention strategies Robust logging of visitors including photographs Staff identification badges that are unique and able to be deactivated if the badge is lost. A lockdown plan that is reviewed annually. Security cameras positioned in multiple areas. High presence of security. Last Updated - 16, May 2022

The nurse is caring for a newborn immediately following birth. Which of the following actions by the nurse will prevent radiant heat loss in the newborn? A. Drying the newborns skin with a towel B. Placing the newborn on a padded, covered surface C. Using warmed, humidified oxygen D. Positioning the bassinet away from windows Submit Answer

Explanation Choice D is correct. Radiant heat loss may occur if the infant is placed near air conditioner vents or drafts. The infant's warmth is lost to the cooler object in this type of heat loss. This will occur despite the surrounding (ambient) air will not fix this problem. The nurse must remove the source of the radiant heat loss, not adjust the air temperature. Choices A, B, and C are incorrect. Drying the infant with a towel would help prevent evaporative heat loss. Placing the newborn on a padded, covered surface would prevent conductive heat loss. Convective heat loss can be prevented by warmed, humidified oxygen. Additional Info Last Updated - 24, Nov 2022

What are the expected fundal assessment findings for a woman who delivered a set of twins one hour ago via Cesarean section? A. The fundus is hard, midline, and 1-2 fingerbreadths above the umbilicus. B. The fundus need not be assessed because of the C-section. C. The fundus is to the right of the umbilicus and soft. D. The fundus is hard, midline, and at the level of the umbilicus. Submit Answer

Explanation Choice D is correct. Regardless of the mode of delivery, this is the normal postpartum fundus at one to two hours. Immediately after delivery and expulsion of the placenta, the uterus is about the size of a grapefruit and is located midline in the abdomen, halfway between the umbilicus and the symphysis pubis. Over the next several hours, the fundus will rise on the midline of the stomach to the level of or slightly above the umbilicus. Subsequently, the height of the fundus decreases by at least 1 cm or one fingerbreadth daily as the uterus goes through the process of involution. By the 10th day, the fundus is usually not palpable. Uterine "involution" refers to the return of the uterus to its pre-pregnancy size and condition. Involution begins soon after the expulsion of the placenta and occurs due to the contraction of the uterine smooth muscle. Assessing the fundus (top of the uterus) is a crucial component of post-delivery assessment—a lack of proper uterine involution results in complications such as postpartum hemorrhage. While assessing the fundus following delivery, you must evaluate for: Orientation: This represents the orientation of the fundus in relation to the umbilicus (referred by terms "to the left of the umbilicus", "midline", or "to the right of umbilicus"). Please note "midline" does not refer to the height of the fundus; instead, it refers to a side-to-side orientation in the midline (normal orientation). The fundus should immediately return to midline after delivery. If it deviates from the midline, it indicates a "distended bladder." Height: Assessing the height of the fundus is crucial to determine fundal descent because this is how the uterus returns to its original position in the true pelvis. Height is measured in fingerbreadths or centimeters (cm) in relation to the umbilicus (1 fingerbreadth = 1 to 1.5 centimeters). Please note immediately after delivery, the fundus is "below" the umbilicus- at midway between the symphysis pubis and umbilicus. The fundus then rises to the level of the umbilicus at about 1-2 hours post-delivery. At 12 hours, the fundus rises to 1 fingerbreadth above the umbilicus. Subsequently, it starts descending. In the next few days, the fundus "descends" at a rate of 1 to 2 cm every 24 hours. Around the 6th day, the fundus is located halfway between the symphysis pubis and umbilicus. A week after birth, the uterus returns to the true pelvis. At 10 to 14 days, the fundus should not be palpable. By six weeks, the uterus returns to pre-pregnancy weight. Choice A is incorrect. The fundus is located at the level of or slightly above the umbilicus at 1 to 2 hours after delivery. At 12 hours, it is about one fingerbreadth above the umbilicus. Note that the fundus rises within a few hours after birth before it begins its descent. Choice B is incorrect. The fundus must be assessed, regardless of the mode of delivery. Choice C is incorrect. The fundus should not be to the right of the midline or soft within a few hours after delivery. It should be in the midline. Right or leftward deviation indicates bladder distension. NCSBN Client Need Topic: Physiological Integrity; Subtopic: Reduction of Risk Potential Last Updated - 15, Dec 2021

A nurse at an obstetric clinic has conducted a teaching class on sexuality during pregnancy. Which of the following comments from a participant would indicate that the teaching has been effective? A. "At around the time I would normally have my period, I should abstain from intercourse." B. "I should no longer have sex during the last trimester of pregnancy." C. "My sexual desire will remain the same for the entire pregnancy." D. "The best time to enjoy sex is in the second trimester." Submit Answer

Explanation Choice D is correct. Sexual pleasure is heightened during the second trimester of pregnancy. In the second trimester, most women experience significant relief from the discomforts of early pregnancy (nausea and vomiting, breast tenderness). The uterus is not too large to interfere with comfort and rest. The second trimester is also the time when pelvic organs are congested with blood, increasing pleasure in sexual activities. Choices A and B are incorrect. As long as risk factors such as preterm labor or incompetent cervix are not present, intercourse should not harm the pregnancy. Sexual intercourse should not be a cause of concern even in the third trimester unless risk factors such as preterm labor or placenta previa are present. Choice C is incorrect. Many women experience changes in sexual desire at different stages in pregnancy, depending on their general sense of well-being and the presence of certain discomforts brought about by the pregnancy. It is not the same throughout pregnancy. Last Updated - 25, Jan 2022

The client, who is 24 weeks pregnant, is complaining to the nurse about her "worsening varicosities." The nurse would advise her to: A. Avoid exercise as blood pools in her legs during movement B. Wear knee-high hose with garters C. Avoid citrus fruits D. Sleep in a side-lying position Submit Answer

Explanation Choice D is correct. Sleeping in a side-lying position ( "SOS," Sleep On Side) moves the fetus away from the inferior vena cava ( IVC). Therefore, the fetus's weight and pressure on the IVC are minimized, promoting venous return. Better venous drainage reduces potential lower extremity swelling, varicose veins, and hemorrhoids in the pregnant woman. Sleeping supine during pregnancy must be avoided because it increases the risk of late stillbirths. Traditionally, the left side-lying position is preferred over the right side because there is a theoretical risk of IVC compression due to the liver being present on the ride side. If the pregnant client is comfortable sleeping on the right side, it should not be discouraged because a 2019 meta-analysis has revealed both left and right-side lying positions are equally safe in pregnancy. Choice A is incorrect. Contrary to the statement, exercise promotes venous return when coupled with frequent rest periods. Pregnant women should be encouraged to exercise regularly. Choice B is incorrect. The client should avoid wearing knee-high stockings with garters. Garters are tight straps present at the top of the stockings to prevent the stockings from slipping down the leg. These garters can cause occlusion of the blood flow and increase the venous pressure in the extremities. The nurse must ensure that the stockings are not causing a garter effect at the knee or thigh level. The nurse should advise the pregnant client to wear graduated compression stockings ( GCS) or thrombo-embolus deterrent stockings ( TEDs). These should be applied up to above the point of varicose enlargement. In pregnancy, GCS or TEDs also serve as mechanical prophylaxis against deep vein thrombosis ( DVT). Choice C is incorrect. The client should increase her intake of vitamin C found in citrus fruits as vitamin C is involved in forming blood vessel collagen and endothelium. Additionally, vitamin C helps increase iron absorption in the gastrointestinal tract. Learning Objective Understand that varicosities are common in pregnancy, and the clients should be advised regarding side-lying sleep positions and graduated compression stockings. Additional Info Varicose veins are a common symptom during pregnancy, affecting about 40% of pregnant women. The gravid uterus puts pressure on the IVC, increasing venous pressure in the pelvis and lower extremities. In addition to the varicose veins in the lower extremities, varicosities in the vulvar region and hemorrhoids are also common in pregnancy. Women should be informed that these do not cause harm. Side-lying position and compression stockings can improve the varicose-related symptoms, but the evidence shows these measures will not prevent varicose veins from actually appearing. Last Updated - 23, Feb 2022

The prenatal client is 7 months pregnant and wants to start an exercise program. The nurse should suggest which of the following exercises to the patient? A. Bike riding B. Circuit training C. Aerial yoga D. Swimming

Explanation Choice D is correct. Swimming is the best exercise at this point in the mother's pregnancy. Swimming is low impact and requires no balance, which can be troublesome with the weight a woman carries in her third trimester. Choices A, B, and C are incorrect. These activities are too high of an intensity for a woman who is just starting an exercise regimen and require careful steadiness. NCSBN client need Topic: Health Promotion and Maintenance Last Updated - 29, Jan 2022

At the initial prenatal visit, and often the subsequent visits, the health care provider will obtain a clean catch urine specimen to look for all of the following, except: A. Ketones B. Sexually transmitted infections C. Glucose D. Testosterone levels Submit Answer

Explanation Choice D is correct. Testosterone is not typically measured in prenatal clients. If a testosterone level needs to be measured, it will be estimated via a blood draw rather than a urinalysis. Choices A, B, and C are incorrect. These are all things tested at the initial and subsequent prenatal visits. Ketones and glucose labs are measured to identify the presence of diabetes. STI labs help health care providers find the best course of a treatment plan for the prenatal client. NCSBN client need Topic: Health Promotion and Maintenance, Ante / Intra / Postpartum Care Last Updated - 26, Oct 202

The nurse in the postpartum ward is looking at laboratory results of clients that just arrived. The nurse would go to which client immediately? A. A patient with WBC of 15,000/cubic mm B. A patient with a creatinine level of 0.8 mg/dL C. A patient with a platelet count of 360,000/cubic mm D. A client with a blood glucose of 260 mg/dL Submit Answer

Explanation Choice D is correct. The average blood glucose level is 70 - 120 mg/dL. The client's blood glucose level is 260, thus warranting the attention and intervention of the nurse. Choice A is incorrect. During labor and after birth, the WBC count would rise to 25,000. This is a normal response of the body and should not warrant any concern. Choice B is incorrect. The serum creatinine level is within normal limits (0.6-1.2 mg/dL for males and 0.5-1.1 mg/dL for females). This does not need any intervention. Choice C is incorrect. Normal platelet count is 150,000 to 450,000. This is within normal limits. Last Updated - 01, May 2022

Increased levels of which of the following hormones is related to hyperemesis gravidarum? A. Testosterone B. Progesterone C. Aldosterone D. Estrogen

Explanation Choice D is correct. The cause of hyperemesis is thought to be related to high levels of estrogen and human chorionic gonadotropin (hCG). Nausea and vomiting, also known as morning sickness, are common during the first trimester of pregnancy for many women. If nausea and vomiting interfere with an adequate intake of fluid and food and persists past 20 weeks of gestation, it is termed hyperemesis gravidarum. The cause is unknown, but elevated hormone levels and the relaxation of smooth muscles, which results in delayed gastric emptying, are believed to contribute to this condition. Hyperemesis can cause problems for the mother and fetus. Severe hyperemesis gravida- darum can result in preterm labor. The dehydration that occurs may lead to reduced placental perfusion and inadequate oxygenation to the fetus. Fetal growth can be compromised, leading to an infant who is small for gestational age. Also, women with hyperemesis gravidarum in the second trimester have an increased risk for preterm labor, pre-eclampsia (i.e. an increase in blood pressure, protein in the urine, and edema), and placental abruption. Choice A is incorrect. Testosterone is the primary male hormone. Choice B is incorrect. Progesterone is a relaxant and does not promote vomiting. Choice C is incorrect. Aldosterone is a steroid hormone. NCSBN Client Need Topic: Health Promotion and Maintenance, Subtopic: Care of the Woman with Hyperemesis Gravidarum Last Updated - 22, Jan 2022

A nurse in a gynecology clinic is assessing a first-time client (G1P0) who is eight weeks pregnant. Which assessment finding would alert the nurse of a high-risk pregnancy? A. The client reports nausea and vomiting four to five mornings per week. B. The client expresses her ambivalence toward the pregnancy to the nurse. C. The client reports intermittent constipation since learning she was pregnant. D. The client reports intermittent vaginal spotting and abdominal cramping.

Explanation Choice D is correct. The first sign of threatened abortion is vaginal bleeding, which is relatively common during early pregnancy. Approximately 25% of pregnant women experience "spotting" or bleeding in early pregnancy, and up to 50% of these pregnancies end in spontaneous abortion. Vaginal bleeding, which may be brief or last for weeks, may be accompanied by uterine cramping, persistent backache, or feelings of pelvic pressure. These added symptoms are more likely to be associated with loss of pregnancy. Choice A is incorrect. Nausea and vomiting are a result of fluctuating hormone levels during pregnancy. Unless nausea and vomiting cause severe dehydration and electrolyte loss, the pregnancy is not considered in jeopardy. Choice B is incorrect. Ambivalence is a normal emotional response to pregnancy, especially in early pregnancy, when the mother realizes a separate individual is growing inside her. The mother may have various feelings, including excitement, anxiety, apprehension, trepidation, worry, fear, eagerness, and anticipation. Moreover, various factors pertaining to the relationship with the father of the child may influence the client's emotional response to the pregnancy. Choice C is incorrect. Increased progesterone level is associated with the relaxation of smooth muscles, resulting in stasis of urine and increasing the risk for urinary tract infections and constipation. Learning Objective Correlate vaginal spotting and abdominal cramping in the early stages of pregnancy as the first signs of a threatened abortion, rendering the client a high-risk pregnancy. Additional Info Women should be advised to notify their health care provider (HCP) if brownish or red vaginal bleeding is noted. Bleeding during the first half of pregnancy should be considered a threatened abortion. The nurse should educate the client on other danger signs, which, if noticed, should prompt the client to immediately contact the clinic, including fever and chills, severe headaches with blurred vision, generalized swelling of the face and limbs, and absence of fetal movement (once the pregnancy is further along). Last Updated - 25, Aug 2022

Shortly after checking into the obstetrics unit, a client currently at 39 weeks gestation spontaneously ruptured her membranes when ambulating to the bathroom. After the client returns to bed, which of the following should be the nurse's initial action? A. Assess the color and quantity of the fluid. B. Perform a vaginal examination to assess the cervix for dilation. C. Inform the client she is now on strict bed rest until further notice. D. Assess the fetal heart tones.

Explanation Choice D is correct. The priority is for the nurse to assess the fetal status following the spontaneous rupture of the client's membranes. Although numerous methods may be utilized to evaluate fetal status, the assessment of fetal heart tones provides reliable information in a relatively prompt manner. Following the assessment of fetal heart tones, the nurse should then assess the color and quality of the amniotic fluid. Choice A is incorrect. Following a spontaneous rupture of the client's membranes, the priority is for the nurse to assess the fetal status via fetal heart tones. Assessment of the color and quantity of the amniotic fluid does not provide any indication regarding the current status of the fetus. Choice B is incorrect. While certain circumstances may justify performing a vaginal examination to assess for cervical dilatation, nothing in this scenario suggests such intervention. As such, in this scenario, the nurse's priority following a spontaneous rupture of the client's membranes remains to assess the fetal status via fetal heart tones. Choice C is incorrect. Fetal head engagement and descent are not dependent on the client's movement during labor. Keeping the client on bed rest is not necessary. Often, the client will be encouraged to ambulate. Learning Objective Following the spontaneous rupture of a 39-week gestation pregnant client's membranes, prioritize assessing the fetal status via assessment of the fetal heart tones. Additional Info To confirm the rupture of a client's membranes, the pH of the fluid may be tested. Vaginal fluid may be tested with Nitrazine paper, which turns deep blue at a pH >6.5 (pH of amniotic fluid is 7.0 to 7.6). Amniotic fluid should be assessed for meconium-stained fluid, other discoloration, etc.

While working in the Neonatal Intensive Care Unit (NICU), you are notified that a "small for gestational age" infant is being brought to the unit. Being a NICU nurse, you understand that this means which of the following? A. The infant's weight is less than 2500 grams. B. The infant's weight is below the 20th percentile. C. The infant's weight is less than 1500 grams. D. The infant's weight is below the 10th percentile. Submit Answer

Explanation Choice D is correct. The term "Small for Gestational Age (SGA)" is used when the infants are smaller than normal for the number of weeks of pregnancy (gestational age). When an infant's weight is below the 10th percentile for the gestational age, it is considered small for gestational age. By definition, about 10 percent of all newborns are labeled as SGA. Not all "Low Birth Weight" babies are SGA. Infants may be of low birth weight but may still fall above the 10th percentile for gestational age. It is important to distinguish SGA from other related terms, "Low Birth Weight (LBW)", "Very Low Birth Weight (VLBW), and "Extremely Low Birth Weight (ELBW)." These definitions are based on the infant's weight at the time of birth. These are not percentile scores and are defined on the absolute weight limit. An LBW infant is defined as an infant with a weight of less than 2500 grams (5 lb. and 8 ounces), regardless of gestational age at the time of birth. A VLBW infant is defined as one with a weight less than 1500 grams at the time of birth. An ELBW infant is less than 1000 grams at the time of birth. Choice A is incorrect. When infants are born at less than 2500 grams, they are considered low birth weight. A classification that considers only the weight and not the gestational age, is LBW, VLBW, and ELBW whereas percentiles are used for the small/average/large for gestational age comparison. Choice B is incorrect. The 20th percentile is considered average for gestational age. Infant's size falling between 10th - 90th percentile is considered average. One that is less than the 10th percentile is "small for gestational age", and greater than 90th percentile is "large for gestational age". Choice C is incorrect. When infants are born at less than 1500 grams, they are considered as "very low birth weight". Percentile scores are used for the small/average/large for gestational age comparison. NCSBN Client Need Topic: Health Promotion and Maintenance, Subtopic: Newborn Last Updated - 13, Dec 2021

The nurse is triaging phone calls at a local obstetrics clinic. Which client situation requires immediate follow-up? A client reporting A. a decrease in fetal movements of ten in the past hour. B. irregular, painful contractions that are decreased with repositioning. C. abdominal cramping following her amniocentesis six hours ago. D. epigastric pain and a frontal headache not relieved with acetaminophen. Submit Answer

Explanation Choice D is correct. These symptoms are strongly suggestive of severe pre-eclampsia. Severe pre-eclampsia manifests as epigastric to right-upper quadrant pain suggestive of a liver injury. This, combined with a frontal headache, is highly concerning for severe pre-eclampsia. The client needs to be immediately evaluated as these symptoms may worsen to an eclamptic seizure. Choice A, B, and C are incorrect. A decrease in fetal movements is only concerning if the fetal movements are less than ten in two consecutive hours. Irregular, painful contractions that are decreased with repositioning is false labor, also known as Braxton-hick contractions. This does not require follow-up; the client should be instructed to hydrate and rest. Abdominal cramping that occurs less than 24 hours from the amniocentesis is normal and does not require follow-up. Additional Info Source : Archer Review Preeclampsia is hypertension (systolic blood pressure ≥140 mm Hg or diastolic ≥90 mm Hg) occurring after 20 weeks of pregnancy in women with previously normal blood pressure, usually accompanied by proteinuria. The client with preeclampsia should be instructed to check her blood pressure and report symptoms that suggest worsening preeclampsia, such as visual disturbance, severe headache, or epigastric pain. Symptoms that suggest fetal compromise, such as reduced fetal movement, also should be taught. Last Updated - 10, Nov 2022

The nurse in the delivery room has just assisted in the delivery of a newborn and is now attempting to deliver the placenta. The nurse understands that expulsion of the placenta would trigger all of the following processes, except: A. Decrease in progesterone B. Decrease in estrogen C. Increase in prolactin D. Production of oxytocin Submit Answer

Explanation Choice D is correct. This is an incorrect statement, therefore the correct answer to the question. Oxytocin production is stimulated by suckling. Suckling stimulates the posterior pituitary gland to produce oxytocin, causing the release of milk from alveoli into the ducts. Choices A, B, and C are incorrect. This is a correct statement. When the placenta is expelled, this causes a decrease in estrogen and progesterone that causes the anterior pituitary gland to increase prolactin. Last Updated - 17, Jan 2022

You are seeing patients in an outpatient obstetrical clinic for their regularly scheduled prenatal appointments. Your patient is a G2P1 32-year-old woman who is 32 weeks pregnant. She says to you that she is concerned because she thinks she is developing striae gravidarum. When you assess the patient, what would you expect to see if she does present with this condition? Select the correct image. A . B . C . D .

Explanation Choice D is correct. This is the image showing striae gravidarum. These are reddish-purple stretch marks on the abdomen, breasts, thighs, and upper arms that are due to increasing levels of the melanocyte-stimulating hormone with the increase in estrogen and progesterone levels. Choice A is incorrect. This is an image showing chloasma. This is also known as the 'mask of pregnancy.' It is hyperpigmentation that appears blotchy and brown over the forehead, cheeks, and nose. Choice B is incorrect. This is an image showing linea nigra. This is a dark streak down the midline of the abdomen that sometimes occurs in pregnancy due to increasing levels of the melanocyte-stimulating hormone with the increase in estrogen and progesterone levels. Choice C is incorrect. This is an image showing the typical presentation of the gravid abdomen around 32 weeks gestation. NCSBN Client Need Topic: Health Promotion and Maintenance Subtopic: Ante/Intra/Postpartum and Newborn Care Last Updated - 11, Nov 2021

A mother in her first trimester of pregnancy is very upset that she feels constantly nauseous. You reassure her that the nausea is common in the first trimester. In addition, which of the following advice would you share with her? Select all that apply. A. Eat dry crackers before getting out of bed in the morning. B. Drink small sips of liquids throughout the day rather than large amounts with meals. C. Eat only three moderate size meals a day. D. Brush teeth immediately after eating to avoid smells and tastes that trigger nausea. Submit Answer

Explanation Choices A and B are correct. Eating dry crackers in the morning before rising is standard advice to help decrease nausea (Choice A). It is recommended to drink small sips of liquids throughout the day rather than drinking large amounts with meals (Choice B). Choice C is incorrect. It would be better to recommend eating small frequent meals spread throughout the day. Low-fat meals are also advised to decrease nausea. Choice D is incorrect. It is better to avoid brushing teeth immediately after a meal and shortly after getting out of bed in the morning to prevent nausea. NCSBN Client Need: Topic: Health Promotion and Maintenance, Subtopic: Antepartum Last Updated - 02, Nov 2021

While working in the neonatal intensive care nursery, you are assigned to take care of a baby who is 31 weeks gestation. Which of the following complications must you know to monitor given the baby's gestational age? Select all that apply. A. Hypoglycemia B. Hypothermia C. Birth injuries D. Fat wasting Submit Answer

Explanation Choices A and B are correct. Infants born before 37 weeks gestation have low stores of glucose and therefore hypoglycemia is a common complication of prematurity. Blood glucose should be monitored closely (Choice A). Preterm infants are at risk for poor thermoregulation and hypothermia due to decreased stores of muscle and fat. Their body temperatures should be regulated via incubator, radiant warming, bundling, or other methods of temperature control, as indicated (Choice B). Choice C is incorrect. Birth injuries are not a common complication for preterm infants as they are typically small and don't experience issues during vaginal delivery. This would be a complication to monitor for an infant that is large for gestational age. Choice D is incorrect. Fat and muscle wasting are not a common complication of preterm infants. They do not have large muscle and fat stores to begin with. This is common in a baby born post-term, who has wasted fat and muscle stores while in utero. NCSBN Client Need: Topic: Physiological Integrity, Subtopic: Basic care, comfort; Newborn Last Updated - 14, Jul 2021

The nurse knows that benefits of delayed cord clamping in the newborn include: Select all that apply. A. Increased blood volume B. Decreased brain hemorrhages C. Decreased risk of polycythemia D. Decreased jaundice Submit Answer

Explanation Choices A and B are correct. Potential benefits of delayed cord clamping in the newborn include increased blood volume, decreased risk of brain hemorrhages, increased blood pressure, lower risk of necrotizing enterocolitis, decreased anemia, and improved neurodevelopmental outcomes. Potential adverse effects of delayed cord clamping include an increased risk of polycythemia and jaundice and possible delay in resuscitation efforts for at-risk newborns. Current research indicates that cord clamping can be delayed for 30-60 seconds in vigorous newborns as long as the team manages the newborn's temperature to keep the infant warm. Choices C and D are incorrect. Delayed cord clamping does not decrease the risk of polycythemia or jaundice. NCSBN Client Need Topic: Reduction of Risk Potential, Sub-topic: Potential for Complications of Diagnostic Tests/Treatments/Procedures, Newborn Last Updated - 23, Dec 2021

Which of the following are bypasses in fetal circulation? Select all that apply. A. Ductus arteriosus B. Foramen ovale C. Ductus pulmonic D. Foramen aortic Submit Answer

Explanation Choices A and B are correct. The ductus arteriosus is a bypass in fetal circulation. It connects the pulmonary artery to the aorta (choice A). The foramen ovale is a bypass in fetal circulation. It is an opening between the right and left atriums of the heart (choice B). Choice C is incorrect. There is no ductus pulmonic; this is not a bypass in fetal circulation. Choice D is incorrect. There is no foramen aortic; this is not a bypass in fetal circulation. Last Updated - 20, Jan 2022

You are working in the newborn nursery taking care of a 2-day old infant with fetal alcohol spectrum disorder and preparing the family for discharge. Which of the following educational points are essential to include? Select all that apply. A. Regular therapy appointments will need to be scheduled. B. An individualized education plan should be formulated with the child's school when he is preparing for kindergarten. C. With proper therapy, the condition will improve. D. A regular infant diet should be followed. Submit Answer

Explanation Choices A and B are correct. Therapy will be incredibly important for this infant after discharge. Physical therapy, occupational therapy, and speech therapy should all be incorporated with this infant's care plan. They will keep track of milestones and help aid in the development, motor skills, and cognitive abilities of the infant. Parents should be educated about the importance of these therapies so that they take them seriously and keep up with their appointments (Choice A). This child will require special education when starting school. The parents should be educated about this need so that they are realistic about their culture and prepared for the future needs of the child. Individualized education plans will be accommodated through the school system; the therapists and health care providers of the child can help inform them (Choice B). Choice C is incorrect. Fetal alcohol spectrum disorder is a lifetime disability. There is no cure. Even with proper occupational therapy, physical therapy, and speech therapy, there are expected delays in the life of this infant. He or she will likely have difficulties with poor judgment, cognition, impulse control, memory, and learning for the rest of their life. Emphasizing the chronicity of this disease may help the mother refrain from consuming alcohol during any future pregnancies and will ensure she is educated about the needs her child will face in the future. Choice D is incorrect. Infants with fetal alcohol spectrum disorder face challenges, including weak growth. They are often of short stature, low weight, and have smaller heads than average. Because of this, their nutritional needs will be unique. A nutritionist should be consulted to work with the family before discharge and teach them about the proper formulas/diet plan for their infant to maximize growth. NCSBN Client Need: Topic: Health Promotion and Maintenance; Subtopic: Newborn Last Updated - 15, Jan 2022

Which of the following medications is contraindicated for a pregnant client? Select all that apply. A. Warfarin B. Finasteride C. Celecoxib D. Clonidine E. Transdermal Nicotine F. Clofazimine

Explanation Choices A and B are correct. Warfarin (coumadin) has a pregnancy category X. It is associated with central nervous system defects, spontaneous abortion, stillbirth, prematurity, illness, and ocular defects when given any time during pregnancy and a fetal warfarin syndrome when given during the first trimester (Choice A). Finasteride (Propecia, Proscar) also has a pregnancy category X, which has a high risk of causing permanent damage to the fetus (Choice B). Fetal age affects the type of drug effect: Before the 20th day after fertilization: Drugs that were given at this time typically have an all-or-nothing effect, killing the embryo or not affecting it at all. Teratogenesis is unlikely during this stage. During organogenesis (between 20 and 56 days after fertilization): Teratogenesis is most likely at this stage. Drugs reaching the embryo during this stage may result in spontaneous abortion, a sublethal gross anatomic defect (exact teratogenic effect), covert embryopathy (a permanent subtle metabolic or functional defect that may manifest later in life), or an increased risk of childhood cancer (e.g., when the mother is given radioactive iodine to treat thyroid cancer); or the drugs may have no measurable effect. After organogenesis (in the 2nd and 3rd trimesters): Teratogenesis is unlikely, but drugs may alter the growth and function of customarily formed fetal organs and tissues. As placental metabolism increases, doses must be higher for fetal toxicity to occur. Choice C is incorrect. Celecoxib (Celebrex) in large doses causes congenital disabilities in rabbits, but it is not known if the effect is the same on humans. Choice D is incorrect. Clonidine (Catapres) crosses the placenta, but no adverse fetal effects have been observed. Choice E is incorrect. Nicotine replacement products have been assigned to pregnancy category C (nicotine gum) and category D (transdermal patches, inhalers, and spray nicotine products). Choice F is incorrect. Clofazimine has been assigned to pregnancy category C. NCSBN Client Need Topic: Physiological Integrity, Subtopic: Pharmacological Therapies; The Effects of Medications on Fetal Development Last Updated - 15, Dec 2021

While reviewing fetal monitoring strips, the labor and delivery nurse notes that the reading is nonreassuring. What features characterize a fetal monitoring strip as nonreassuring? Select all that apply. A. Fetal heart rate less than 110 beats/minute. B. Increase in variability. C. Late decelerations D. Mild variable decelerations Submit Answer

Explanation Choices A and C are correct. A fetal heart rate less than 110 beats/minute or greater than 160 beats/minute is nonreassuring (Choice A). Late decelerations are an ominous sign, therefore, immediate interventions should be taken to improve the fetal heart rate; they are characteristic of a nonreassuring heart rate (Choice C). Choice B is incorrect. An increase in variability is a reassuring factor. A decrease in variability would be nonreassuring. Choice D is incorrect. Mild, variable decelerations are okay, only when the variable decelerations are severe are they nonreassuring. NCSBN Client Need Topic: Physiological AdaptationSubtopic: Alterations in Body Systems Last Updated - 16, Feb 2022

The nurse is observing a client who has been in labor for 16 hours. For which of the following observations, should she notify the healthcare provider? Select all that apply. A. FHR 170-200 for 20 minutes B. Early decelerations C. Variable decelerations D. Moderate variability Submit Answer

Explanation Choices A and C are correct. A fetal heart rate of 170-200 for 20 minutes is fetal tachycardia and should be reported to the healthcare provider. Any increase in fetal heart rate above 160 is considered tachycardia. When it persists for longer than 10 minutes, it is problematic and requires intervention (Choice A). Variable decelerations need to be reported to the healthcare provider immediately. They are sharp and profound drops in the fetal heart rate unrelated to the time of contractions are a non-reassuring sign on a fetal heart rate strip. Variable decelerations are caused by cord compression, such as a prolapsed cord, and are an emergency requiring quick nursing intervention (Choice C). Choice B is incorrect. Early decelerations do not need to be reported to the healthcare provider. They occur when the fetal heart rate decreases at the same time as a contraction and are followed by a return to baseline. They occur due to the pressure of the fetus's head on the pelvis or soft tissue and the nurse requires no intervention after an early deceleration. Choice D is incorrect. Moderate variability does not need to be reported to the healthcare provider. Variability on a fetal heart rate is defined as the fluctuations in the fetal heart rate from baseline. A moderate amount of variability is what is expected and is considered a reassuring sign. NCSBN Client Need: Topic: Physiological Integrity, Subtopic: Reduction of Risk Potential; Problems with Labor and Delivery Last Updated - 14, Feb 2022

Which of the following neurological assessments would be considered abnormal in a newborn? Select all that apply. A. High pitched cry B. Pupils are 2mm, equal, round, and react briskly to light. C. Lethargy D. Sleeping between each feeding Submit Answer

Explanation Choices A and C are correct. A high, pitched cry is an irregular finding in a newborn. It can be a sign of withdrawal in neonatal abstinence syndrome, or a sign of increased ICP if there is birth trauma (Choice A). For the level of consciousness, lethargy is not a normal finding. We expect the newborn to be alert. Lethargic, obtunded, stuporous, or comatose are all abnormal findings (Choice C). Choice B is incorrect. This is the usual pupil assessment. 2-3 mm, equal in size, round in shape, and briskly reactive to light are the expected findings for a pupil assessment in a newborn. Choice D is incorrect. For behavior, an infant is expected to be sleeping between their feedings. This is considered appropriate for the age and is a reasonable assessment finding. NCSBN Client Need: Topic: Physiological Integrity, Subtopic: Physiological adaptation, Newborn Last Updated - 07, Nov 2021

Which of the following are required for a nonstress test to be considered reactive? Select all that apply. A. Two increases in the fetal heart rate of 15 beats per minute. B. Two decreases in the fetal heart rate of 15 beats per minute. C. Two increases in the fetal heart rate for 15 seconds. D. Two decreases in the fetal heart rate for 15 seconds. Submit Answer

Explanation Choices A and C are correct. For a nonstress test to be reactive, there must be two accelerations. Acceleration is defined as an increase in fetal heart rate by 15 beats per minute for at least 15 seconds with movement (Choice A). For a nonstress test to be reactive, there must be two accelerations. Acceleration is defined as an increase in fetal heart rate by 15 beats per minute for at least 15 seconds with movement (Choice C). Choice B is incorrect. Any decrease in fetal heart rate is a deceleration, which is an indicator of fetal distress and a nonreassuring sign. Slowdowns would lead to a nonreactive nonstress test. Choice D is incorrect. Any decrease in fetal heart rate is a deceleration, which is an indicator of fetal distress and a nonreassuring sign. Slowdowns would lead to a nonreactive nonstress test. NCSBN Client Need: Topic: Health promotion and maintenance, Subtopic: Antepartum Additional Info Source : Archer Review Last Updated - 22, Nov 2022

The nurse assists a mother in labor to the bathroom and notes that the fetal heart rate increases from 130 to 190. She sits the mother back down in bed, and the fetal heart rate remains 190. Which of the following nursing actions would be appropriate? Select all that apply. A. Lie the mother down on her left side B. Decrease the rate of her IV fluids C. Administer oxygen D. Continue to just observe the mother Submit Answer

Explanation Choices A and C are correct. The nurse has noted fetal tachycardia. Any increase in fetal heart rate above 160 is considered tachycardia. When it persists for longer than 10 minutes, it is problematic and requires intervention. Any non-reassuring fetal heart rate will require intervention. One could remember these interventions with the mnemonic: LION: lie the mother on her left side, increase IV fluids, oxygen, and notify the healthcare provider. In this case, the non-reassuring sign of fetal tachycardia necessitates intervention, therefore lying the mother on her left side is an appropriate intervention. Administering oxygen is an appropriate nursing intervention for the noted fetal tachycardia. The idea is to improve fetal oxygenation. This will go along with repositioning the mother onto her left side, increasing the rate of IV fluid administration, and notifying the healthcare provider. Choice B is incorrect. Decreasing the rate of the mother's IV fluids is not appropriate. Instead, the nurse should increase the IV fluid rate to help better facilitate blood perfusion to the placenta and fetus. Choice D is incorrect. It is inappropriate to continue to observe the mother. The nurse has noted fetal tachycardia, a non-reassuring sign that requires intervention. The nurse should lay the mother on her left side, increase her IV fluids, administer oxygen, and notify the healthcare provider. NCSBN Client Need Topic: Effective, safe care environment; Subtopic: Coordinated care Last Updated - 29, Nov 2021

Your client comes to the clinic during the second trimester of her pregnancy. She is in the clinic for a "quad screen" and exam. In teaching her about the quad screen, you tell her that this procedure evaluates the chance of carrying a baby with: Select all that apply. A. Down syndrome B. Tay-Sachs disease C. Spina bifida D. Cystic fibrosis Submit Answer

Explanation Choices A and C are correct. The quad screen, or quadruple marker test, is done in the second trimester of pregnancy and includes measuring levels of AFP, HCG, estriol, and inhibin A. The clinician uses this test to evaluate the chance of carrying a baby with Down syndrome and spina bifida. These conditions can also be diagnosed when an ultrasound during the first trimester is not done or is not conclusive. As DNA screening improves, that diagnostic method might be used instead of the quad screen. Choices B and D are incorrect. The screen does not test for Tay-Sachs disease or cystic fibrosis. NCSBN Client Need Topic: Reduction of Risk Potential, Subtopic: Diagnostic Tests; Antepartum Last Updated - 24, Jun 2021

Which of the following are true regarding physiological changes during pregnancy? Select all that apply. A. Increase in heart size B. Increase in gastric motility C. Reduced renal threshold for glucose D. Decreased basal metabolic rate

Explanation Choices A and C are correct. There is an increase in heart size during pregnancy, as well as a heart position shift upward and to the left due to the displacement of the diaphragm as the uterus enlarges (Choice A). The renal threshold for glucose is reduced during pregnancy (Choice C). Choice B is incorrect. There is a decrease in gastric motility, which can sometimes cause poor appetite. Choice D is incorrect. There is an increased basal metabolic rate as metabolic function increases during pregnancy. NCSBN Client Need: Topic: Health Promotion and Maintenance, Subtopic: Antepartum Last Updated - 10, Feb 2022

Which of the following signs or symptoms would you expect in a postpartum patient experiencing subinvolution? Select all that apply. A. Boggy uterus B. Board-like abdomen C. Decreased fundal height D. Increased bleeding Submit Answer

Explanation Choices A and D are correct. A boggy uterus is a sign of subinvolution. Boggy, refers to a womb that is not firm and contracting as it is expected to in the postpartum stage. The uterus should contract to clamp down and prevent bleeding, but in sub-involution, it does not do so (Choice A). Increased bleeding is a sign of subinvolution. When the uterus is not contracting and clamping down on itself, it is not preventing postpartum bleeding. A healthy postpartum uterus would be contracting firmly, but this does not occur with subinvolution, so the mother is at risk for increased bleeding (Choice D). Choice B is incorrect. A board-like abdomen is not an expected finding with subinvolution. In subinvolution, the uterine muscle is not contracting as it should, therefore it is boggy and soft. A board-like abdomen is usually described as a stomach that has filled with blood due to an abruption or a traumatic accident. Choice C is incorrect. Fundal height would not be decreased in subinvolution. Instead, it would be increased. We expect the fundal height to decrease by 1 cm per day, but with subinvolution, the uterus is not returning to its standard pre-pregnancy size, so it stays at the same height. Choice D is correct. Increased bleeding is a sign of subinvolution. When the uterus is not contracting and clamping down on itself, it is not preventing postpartum bleeding. A healthy postpartum uterus would be contracting firmly, but this does not occur with subinvolution, so the mother is at risk for increased bleeding. NCSBN Client Need: Topic: Physiological Integrity, Subtopic: Physiological adaptation; Labor and Delivery Last Updated - 11, Jul 2021

You are called to the delivery of an infant that is 41 weeks gestation. And they suspect meconium in the amniotic fluid. After birth, which of the following signs would help you confirm a meconium delivery? Select all that apply. A. Brown-tinged amniotic fluid B. Thick, white substance coating the newborn C. Vigorous cry D. Brown discoloration of the infant's nails Submit Answer

Explanation Choices A and D are correct. If the amniotic fluid is tinged brown, it is a good indication that the meconium was passed before delivery (Choice A). Brown discoloration of the infant's nails, umbilical cord, or tongue can all indicate a meconium aspiration (Choice D). Choice B is incorrect. A thick, white substance coating the newborn is known as vernix caseosa. This is a potent substance and serves to moisturize the newborn's skin. Choice C is incorrect. A vigorous cry is a good sign in a newborn. This alone is not an indicator of meconium aspiration. If there is meconium in the fluid and then the infant starts to cry vigorously, it can then lead to meconium aspiration. NCSBN Client Need: Topic: Physiological Integrity, Subtopic: Basic care, comfort; Newborn Last Updated - 07, Nov 2021

You are providing education to a mother who gave birth three weeks ago. She has developed mastitis. Which of the following educational points are appropriate? Select all that apply. A. Continue to breastfeed your child normally. B. If unable to breastfeed, express milk every 2 hours. C. Do not take antibiotics. D. Wear a supportive bra without an underwire. Submit Answer

Explanation Choices A and D are correct. It is essential to educate mothers with mastitis that they should continue to breastfeed. The infection will not be passed to their child and they do not need to worry about any adverse effects for their infants. By continuing to breastfeed, the clogged milk ducts should become unclogged and mastitis should improve (Choice A). Wearing a supportive bra but one without an underwire is appropriate educational advice for a mother with mastitis. The support will help with the pain and tenderness in the breasts, but an underwire could cause clogged milk ducts, so it should be avoided (Choice D). Choice B is incorrect. While it is important to educate mothers that they should continue to breastfeed, they should only express milk every 4 hours if unable to breastfeed. Expressing milk every 2 hours is too frequent and could cause additional irritation. Choice C is incorrect. Mastitis is sometimes caused by an infection for which some healthcare providers will prescribe antibiotics. Advising mothers not to take medicines is not an appropriate education. NCSBN Client Need: Topic: Psychosocial Integrity, Subtopic: Pediatric Development Last Updated - 04, Jan 2022

The nurse is teaching a client who is pregnant about how to increase the fiber in her diet. It would be appropriate for the nurse to recommend which foods? Select all that apply. A. Bran muffin B. Kidney beans C. Brown rice D. Whole wheat pasta E. Corn F. French fries Submit Answer

Explanation Choices A, B, C, D, and E are correct. Fiber-dense foods are an excellent choice for individuals who are (and are not) pregnant. They sustain satiety and prevent spikes in blood glucose. Bran, vegetables, whole wheat, beans, and lentils are fiber-rich and would be appropriate to recommend to the client. Choice F is incorrect. French fries have very low nutritional value. They are rich in oil and sodium and often lead to fluid retention because of the high sodium level. Additional Info Common sources of dietary fiber include: Fruits and vegetables (with skins, when possible): Apples, strawberries, pears, carrots, corn, potatoes with skins, and broccoli Whole grains and whole grain products: Whole wheat bread, bran muffins, bran cereals, oatmeal, brown rice, and whole-wheat pasta Legumes: Peas, lentils, kidney beans, lima beans, baked beans, and nuts Last Updated - 09, Jul 2022

Which of the following integumentary assessments in the newborn are normal? Select all that apply. A. Lanugo B. Milia C. Mongolian spots D. Vernix caseosa Submit Answer

Explanation Choices A, B, C, and D are all correct. A is correct. Lanugo is fine, soft hair that covers the body and limbs. This is a common finding in newborns and is considered normal. B is correct. Milia are small white bumps typically found on the noses and cheeks of newborns. They are very common, considered normal, and usually go away on their own. C is correct. Mongolian spots are usual in newborns. They are a type of birthmark due to the extra pigment in certain parts of the skin. D is correct. Vernix caseosa is the "cheese-like" coating that covers the skin of a newborn immediately after birth. This is a normal finding and should not be removed from the baby until their first bath, as it provides moisture to their skin. NCSBN Client Need: Topic: Physiological Integrity, Subtopic: Physiological adaptation, Newborn Last Updated - 25, Sep 2021

While providing education to a group of expected mothers regarding the prevention of postpartum thrombophlebitis, you know they understand your teaching when they make which of the following statements? Select all that apply. A. "After we give birth, we are at an increased risk of clots for 6 to 8 weeks." B. "We shouldn't go on car rides longer than 4 hours for a few weeks after we give birth." C. "After delivery, we should get up and walk as soon as we are able to prevent clots from forming." D. "Trying not to cross our legs will help prevent clots from forming." Submit Answer

Explanation Choices A, B, C, and D are all correct. A is correct. Mothers are at an increased risk for clots for about 6 to 8 weeks after delivery. This is due to a natural increase in clotting factors in the body at this time. When there are increased clotting factors, clots form more readily. Therefore mothers are at risk for developing postpartum thrombophlebitis. B is correct. You should advise mothers not to go on car rides longer than 4 hours for a few weeks after they give birth. This is due to the increased amount of clotting factors present after birth, which puts them at higher risks for clots. Sitting still in a car for longer than 4 hours could be dangerous due to the likelihood of developing a clot. C is correct. This is excellent advice to share with expecting mothers. One of the essential ways to prevent postpartum thrombophlebitis is early ambulation. By encouraging them to get up and walk as soon as they are able, the likelihood of them developing clots will decrease. D is correct. One way to help prevent clots after delivery is by discouraging mothers from crossing their legs. When our legs are crossed for a prolonged period, increased pressure and immobility can lead to clot development. These mothers should be encouraged to ambulate as soon as they are able. NCSBN Client Need: Topic: Physiological Integrity, Subtopic: Physiological adaptation; Maternal and Newborn Health; Postpartum Last Updated - 10, Apr 2022

You are a nurse in the local childcare facility. You are feeding an infant from a bottle containing expressed breast milk from the mother, halfway through the feeding, you realize that the breastmilk you are supplying is not for this child. You have mistakenly picked up the breastmilk that was for another woman's child. You should: Select all that apply. A. Inform the parent of the child that you are feeding. B. Inform the mother of the child whose milk you fed to the child. C. Complete an incident report per facility policy. D. Inform the providers who are caring for the infants. Submit Answer

Explanation Choices A, B, C, and D are correct. All of these actions are appropriate and expected in this situation. Also, the team should assess both of the mothers for any infectious process. Additionally, the nurse should educate both sets of parents that the risk of transmission of the disease is small. The mother may have concerns about exposure to hepatitis B and C; however, these infections cannot be spread from a woman to an infant through breastmilk. Probably the most critical intervention is to put processes in place to prevent mix-ups of milk from happening again. NCSBN Client Need Topic: Safety and Infection Control, Sub-topic: Reporting of Incident, Newborn Last Updated - 12, Jan 2022

For a nonstress test to be considered reactive, several factors have to be present. Which of the following are components of a reactive nonstress test? Select all that apply. A. The test occurs over a 20-minute period B. There are 2 or more accelerations C. Accelerations are 15 beats/minute lasting 15 seconds D. Moderate variability is present Submit Answer

Explanation Choices A, B, and C are correct. A standard nonstress test occurs over 20 minutes. If the required two accelerations of 15 beats/minute over 15 seconds are not met in 20-minutes, the analysis is extended to 40 minutes (Choice A). There must be at least two accelerations in a 20-minute time frame for the nonstress test to be reactive (Choice B). The accelerations must be at least 15 beats/minute and last 15 seconds during the nonstress test for the test to be reactive (Choice C). Choice D is incorrect. Although moderate variability is a reassuring sign, variability is not a component of a nonstress test and is therefore not a part of it's reading. NCSBN Client Need: Topic: Health Promotion and Maintenance, Subtopic: Antepartum Additional Info Source : Archer Review Last Updated - 22, Nov 2022

The nurse is caring for a 30-year-old patient who has developed iron-deficiency anemia during pregnancy. Which complication would this patient be at an increased risk for due to iron deficiency anemia? Select all that apply. A. Low birth weight B. Preterm delivery C. Gestational diabetes D. Perinatal mortality Submit Answer

Explanation Choices A, B, and D are correct. During pregnancy, there is an increased demand for oxygen to supply both the mother and the developing fetus. Iron deficiency anemia occurs as a result of insufficient amounts of iron (needed to make hemoglobin) to meet oxygen demand. Iron deficiency anemia is associated with an increased risk for low birth weight, preterm delivery, and perinatal mortality. Choice C is incorrect. Iron deficiency anemia in pregnancy is not associated with an increased risk of developing gestational diabetes. NCSBN Client Need Topic: Maternal and newborn health, Subtopic: ante/Intra/postpartum care, illness management, pathophysiology Last Updated - 10, Feb 2021

The nurse is assessing a client with preeclampsia. Which clinical findings should the nurse anticipate? Select all that apply. A. Hyperreflexia B. Headache C. Uncontrolled vomiting D. Epigastric pain E. Glycosuria Submit Answer

Explanation Choices A, B, and D are correct. Hyperreflexia, headache, and epigastric pain are typical symptoms of preeclampsia. Preeclampsia is a pregnancy complication characterized by high blood pressure and signs of damage to another organ system, most often the liver and kidneys. Hyperreflexia is a common finding and may occur with ankle clonus. These findings arise because of neuromuscular irritability. Other findings associated with preeclampsia include hypertension, facial swelling, and proteinuria. Choice C is incorrect. Uncontrolled vomiting is the defining characteristic of hyperemesis gravidarum. Glycosuria is not specific to preeclampsia. This finding could be expected or concerning for diabetes mellitus. Proteinuria would be found in preeclampsia. Additional Info Source : Archer Review Last Updated - 27, Dec 2022

You are caring for a young woman who is pregnant for the first time. Common possible complications you should inform her about include: Select all that apply. A. Anemia B. Mood changes C. Hypotension D. Nausea and vomiting Submit Answer

Explanation Choices A, B, and D are correct. Possible complications during pregnancy include anemia, mood changes, and nausea/vomiting. Anemia is typically caused by dilution of red blood cells as blood volume increases. Depression usually occurs after birth and is often called postpartum depression or "baby blues." However, as hormones change during pregnancy, the mother-to-be can experience mood changes. Nausea and vomiting usually occur during the first trimester as a result of increasing levels of human chorionic gonadotropin (HCG). This "morning sickness" is thought to be a sign of a healthy pregnancy during the first three months, but when the vomiting is persistent and prolonged, it can result in hyperemesis gravidarum. This condition may require intervention to prevent weight loss and dehydration. Choice C is incorrect. Pregnancy often leads to hypertension rather than hypotension. NCSBN Client Need Topic: Physiological Adaptation, Subtopic: Alterations in Body Systems, Antepartum Last Updated - 18, May 2021

The nurse is planning a staff development conference about the causes of labor dystocia. It would be correct for the nurse to identify which of the following may cause a delayed progression during labor? Select all that apply. A. Magnesium sulfate infusion B. Oxytocin infusion C. Uterine overdistention D. Hypoglycemia E. Epidural analgesia Submit Answer

Explanation Choices A, C, D, and E are correct. Magnesium sulfate relaxes the uterus and may decrease the intensity of uterine contractions. A decrease in intensity will decrease the progression of labor. Often when magnesium sulfate is infused, oxytocin may be used in conjunction. Uterine overdistention is a cause of labor dystocia because when the uterus is stretched, it does not contract properly. Hypoglycemia is a cause of delayed labor progression because of the maternal fatigue it induces. While epidural analgesia provides effective pain control, the decrease in sensation will also decrease the woman's drive to push and interfere with the internal rotation mechanism. Choice B is incorrect. Oxytocin infusion would progress labor as it stimulates uterine contractions. This medication is often used to remedy labor dystocia. Additional Info The following factors may contribute to labor dystocia, which may reduce the progression of labor: Maternal fatigue Uterine overdistention such as with multiple gestation Maternal inactivity Uncontrolled maternal pain Fluid and electrolyte imbalance Hypoglycemia Excessive analgesia or anesthesia Murray, S., McKinney, E. (032018). Foundations of Maternal-Newborn and Women's Health Nursing, 7th Edition. Last Updated - 13, Oct 2022

The nurse is caring for a client experiencing prolonged labor with hypotonic contractions. Which of the following actions should the nurse take? Select all that apply. A. Encourage frequent voiding B. Maintain strict bedrest C. Prepare for a prescribed infusion of oxytocin D. Encourage frequent repositioning E. Prepare for an infusion of intravenous (IV) fluids

Explanation Choices A, C, D, and E, are correct. Prolonged labor with hypotonic contractions is classified as labor dystocia. Labor dystocia is a broad term that indicates that labor is not progressing. Key interventions for a client experiencing labor dystocia include encouraging the client to void frequently (when she feels the urge) because a full bladder will impede uterine contractions. A potential infusion of oxytocin to augment uterine contractions is a plausible prescription to be anticipated from the primary healthcare provider (PHCP). Frequent maternal repositioning is a key and noninvasive intervention that helps with fetal descent and effective contractions. The nurse should keep the client upright and encourage frequent repositioning. Fluid and electrolyte imbalances may be a cause of stunted labor. The nurse should be prepared to administer parenteral fluids because fluid and electrolyte abnormalities may cause labor dystocia. Choice B is incorrect. Strict bedrest is not an effective intervention considering her hypotonic labor. Dystocia can present in different forms, but a key intervention is frequent maternal repositioning, especially upright. This could be the client ambulating or standing upright under a warm shower. Additional Info The following factors may contribute to labor dystocia, which may reduce the progression of labor: Maternal fatigue Uterine overdistention such as with multiple gestation Maternal inactivity Uncontrolled maternal pain Fluid and electrolyte imbalance Hypoglycemia Excessive analgesia or anesthesia Murray, S., McKinney, E. (032018). Foundations of Maternal-Newborn and Women's Health Nursing, 7th Edition. Last Updated - 29, Dec 2022

The nurse is assessing her prenatal client for sexually transmitted infections by looking for risk factors. Which of the following are risks of acquiring an STI? Select all that apply. A. Low socioeconomic status B. A monogamous relationship C. A past history of working in the sex industry D. Illicit drug use E. History of cancer F. Previous history of STIs Submit Answer

Explanation Choices A, C, D, and F are correct. Low socioeconomic status, a history of being a sex worker, illicit drug use, and a previous history of sexually transmitted infections (STIs) are all risk factors for contracting STIs. Other factors include numerous sexual partners and being unmarried. Choices B and E are incorrect. A history of cancer and exclusive relationships are not examples of risk factors for acquiring an STI. NCSBN client need Topic: Health Promotion and Maintenance, High-Risk Behaviors Last Updated - 15, Feb 2022

Which of the following statements is true regarding fetal circulation? Select all that apply. A. There are high pressures in the fetal lungs causing decreased pulmonary circulation. B. Blood shunts from left to right in the fetal circulation. C. The ductus venosus allows freshly oxygenated blood to go to the fetal brain first. D. There are higher pressures in the right atrium in the fetal circulation. Submit Answer

Explanation Choices A, C, and D are correct. A is correct. In fetal circulation, the alveoli are filled with fluid. This causes high pressures in the fetal lungs, which shunts blood away from the pulmonary circulation. C is correct. The ductus venosus is a bypass in fetal circulation that shunts blood away from the weak fetal liver and to the brain. This allows the brain to get fresh oxygen first. D is correct. The pressures on the right side of the heart are higher in fetal circulation than on the left side of the heart. Choice B is incorrect. Blood shunts from right to left in the fetal circulation; this is due to increased pulmonary pressures caused by the fluid-filled alveoli. The high pulmonary pressures increase pressure on the right side of the heart, creating a gradient across the foramen ovale shunting blood from right to left. NCSBN Client Need: Topic: Physiological Integrity, Subtopic: Physiological adaptation, Newborn Last Updated - 07, Nov 2021

You are on the team in the delivery room caring for a newly born infant. After completing the initial assessment of the infant, you know that positive-pressure ventilation is indicated if which of the following is evident? Select all that apply. A. The infant is apneic. B. The bottom of the infant's feet are blue. C. The infant's heart rate is < 100 beats per minute. D. The infant is gasping. Submit Answer

Explanation Choices A, C, and D are correct. Apnea, gasping, or heart rate less than 100 bpm are clear indications that the team should begin positive pressure ventilation (PPV) within one minute after birth. The unit can also consider a trial of PPV if it cannot maintain oxygen saturation despite the use of oxygen or continuous positive airway pressure (CPAP). Choice B is incorrect. It is not unusual for some degree of cyanosis to be apparent following birth, so the fact that the bottoms of the infant's feet are blue would NOT indicate the need for PPV. NCSBN Client Need Topic: Reduction of Risk Potential, Sub-topic: Changes/Abnormalities in Vital Signs; Newborn Additional Info Last Updated - 08, Nov 2021

You are caring for a pregnant client who has an order to be on partial bed rest with bathroom privileges. You know that the side effects of this order can include: Select all that apply. A. Deep vein thrombosis B. Fetal demise C. Alterations in mood D. Undesirable weight gain Submit Answer

Explanation Choices A, C, and D are correct. Prolonged bed rest can result in deep vein thrombosis (particularly of the legs), alterations in mood due to stress and anxiety, and undesirable weight gain due to inactivity. Although bed rest is not ordered often, the nurse must understand that compression stockings and ankle exercises might be requested to prevent DVT. The client should have an opportunity to talk about their feelings related to the bedrest. The nurse should consult the nutritionist to work with the client and obstetrician to ensure a healthy diet that takes into account the decreased activity. Choice B is incorrect. Unless the fetus is stressed in other ways, bedrest by itself does NOT result in fetal demise. NCSBN Client Need Topic: Reduction of Risk Potential, Sub-topic: Potential for Alterations in Body Systems; Antepartum Last Updated - 05, Sep 2021

While caring for a newly pregnant mother, the nurse notes that she has a rubella infection. Which of the following conditions would the nurse be concerned about in this case? Select all that apply. A. Intrauterine growth restriction (IUGR) B. Hemolytic disease of the newborn C. Hydrocephaly D. Large for gestational age infant (LGA) E. Stillbirth Submit Answer

Explanation Choices A, C, and E are correct. Women infected with rubella are at an increased risk of having a miscarriage or a stillbirth. Their infants are more likely to suffer from intrauterine growth restriction and hydrocephaly. Choice B is incorrect. Hemolytic disease of the newborn is an alloimmune condition that occurs when the mother is Rh-negative and is pregnant with an Rh-positive baby. Choice D is incorrect. Women infected with rubella while pregnant are not at an increased risk for delivering an infant who is large for gestational age. Last Updated - 26, Jan 2022

Which of the following statements about reflexes in the newborn assessment are true? Select all that apply. A. The Babinski reflex is also known as the startle reflex. B. A positive Babinski sign is normal in the newborn. C. The Moro reflex is demonstrated when the infant is startled and stretches out their arms in response. D. The Moro reflex is pathologic in the newborn. Submit Answer

Explanation Choices B and C are correct. A positive Babinski sign is when the toes splaying outward after stroking the plantar surface of the foot. It is normal in the newborn but pathologic in the adult population (Choice B). When a baby is startled and responds by suddenly stretching out his arms, this is the Moro reflex (Choice C). Choice A is incorrect. The Moro reflex is also known as the startle reflex, not the Babinski reflex. Choice D is incorrect. The Moro reflex is standard in the newborn and is not pathologic. NCSBN Client Need: Topic: Physiological Integrity, Subtopic: Physiological adaptation, Newborn Last Updated - 07, Nov 2021

Which of the following signs and symptoms would you expect to see in an infant withdrawing from heroin? Select all that apply. A. Temperature 36.5 degrees Celsius B. Respiratory rate 88 C. Diaphoretic D. Constipation Submit Answer

Explanation Choices B and C are correct. A respiratory rate of 88 is tachypneic, which is what we would expect for an infant experiencing neonatal abstinence syndrome (NAS). Tachypnea is a common sign of NAS, as is respiratory distress. It is not uncommon to appreciate an increased work of breathing, including things such as nasal flaring, head bobbing, and retractions in these infants (Choice B). Diaphoresis or excessive sweating is a common symptom of infants in neonatal abstinence syndrome. Most of these infants are incredibly irritable, hot, and sweaty. It is similar to the withdrawal you would expect in an adult that goes cold turkey on a drug (Choice C). Choice A is incorrect. A temperature of 36.5 degrees Celsius is an average temperature, whereas an infant withdrawing from heroin would likely present with a fever. For NAS scoring, an illness is higher than 37.8 degrees Celsius. Most of these infants are incredibly irritable, hot, and sweaty. It is similar to the withdrawal you would expect in an adult that goes cold turkey on a drug. Choice D is incorrect. In an infant experiencing neonatal abstinence syndrome, you would expect to see very loose frequent stools, not constipation. These loose stools are so prevalent that many of these infants end up with horrible skin breakdown due to sitting in diapers filled with loose stool. It is essential to know that diarrhea is a common sign of NAS, so that you may monitor for these complications. NCSBN Client Need: Topic: Health Promotion and Maintenance; Subtopic: Newborn Last Updated - 10, Nov 2021

Which of the following are symptoms of true labor? Select all that apply. A. Contractions that dissipate with walking B. Contractions that come in regular intervals C. Lower back pain D. Contractions of consistent intensity Submit Answer

Explanation Choices B and C are correct. Contractions that come in regular intervals are a sign of actual labor. You should educate mothers to seek care for contractions that occur at regular intervals, become stronger, and closer together with time. These contractions will also not go away with a change in position or activity (Choice B). Lower back pain is a sign of actual labor. Both lower back pain and pelvic pain indicate real labor, so mothers should be educated to seek treatment when such symptoms present. 'False' labor, or Braxton hicks, do not present with these symptoms. They are much weaker than contractions and typically subside with a change in position or by going on a short walk (Choice C). Choice A is incorrect. Contractions that fade away with activity, a change in position, or rehydration are not a sign of actual labor. These are more likely Braxton Hicks contractions, which do not indicate labor. They are 'practice contractions' for the uterine muscle. Real contractions will not fade with activity; they come in regular intervals, become closer together, and get more intense as time goes on. Choice D is incorrect. Contractions of consistent intensity indicate Braxton hicks or 'false' labor. When a mother is in real labor, her contractions will increase in intensity over time. Education should be provided on the difference between Braxton hicks and actual labor so that mothers know when to call their OB and seek treatment. NCSBN Client Need: Topic: Physiological Integrity, Subtopic: Physiological adaptation; Labor and Delivery Last Updated - 24, Jan 2022

Which of the following conditions are considered risk factors for postpartum hemorrhage? Select all that apply. A. Microcephaly B. Dystocia C. Placenta previa D. Singleton pregnancy Submit Answer

Explanation Choices B and C are correct. Dystocia, which is prolonged and painful labor, is a risk factor for postpartum hemorrhage. Prolonged labor, specifically, can dramatically increase the risk of postpartum hemorrhage (Choice B). Placenta previa is a risk factor for postpartum hemorrhage. In placenta previa, the placenta is covering the cervix of the mother rather than sitting in the fundus of the uterus as it should be. This puts the mother at risk for postpartum hemorrhage (Choice C). Choice A is incorrect. Microcephaly is a newborn complication where the newborn is born with a head smaller than average. This is not a risk factor for a woman to experience postpartum hemorrhage. If you selected this answer, you might have gotten it confused with macrosomia, a condition where the infant is larger than average, specifically higher than 4,000 g. This is a risk factor for postpartum hemorrhage. Choice D is incorrect. A singleton pregnancy or a pregnancy with only one fetus does not pose a risk for postpartum hemorrhage. The risk factor for postpartum hemorrhage occurs with multiples, such as twins or triplets. NCSBN Client Need: Topic: Physiological Integrity, Subtopic: Physiological adaptation; Labor and Delivery Additional Info Source : Archer Review Last Updated - 10, Nov 2022

Which of the following obstetrical procedures can be used to assist the head of the fetus during vaginal delivery? Select all that apply. A. Amniotomy B. Forceps assisted delivery C. External version D. Vacuum assisted delivery

Explanation Choices B and D are correct. Forceps are tools used to help pull on the head of the baby to assist with the delivery. Vacuum-assisted delivery is a method where suction is applied to the head of the baby and pulled while the mother pushes. This helps to deliver the head of the infant. Choice A is incorrect. An amniotomy is the use of a hook or finger to break the amniotic sac. This helps to induce labor but does not assist in the delivery of the head of the fetus. Choice C is incorrect. The external version is a technique used when the baby is not in an appropriate position for vaginal delivery. The external cephalic version is used to turn a fetus from a breech position or side-lying (transverse) position into a more favorable head-down (vertex) position to help prepare the baby for a vaginal delivery. The external version is typically done before the labor begins, often around 37 weeks. Occasionally, it is done during labor but before the membranes have ruptured. If the amniotic sac has ruptured or if there is not enough amniotic fluid around the fetus (oligohydramnios), external version must not be done as it may end up injuring the fetus. External version does not directly assist in the delivery of the head of the fetus. NCSBN Client Need: Topic: Physiological Integrity Subtopic: Risk potential reduction Last Updated - 19, Dec 2021

A 25-year-old female client at ten weeks gestation has mild fatigue. While reassuring her that this is expected, the nurse also knows that which of the following are regular changes during various trimesters of a healthy pregnancy? Select all that apply. A. Thyroid gland decreases in size B. Maternal blood volume increases C. Intestinal mobility increases D. Diastolic blood pressure decreases Submit Answer

Explanation Choices B and D are correct. To provide adequate nutrition and gas exchange for the developing fetus, a woman's body undergoes several changes during pregnancy, including cardiovascular, hematologic, metabolic, renal, and respiratory changes. In a healthy pregnancy, maternal blood volume may increase by as much as 40 to 50% by week 32 of the pregnancy. Despite this increase in red blood cell production, the mother may develop dilutional physiological anemia. Mild to moderate fatigue may be experienced. During the second trimester, the nurse might note a decrease in diastolic blood pressure. Cardiac output may decrease as the mother changes positions. Choice A is incorrect. The thyroid and pituitary glands typically increase in size during pregnancy, not decrease. Reflecting the increased metabolic needs during pregnancy, TSH (thyroid-stimulating hormone) increases, and therefore, thyroid volume increases. Choice C is incorrect. Intestinal mobility decreases as progesterone levels increase to allow for increased absorption of nutrients. The nurse should be aware that this change may also increase the risk of constipation. NCSBN Client Need Topic: Health Promotion and Maintenance; Subtopic: Ante/Intra/Postpartum Care Last Updated - 11, Jan 2022

The nurse is teaching parents about antepartum testing. Which statements should the nurse include? Select all that apply. A. "Oral glucose tolerance testing will measure fetal activity at certain intervals." B. "A nonstress test may be used to measure fetal heart rate." C. "Amniocentesis may be used to assess if you have preeclampsia." D. "Chorionic villus sampling may be done to assess for neural tube defects." E. "You may need to fill up your bladder prior to an ultrasound." Submit Answer

Explanation Choices B and E are correct. A nonstress test is performed in the third trimester if the client has indications such as a high-risk pregnancy that may result in a stillbirth or complications such as fetal hypoxia. Ultrasounds typically require a full bladder as the fluid moves the uterus upward and assists with visualization. Choices A, C, and D are incorrect. Oral glucose tolerance testing is completed between 24-28 weeks of gestation. This test is used to determine if the client has gestational diabetes and does not take into account fetal activity. Amniocentesis is an antepartum test that may be used to determine the gender of the fetus, lung maturity, neural tube defects, or chromosomal abnormalities. Chorionic villus sampling is a test that may be performed as early as ten gestational weeks to determine if the fetus has any chromosomal abnormalities. Additional Info Amniocentesis is performed for a variety of indications and at different gestational ages. Common indications for amniocentesis include identifying chromosomal, metabolic, or genetic abnormalities. Amniocentesis can assist in determining fetal lung maturity (FLM) status. After this procedure, women should be instructed to report signs of bleeding, amniotic fluid that continues to leak after 24 hours, severe cramping that lasts several hours, or a temperature greater than 100.4°F Last Updated - 26, Jul 2022

Which of the following signs and symptoms are expected for your patient experiencing placental abruption? Select all that apply. A. Painless bleeding B. Dark red bleeding C. Hypotension D. Rigid abdomen Submit Answer

Explanation Choices B, C, and D are correct. B is correct. A massive amount of dark red bleeding is a prominent sign of placental abruption. This is due to the placenta separating from the wall of the uterus. This enormous amount of bleeding causes hypotension as the mother enters hypovolemic shock and fetal distress as perfusion to the baby decreases dramatically. C is correct. Due to the massive amounts of dark red bleeding, hypotension is a sign of placental abruption. When the mother loses large amounts of blood, her blood pressure will drop. This hypovolemia is treated with IV fluids and blood products such as PRBCs. D is correct. A rigid, board-like abdomen is a sign of placental abruption. This is also due to massive blood loss. As the placenta separates from the wall of the womb, blood starts to accumulate in the abdomen, causing it to become rigid, and board-like. Choice A is incorrect. Painless bleeding is NOT a sign of placental abruption. Instead, it is a sign of placenta previa. In placenta previa, the placenta is covering the cervix. This causes painless bleeding. In placental abruption, the placenta separates from the wall of the womb. This creates a massive amount of very painful dark red bleeding. It is important to remember the difference between these two emergencies. Placenta previa presents with painless bleeding, whereas placental abruption presents with painful bleeding. NCSBN Client Need: Topic: Physiological Integrity, Subtopic: Physiological adaptation; Labor and Delivery Last Updated - 08, Oct 2021

The nurse is caring for a pregnant client who is experiencing late decelerations. Which of the following actions should the nurse take? Select all that apply. A. Initiate intravenous magnesium sulfate B. Reposition the patient on her side C. Administer oxygen via face mask D. Discontinue oxytocin infusion E. Prepare for an amnioinfusion Submit Answer

Explanation Choices B, C, and D are correct. Late decelerations are a non-reassuring fetal heart pattern that requires immediate intervention. The nurse should place the client in a left lateral position, administer oxygen via face mask, and discontinue oxytocin. Repositioning the client will relieve the vena cava compression. Oxygen will correct any maternal hypoxia, and oxytocin should be stopped because uterine contractions decrease uteroplacental blood flow. Choices A and E are incorrect. Magnesium sulfate is indicated for clients with preeclampsia or eclampsia. It has no use for late decelerations. Amnioinfusion of an isotonic solution is utilized for variable decelerations - not late decelerations. Additional Info Source : Archer Review Late decelerations are primarily caused by uteroplacental insufficiency, and the client should be repositioned into a left lateral position. If the left lateral position is ineffective, then the nurse may consider using the right lateral position. The nurse should also consider prescribed intravenous (IV) fluids to restore maternal blood volume.

While participating in interdisciplinary rounds on the Mother-Baby floor, the provider mentions that your 2-day old patient is at risk for phenylketonuria (PKU). Which of the following statements are true regarding this condition? Select all that apply. A. It is a genetic disorder that is autosomal dominant. B. Children with phenylketonuria commonly have a musty odor to their urine C. Hypopigmentation of the hair, skin, and irises is a prominent sign of the disorder. D. All 50 states require routine screening of newborns for phenylketonuria. Submit Answer

Explanation Choices B, C, and D are correct. Phenylketonuria, or PKU, is a genetic disorder that results in central nervous system damage from toxic levels of the essential amino acid phenylalanine. The musty odor urine smell and hypopigmentation of the hair, skin, and irises are signs of PKU. It is also true that all 50 states require routine screening of newborns for this disorder. Choice A is incorrect. The disease is inherited in an autosomal recessive manner. NCSBN client need: Topic: Physiological Adaptation Subtopic: Alterations in Body Systems Last Updated - 01, Feb 2022

Which of the following would not be a normal change during late pregnancy? Select All That Apply. A. Waddling gait B. Sudden edema C. Vaginal bleeding D. Dark cloudy urine Submit Answer

Explanation Choices B, C, and D are correct. Sudden edema is abnormal and may indicate preeclampsia (Choice B). Vaginal bleeding (more than scant spotting) is never healthy in pregnancy before the start of labor (Choice C). Dark cloudy urine is abnormal and suggests infection or renal impairment (Choice D). Choice A is incorrect. Increased levels of relaxin loosen the cartilage between the pelvic bones, which results in the characteristic "waddling" walk of the third trimester. This is a healthy change during pregnancy. NCSBN Client Need Topic: Physiological Integrity, Subtopic: Reduction of Risk Potential; Pregnant Women Additional Info Source : Archer Review Last Updated - 10, Nov 2022

The nurse is caring for a client at 32 gestational weeks. Which laboratory data should be reported to the primary healthcare provider (PHCP)? Select all that apply. A. Hemoglobin 11.5 g/dL B. Platelets 90,000 mm3 C. Fasting blood glucose 254 mg/dL D. White blood cell 9,500 mm3 E. Creatinine 3.9 mg/dL

Explanation Choices B, C, and E are correct. These laboratory values are abnormal and require follow-up. A platelet count of fewer than 150,000 mm3 is concerning for thrombocytopenia and suggests severe preeclampsia. The blood glucose is significantly elevated as the normal fasting blood glucose is 70-100 mg/dL. This client has clinical hyperglycemia and requires follow-up. Finally, the creatinine is quite elevated, suggesting acute kidney injury. This, combined with a low platelet count, is more convincing of severe preeclampsia. Choices A and D are incorrect. These findings are within normal limits. For a client who is pregnant, the normal hemoglobin level may decrease to 11.5 g/dL without any intervention. Finally, this white blood cell count is normal (the normal range is 5000 - 10000 mm3). Additional Info The following are the clinical criteria for severe preeclampsia If one or more of the following criteria are present: 1. Blood pressure of ≥160 mm Hg systolic or ≥110 mm Hg diastolic or higher on two occasions at least 6 hr apart while the patient is on bed rest 2. Oliguria of <500 mL in 24 hr 3. Cerebral or visual disturbances 4. Pulmonary edema or cyanosis 5. Epigastric or right upper quadrant pain 6. Impaired liver function as indicated by abnormally elevated blood concentrations of liver enzymes (to twice normal concentration), severe persistent right upper quadrant or epigastric pain unresponsive to medication and not accounted for by alternative diagnoses, or both 7. Thrombocytopenia 8. Renal insufficiency Last Updated - 03, Nov 2022

You are educating a 25-year-old obese client with a body mass index (BMI) of 31 at 12 weeks gestation, who presents for a routine antenatal check-up. She gained 3 pounds compared to pre-pregnancy weight. Which of the following statements by the client reflect correct understanding regarding recommended weight changes in pregnancy? Select all that apply. A. "Since I am obese, I should try to lose weight now to limit my risk of gestational diabetes." B. "Typically, there is 3 to 6 pounds of weight gain during the first trimester of pregnancy." C. "In the third trimester, a weight gain of 2 pounds or more each week is considered high." D. "I should aim to gain a total of 25 to 35 pounds during this pregnancy." E. "Going forward in my pregnancy, I should aim to gain ½ pound per week." Submit Answer

Explanation Choices B, C, and E are correct. Weight gain is considered crucial during pregnancy. A pregnant woman should be educated regarding what is deemed to be reasonable in terms of pregnancy weight gain and the implications of gaining too much or too little weight. The client needs to keep track of the rate of weekly weight gain. Guidelines have been proposed to assist with determining the rate of healthy weekly weight gain. Weight gain of 3 to 6 pounds during the entire first trimester (first three months) is considered normal and healthy (Choice B). Gaining 2 pounds or more per week at any time (Choice C) during pregnancy would be abnormally high and such a client should focus on limiting the further rate of weight gain. The client in the question has already gained 3 pounds, which is healthy. Going forward, she should aim to gain about 8 to 17 pounds in the next six months (about half a pound per week for the rest of her pregnancy). This is based on the recommended weight gain of 11-20 pounds during the entire pregnancy for someone with a BMI of 30 or above (obese). Recommended weight gain is based on pre-pregnancy BMI and is shown in the table below: Choices A and D are incorrect. Weight-loss is dangerous during pregnancy. Regardless of their pre-pregnancy weight, every woman is expected to gain weight during pregnancy. The amount of recommended weight gain, however, is based on their pre-pregnancy BMI. A weight gain of 25 to 35 pounds (Choice D) is an ideal range recommended for those clients with healthy pre-pregnancy BMI (18.5 to 24.9). For an obese client, gaining 11 to 20 pounds during the entire pregnancy is considered ideal. Gaining more than the recommended weight will put the clients at risk for maternal hyperglycemia, reduced glucose tolerance, and increased risk of fetal complications. Fetal complications include increased risk of preterm delivery, having a newborn who is large for gestational age (LGA), and requiring a cesarean delivery. Gaining less than recommended weight increases the risk for small for gestational age (SGA) babies. The following table gives recommendations for a healthy rate of weekly weight gain: Last Updated - 13, Nov 2021

The nurse is caring for a client scheduled for an amniocentesis. Which of the following statements would require follow-up? Select all that apply. A. "This test may tell me the gender of my baby." B. "I will receive intravenous (IV) sedation for this test." C. "I may have cramping after this procedure." D. "I may be given a medicine to stimulate contractions." E. "The results will tell me how my baby will handle labor." Submit Answer

Explanation Choices B, D, and E are correct. These statements are false and require follow-up. Amniocentesis is an ultrasound-guided test used in the detection of fetal abnormalities. Under ultrasound guidance, a thin needle is inserted into the amniotic sac to remove a sample of amniotic fluid. The fluid is then sent for analysis. Amniocentesis does not require intravenous sedation (choice B). The client will not be given medication to stimulate contractions for the amniocentesis procedure (choice D). Ureterotonic drugs (oxytocin challenge) are administered in a contraction stress test (CST), not amniocentesis. Once again, a CST will determine how the fetus will handle labor, not an amniocentesis (choice E). These statements require follow-up counseling and education to correct the client's understanding. Choices A and C are incorrect. These statements reflect correct understanding and do not require follow-up. The gender of the fetus can indeed be determined based on the results of amniocentesis (choice A). This procedure can be performed as early as fourteen weeks. The client may also expect cramping after this procedure, as this is the most common complaint (choice C). Learning Objective Understand that the amniocentesis procedure is indicated in a high-risk pregnancy to detect chromosomal abnormalities and neural tube defects. Additionally, it helps in gender identification. Additional Info An amniocentesis may be conducted on patients who have a high-risk pregnancy. This procedure can be performed as early as fourteen weeks and is often used to detect fetal neural tube defects and chromosomal abnormalities. Amniocentesis can also be used later in pregnancy to determine fetal lung maturity. The client will need to consent before the procedure, and if the client is Rh-negative, she will need to receive prescribed RhoGAM to prevent Rh-isoimmunization. Complications such as infection or bleeding may occur. The client should be instructed to report post-procedure fever, leaking of amniotic fluid, or bleeding. Last Updated - 25, Apr 2022

Which of the following antepartum test results indicate a need to further follow up? Select all that apply. A. Contraction stress test - negative B. Nonstress test - reactive C. Contraction stress test - positive D. Nonstress test - nonreactive Submit Answer

Explanation Choices C and D are correct. A positive contraction stress test means the baby had decelerations in response to contractions and therefore, may not tolerate labor. Therefore, follow-up is needed (Choice C). A nonreactive nonstress test means that the baby did not have two or more 15 by 15 accelerations during the 20 minute test period and is not responding appropriately to movement. Follow-up would be needed for this test result, most likely with a contraction stress test (Choice D). Choice A is incorrect. In a contraction stress test, the result we want is negative. This means that the baby did not have decelerations in response to contractions. Follow up would not be needed for this test result. Choice B is incorrect. In a non-stress test, the result we want is reactive. This means that the baby had two or more 15 by 15 accelerations during the 20 minute test period and is responding appropriately to movement. Follow up would not be needed for this test result. NCSBN Client Need: Topic: Physiological Integrity, Subtopic: Physiological adaptation, Newborn Additional Info Source : Archer Review Last Updated - 22, Nov 2022

Which of the following statements is true regarding the premature rupture of membranes (PROM)? Select all that apply. A. PROM is when the membranes rupture before 37 weeks gestation. B. In a normal delivery, membranes are expected to rupture before labor begins. C. A priority nursing intervention with PROM is to monitor for infection. D. When observing the fluid after the rupture of membranes, it should be clear and without odor. Submit Answer

Explanation Choices C and D are correct. A priority nursing intervention with PROM is to monitor for infection. When the membranes are ruptured before labor begins, the baby is then exposed to bacteria and pathogens of the outside world. These germs can enter the birth canal and infect both the mother and the infant. One of the most critical observations you must make is of the color, odor, consistency, and amount of the amniotic fluid when the rupture of membranes occurs. Any discolored or malodorous fluid may indicate an infection. After the breakdown of membranes occurs, the nurse should monitor the mother's temperature, WBC count, CRP, and other markers of disease (Choice C). It is essential to assess the color, odor, consistency, and amount of fluid when the rupture of membranes occurs. If the liquid is green or yellow and rancid, it is indicative of infection. If the fluid is brown or black, it is indicative of meconium passing in utero. The expected finding of amniotic fluid is a clear fluid with no odor (Choice D). Choice A is incorrect. PROM stands for Premature Rupture of Membranes. This is defined as the rupture of membranes (or "water breaking") before labor begins. This term is not related to what gestation the membranes rupture. If the membranes rupture before 37 weeks gestation, the correct terminology is PPROM. This stands for Preterm Premature Rupture of Membranes. The Preterm part of this acronym is what refers to the membranes rupturing before 37 weeks gestation. Choice B is incorrect. It is not expected that the membranes will rupture before the labor begins. In a standard delivery, there is a rupture of membranes after the mother has already started having regular contractions, dilating, and effacing. When the layers do separate before labor has begun, it is called PROM, or premature rupture of membranes. NCSBN Client Need: Topic: Physiological Integrity, Subtopic: Physiological adaptation; Labor and Delivery Last Updated - 10, Feb 2022

You are discussing the signs and symptoms of postpartum thrombophlebitis with an expectant mother. You should tell her to monitor for which of the following symptoms? Select all that apply. A. +2 pulses B. Bilateral redness C. Edema in one leg D. Tenderness Submit Answer

Explanation Choices C and D are correct. It is right to advise the mother to monitor for edema in one leg as a sign of postpartum thrombophlebitis. There will be edema, pain, redness, and tenderness in whichever leg the clot is occluding (Choice C). It is right to advise the mother to monitor for tenderness in one leg as a sign of postpartum thrombophlebitis. There will be edema, pain, and redness in whichever leg the clot is occluding (Choice D). Choice A is incorrect. +2 pulses are healthy, intense pulses. In postpartum thrombophlebitis, you would expect to see weak +1 pulses in the affected leg. This is due to decreased blood flow into the leg where the clot is preventing blood flow. Choice B is incorrect. You would expect to see unilateral redness in a patient with postpartum thrombophlebitis, so you would not advise this mother to monitor for bilateral redness. The edema, pain, redness, and tenderness are all unilateral, relating to the clot in one leg. NCSBN Client Need: Topic: Physiological Integrity, Subtopic: Physiological adaptation; Labor and Delivery Last Updated - 12, Feb 2022

Which of the following signs are indicative of respiratory distress in the newborn? Select all that apply. A. Respiratory rate of 48 breaths per minute B. Mild subcostal retractions C. Nasal flaring D. Head bobbing

Explanation Choices C and D are correct. Nasal flaring is a sign of respiratory distress. If the newborn is working hard to breathe, they use extra effort when trying to pull air in through their nose and their nares flare out with inhalation. This is a sign that they are struggling to breathe and indicates respiratory distress (Choice C). Head bobbing is a severe sign of respiratory distress in the newborn. As they work harder and harder to breathe, they start using the muscles in their neck to pull their head forward with each inhalation. This is a sign that they are struggling to breathe and indicates respiratory distress (Choice D). Choice A is incorrect. A respiratory rate of 20 to 60 breaths per minute is standard in the newborn. Choice B is incorrect. Mild subcostal retractions are not considered a normal finding in the newborn, however, they are not enough to indicate respiratory distress. Since newborns breathe using their abdominal muscles, it is common to see some mild subcostal retractions, especially when they cry. Moderate to severe withdrawals would indicate respiratory distress. NCSBN Client Need: Topic: Physiological Integrity, Subtopic: Physiological adaptation, Newborn Last Updated - 31, May 2021

You are providing education to an HIV+ mother about what she will need to do for her baby after he is born. Which of the following teaching points are appropriate? Select all that apply. A. Follow exclusive breastfeeding to ensure your infant receives proper nutrition. B. Ensure your infant receives all vaccines on time. C. Monitor your infant closely for signs of HIV for at least 18 months. D. Keep track of your baby's weight gain and notify the pediatrician if he doesn't gain weight as expected. Submit Answer

Explanation Choices C and D are correct. This is a critical education point for HIV+ mothers. Infants born to an HIV+ mother are usually asymptomatic at birth and do not begin demonstrating signs of immunodeficiency until 2-3 months of life. The mother's antibodies persist for 18 months, so continuing to watch the child closely will be very important. If they start to exhibit signs such as persistent diarrhea, poor feeding, or weight loss, they should see their pediatrician immediately (Choice C). This is a critical education point for HIV+ mothers. Weight loss is a classic sign of HIV and may not start to show until 2-3 months of life. Even if the baby is not actually losing weight, they may be gaining it much more slowly than expected. This will cause them to fall off the growth chart as they are not keeping up with their peers. Parents should be taught to monitor for this very carefully and have a close relationship with their pediatrician to track the child's growth and weight gain (Choice D). Choice A is incorrect. This is not the current WHO recommendation. Since HIV can be transmitted in breast milk, HIV+ mothers need to use formula instead of breastfeeding their babies. This may be difficult for mothers who want to breastfeed their baby, so review the WHO recommendations with them and explain how their antibodies passing through the breastmilk can transmit HIV to the infant. Also, provide education about formula, proper mixing method for calories, and safe storage of formula. Choice B is incorrect. Usually, this would be excellent advice for a new mother, but in the case of an HIV+ mother, it is not correct. No live vaccinations should be administered to an infant until it is confirmed they are HIV negative. This includes MMR (measles, mumps, rubella), smallpox, chickenpox, rotavirus, and yellow fever vaccines. All other vaccines can be administered on schedule. NCSBN Client Need: Topic: Health Promotion and Maintenance; Subtopic: Newborn Last Updated - 20, Sep 2021

The nurse is caring for a 1-day old newborn client diagnosed with jaundice. Which of the following statements is true regarding jaundice in newborns? Select all that apply. A. Jaundice within the first 24 hours of life is physiologic. [17%] B. Unconjugated bilirubin is excreted in the stool. [14%] C. Assessing a newborn for jaundice involves inspection of the skin, sclera, and mucous membranes. [34%] D. When treating a jaundiced infant with phototherapy, an important nursing consideration is to ensure their eyes and genitals are covered.

Explanation Choices C and D are correct. When assessing an infant suspected to have jaundice, the most important thing to do will be to evaluate the skin, sclera, and mucous membranes (Choice C). When bilirubin levels are high, there will be a yellow tinge to these areas due to the high levels of the bilirubin pigment in the blood. Jaundice usually starts in the face and forehead area, so the nurse should begin her assessment looking there. The sclera and mucous membranes are a common location to appreciate the yellow discoloration, especially in a patient with darker skin. When treating a jaundiced infant with phototherapy, important nursing considerations are to ensure their eyes and genitals are covered (Choice D). Phototherapy helps reduce serum bilirubin levels by converting bilirubin into water-soluble isomers that can be eliminated without conjugation in the liver. Phototherapy light can be harmful to the infant's eyes and genitals. Nurses should ensure these areas are covered with an eye mask and a diaper. Choice A is incorrect. Jaundice, within the first 24 hours of life, is pathologic. This means that there is some other disease process or condition, causing jaundice that needs to be investigated. Often, etiology includes ABO incompatibility leading to hemolysis, or liver pathology. Physiologic jaundice is noted 2-3 days after birth and is simply due to the normal process of the infant's liver taking over the processing of bilirubin. Since the liver is not mature enough to conjugate and excrete the bilirubin in the bile, predominantly unconjugated physiological jaundice appears. This type of disease should not be of any concern. Choice B is incorrect. It is the conjugated bilirubin that gets excreted in the stool, not unconjugated. Unconjugated bilirubin is the waste product that is released when the heme is released from hemoglobin in the process of red blood cell breakdown. It is transported to the liver to be converted into conjugated bilirubin. When converted to the conjugated form, it can be excreted in bile and reaches the small intestine. Small bowel bacteria convert this conjugated form to stercobilinogen and urobilinogen. Stercobilinogen gives the yellow color to feces. Urobilinogen is water-soluble and is detected in the urine. In obstructive jaundice, conjugated bilirubin does not reach the intestine, which results in pale stools. In newborns with hemolysis or liver pathology, the unconjugated form overwhelms the immature liver's capacity to conjugate. Hence, pathological jaundice ensues within 24 hours in such circumstances. NCSBN Client Need: Topic: Physiological Integrity; Subtopic: Basic care, comfort Last Updated - 10, Nov 2021

The nurse is educating a pregnant client who is admitted with deep vein thrombosis in her left lower extremity. The client is at 24 weeks of gestation. The client is placed on Low Molecular Weight Heparin (LMWH). Which of the following statements by the client indicate that she understands the education regarding LMWH? Select all that apply. A. "My blood thinner will be stopped soon after delivery." B. "Since I am on LMWH, I must have a planned cesarean section." C. "I hate injections. I will likely switch to warfarin after delivery." D. "There is an increased risk of fractures with long term LMWH therapy." E. "If I notice blisters or black-red areas at the injection site, then I will hold LMWH and immediately contact the doctor." F. "If I decide to switch to warfarin after delivery, then I should not breastfeed." Submit Answer

Explanation Choices C, D, and E are correct. LMWH is the anticoagulant of choice during pregnancy because it does not cross the placenta, but one may switch to warfarin in the post-partum period. While warfarin is contraindicated during active pregnancy due to its potential to cause congenital fetal disabilities, maternal/fetal bleeding, and miscarriages, it is considered safe in the post-partum period. Warfarin is also safe in lactating/breastfeeding women. The client will need a minimum of 3 to 6 months of anticoagulation and therefore will be requiring anticoagulation for a few weeks even after delivery. Additionally, the risk of recurrent venous thromboses is high up to 6 weeks post-partum. Since the client does not like injections, oral anticoagulation with warfarin is an option for her after delivery (Choice C). Long-term treatment with LMWH may decrease bone mineral density (osteopenia, osteoporosis) and increase the risk of fractures (Choice D). For those with pre-existing osteoporosis and fracture risk, close monitoring is needed when on long-term LMWH therapy. It is common to have some bruising and swelling at the injection site. However, the presence of blisters and necrotic areas (blackish-red central portion) at the injection site may serve as a warning sign of a complication known as heparin-induced-thrombocytopenia (HIT). If the patient or the nurse notes necrotic areas at the injection site, the physician must be notified immediately and LMWH must be discontinued (Choice E). HIT is a rare but dangerous complication with unfractionated and low-molecular-weight heparins. The body forms antibodies to heparin that may cross-react with platelet antigens and cause a drop in the platelets. Despite a decline in the platelet count, HIT is associated with the formation of thromboses (deep vein thrombosis and pulmonary embolism) because HIT antibodies disrupt and activate the clotting system. If HIT is confirmed, heparin should never be re-initiated. Other anticoagulants, such as argotraban or fondaparinaux, are used. Choice A is incorrect. Anticoagulation will not be stopped soon after delivery. The client will need a minimum of 3 to 6 months of anticoagulation and therefore will be requiring anticoagulation for a few weeks even after delivery. Anticoagulation in a client with a history of venous thromboses should never be discontinued immediately after birth because there is a high risk of recurrent venous thromboses at least up to 6 weeks post-partum. Choice B is incorrect. It is not mandatory that the patient have a cesarean section. Normal vaginal delivery is still possible with adequate planning. If the patient is in labor, she should hold LMWH and notify the physician immediately. LMWH should be held at least 24 hours before the planned delivery. If the client ended up receiving LMWH within 24 hours of delivery, she should not be given an epidural or spinal anesthesia. Choice F is incorrect. Warfarin is safe in lactating women as it does not pass into breast milk to any measurable level. The client should be encouraged to breastfeed. Both LMWH and warfarin are safe to take when breastfeeding. NCSBN Client Need: Topic: Pharmacological therapies; Subtopic: Adverse Effects/Contraindications/Side Effects/Interactions Last Updated - 15, Nov 2021

The nurse is performing an assessment on a term newborn four hours after delivery. Which assessment findings require follow-up? Select all that apply. A. Head circumference of 34 cm B. Chest is 2 cm smaller than the head C. Vernix caseosa in the skin folds D. Positive Babinski reflex E. Asymmetrical gluteal folds F. Jaundice noted in the head Submit Answer

Explanation Choices E and F are correct. Jaundice may be classified as pathologic or physiologic. Jaundice with an onset of less than 24-hours is pathologic and concerning as this may indicate hemolysis. Asymmetrical gluteal folds are not an expected finding because this suggests developmental dysplasia of the hip. Choices A, B, C, and D are incorrect. These assessment findings are within normal limits and do not require follow-up. A head circumference of 34 cm is normal. The normal head circumference of a term newborn is 32 to 38 cm. It is expected for the chest to be 2 to 3 cm smaller than the head. Vernix caseosa in the skin folds is a normal finding. This biofilm is usually washed away after birth. A Babinski reflex that is positive is an expected finding. This reflex may be elicited by stroking the lateral sole of the foot from the heel to across the base of the toes. This reflex disappears in eight to nine months. Additional Info When measuring the head circumference of a newborn, the nurse should measure around the fullest part of the head, with the tape placed around the occiput and just above the eyebrows. When measuring the chest circumference of a newborn, the nurse should measure the chest at the level of the nipples. The normal head circumference for a term newborn is 32 to 38 cm. The normal chest circumference for a term newborn is 30 to 36 cm. It is normal for the chest to be 2 to 3 cm smaller than the head. If molding of the head is present, the head and chest measurements may be equal at birth. Last Updated - 04, May 2022

At the time of birth, the nurse should accomplish the following tasks: Assess the newborn's heart rate Provide positive pressure ventilation Assess muscle tone and presence of respiratory effort Clear secretions as needed. The correct sequence for these tasks is: Assess the newborn's heart rate. Clear secretions as needed. Provide positive pressure ventilation. Assess muscle tone and presence of respiratory effort. Submit Answer

Explanation Correct answer: According to the American Heart Association and the American Academy of Pediatrics Neonatal Resuscitation Program algorithm, the team should first assess the newborn's muscle tone and breathing. If those are abnormal, the team should provide a patent airway, including positioning and suctioning as needed. At the same time, the team should ensure that the infant is warm. The third task is to assess the newborn's heart rate to ensure that it is at least 100 beats per minute. The fourth task in this sequence is to provide positive pressure ventilation if the heart rate is less than 100 bpm. NCSBN Client Need Topic: Health Promotion and Maintenance, Sub-Topic: Newborn Care Last Updated - 07, Nov 2021

The nurse is assessing a client who gave birth to twins at 31 weeks of gestation and to a stillborn infant at 23 weeks of gestation. One year later, she had a spontaneous abortion at 12 weeks of gestation. She is currently pregnant at 25 weeks of gestation. It would be correct for the nurse to document this client's GTPAL as A. G4-T0-P2-A1-L2 B. G4-T0-P1-A2-L2 C. G4-T0-P1-A2-L3 D. G4-T1-P1-A1-L2 Submit Answer

Explanation G = 4, T = 0 (no pregnancies went to term), P = 2 (the twins count as 1 and the stillborn infant counts as 1 pregnancy ending in preterm birth), A = 1 (the 12 weeks spontaneous abortion), L = 2 (the two living children) Additional Info A method for calculating gravida and para is to separate pregnancies and their outcome using the acronym GTPAL: G = gravida, T = term, P = preterm, A = abortions, and L = living children. G = pregnancies or gravida, T = term pregnancies delivered, P = preterm pregnancies delivered, A = abortions (spontaneous and induced), and L = living children Term pregnancies are any pregnancy 37 weeks or greater; preterm is any pregnancy 20-36 weeks; abortions are any abortions spontaneous or induced prior to 20 weeks. Last Updated - 08, May 2022

While assessing your patient in active labor, you evaluate the fetal monitor and note late decelerations and significantly decreased variability. The patient is on a Pitocin infusion. Upon observing this nonreassuring fetal heart rate, the nurse should take the following actions in what order? Administer oxygen Stop the Pitocin infusion Identify the cause Prepare for delivery if unresolved. Change the mother's position Submit Answer

Explanation The first action the nurse should take is to identify the cause of the nonreassuring fetal heart rate. Is the patient lying on her back? Is the Pitocin drip inappropriately titrated? There are many causes of nonreassuring fetal heart rates, and sometimes a simple intervention can fix the problem. After the nurse tries to identify the cause, she should stop the Pitocin infusion. Remove the potential cause or contributing factor first. Even if the fetal heart rate seems to be improving, the Pitocin infusion should be stopped to prevent any further decelerations or decreased variability. Next, the nurse should change the mother's position and lay the mother in the left lateral position. If she is on her back, the fetus could be putting pressure on her descending aorta, or the fetus could be compressing the umbilical cord, and a simple change of position will resolve this. Next, the nurse should begin administering oxygen to the mother via a simple face mask at 8-10 L/min. This will optimize oxygenation to the fetus. Lastly, if still unresolved, the nurse needs to prepare for an emergency delivery. NCSBN Client Need Topic: Physiological Adaptation Subtopic: Medical Emergencies Last Updated - 13, Nov 2019

The nurse is caring for a client experiencing variable decelerations. The nurse observes the umbilical cord protruding through the vagina. Place the priority actions in the correct order. Stay with the client and call for help Apply pressure to lift the presenting fetal part Place the client in Trendelenburg position Prepare for delivery Administer oxygen Submit Answer

Explanation The priority nursing action is to apply pressure to the presenting fetal part. This will lift the fetus off the prolapsed umbilical cord and restore blood flow to the fetus. The nurse cannot let go until the health care provider arrives to deliver the fetus. Next, the nurse should stay with the client and call for help. This is a medical emergency, and the nurse must remain with the client to ensure safety. Next, the nurse needs someone to place the client in Trendelenburg's or knees to chest position. This will assist with keeping the presenting fetal part off the umbilical cord so that blood flow to the fetus continues. Because the nurse must remain with their hand applying pressure to the presenting fetal part, additional staff are needed to help with this. Next, the nurse needs someone to administer oxygen to the mother via a simple face mask at 8-10 L/min. This will optimize oxygenation to the fetus. Again, additional staff is needed to help with this action. Lastly, the nurse needs to prepare for the immediate delivery of the fetus. This is the only way to resolve this medical emergency. Additional Info Source : Archer ReviewSource : Archer Review Variable decelerations suggest an interruption of oxygenation at the level of the umbilical cord, where cord vessels may be compressed. Physiologically, the thin-walled umbilical vein is occluded during umbilical cord compression in umbilical cord prolapse or oligohydramnios. The nurse should execute actions immediately, such as summoning help and relieving pressure off the umbilical cord. Having the client reposition is essential; the nurse may have the client assume positions such as knee-chest position, Trendelenburg position and hips elevated with pillows, with side-lying position maintained. Last Updated - 27, Dec 2022

The nurse is assessing a client who is five months pregnant and has a son born at 40 weeks of gestation and a daughter born at 33 weeks of gestation. It would be correct for the nurse to document this client's GTPAL as A. G4-T1-P1-A0-L2 B. G3-T1-P1-A0-L2 C. G3-T1-P1-A0-L3 D. G3-T2-P0-A0-L2 Submit Answer

Explanation This client is currently pregnant and has had two other pregnancies prior to the current one. This would make her a gravida three (G3). She has delivered one child at 40 weeks of gestation, which is one term birth (T1). She delivered one child at 33 weeks (P1). She has had two live births (L2). Additional Info A method for calculating gravida and para is to separate pregnancies and their outcome using the acronym GTPAL: G = gravida, T = term, P = preterm, A = abortions, and L = living children. G = pregnancies or gravida, T = term pregnancies delivered, P = preterm pregnancies delivered, A = abortions (spontaneous and induced), and L = living children Term pregnancies are any pregnancy 37 weeks or greater; preterm is any pregnancy 20-36 weeks; abortions are any abortions spontaneous or induced prior to 20 weeks. Last Updated - 08, May 2022


Related study sets

International Marketing Chapter 8 SmartBook Assignment

View Set

PSYCH 105 John Lothes final exam review

View Set

Combo with "marketing test 3" and 12 others

View Set

Acceleration, Forces, & Newton's Laws Review

View Set

Acting Ethically and Socially Responsible

View Set

AP Bio Chapter 16 Practice: The Molecular Basis of Inheritance

View Set

FRL CH 7 Bonds and Bond Valuation

View Set